CH. 46-49 Control>Find

Réussis tes devoirs et examens dès maintenant avec Quizwiz!

In order to maintain adequate perfusion to the vital organs of the body a mean arterial pressure (MAP) of 60 mmHg must be maintained. MAP is a calculation of average blood pressure using systolic and diastolic blood pressure measurements. MAP is calculated as follows: A) MAP = DBP + (SBP-DBP)/3 B) MAP = SBP + (SBP-DBP)/2 C) MAP = DBP + (SBP-DBP)/2 D) MAP = SBP + (SBP-DBP)/3

A) MAP = DBP + (SBP-DBP)/3

Which of the following devices are used to suction stomach contents in the event of an overdose or poisoning? A) NG or OG tube B) CPAP or BiPAP C) Hickman tube D) AV shunt

A) NG or OG tube

The mammalian diving reflex: A) shunts blood to the heart and brain. B) results in elevation of PaCO2. C) shunts blood from the core to the abdominal viscera. D) causes involuntary apnea.

A) shunts blood to the heart and brain.

The trauma system is based on the principle that serious trauma is a: A) surgical disease B) medical disease C) psychological disease D) mental disease

A) surgical disease

You are caring for a patient whom you suspect has had a stroke. She is unable to form her words properly making it difficult for you to understand her. Which of the following would describe the condition of her speech? A) Aphasia B) Ataxia C) Dysphagia D) Hypophasia

Answer: A - Aphasia

Which of the following would be considered a bloodborne pathogen? A) Hepatitis B B) Tuberculosis C) Measles D) Hepatitis A

Answer: A - Hepatitis B

Pelvic inflammatory disease is BEST described as a(n): A) Infection of the female reproductive organs. B) Pain experienced as a female ovulates. C) Particularly painful menstrual period. D) Ovarian cyst that has ruptured.

Answer: A - Infection of the female reproductive organs.

You have a patient who was possibly bitten by a rabid dog. Which of the following is CORRECT about rabies? A) Infection can lead to Bell's palsy, headaches, and meningitis. B) The virus travels along nerves from the bite to the brain. C) It has been identified as a potential weapon of bioterrorism. D) The virus causes inflammation of brain tissue.

Answer: B - The virus travels along nerves from the bite to the brain.

Which of the following is TRUE regarding NKDC relative to DKA? A) Blood glucose is lower than the patient with DKA. B) There is a lack of ketone formation. C) Alkalosis results from inadequate tissue perfusion. D) All of the above are true.

Answer: B - There is a lack of ketone formation.

Your 59-year-old patient is suffering an acute myocardial infarction and is a candidate for fibrinolytic therapy. How do fibrinolytics work? A) They cause platelets to become sticky. B) They break down blood clots. C) They reduce cardiac dysrhythmias. D) They increase the blood pressure.

Answer: B - They break down blood clots.

Angulated knee dislocations can be immobilized with: A) A bipolar traction splint B) Two medium rigid splints C) A pillow splint D) A unipolar splint

Answer: B - Two medium rigid splints

Which structure, when injured, causes the most rapid death? A) Spleen B) Vena cava C) Large intestine D) Stomach

Answer: B - Vena cava

The dose of glucagon when given IM is: A) 50 mg IV. B) 5 mg IV. C) 1 mg IM. D) 10 mg IM.

Answer: C - 1 mg IM.

What is the purpose of being trained with the National Disaster Medical​ System? A. to understand how to contact federal authorities during a disaster B. to receive joint training with other teams to respond to disasters outside your jurisdiction C. to receive training on how to care for patients during a hazardous materials incident D. to understand how other jurisdictions respond to​ mass-casualty incidents

B. You may become involved in an MCI outside your jurisdiction as part of mutual aid or because you opt to be active in the National Disaster Medical System​ (NDMS). NDMS is a section of the U.S. Department of Health and Human Services that is responsible for managing the federal medical response to an emergency or disaster. It consists of locally based teams that respond jointly where disaster medical assistance is required.

Your patient, who is expecting her second child in two days, reports a mucous-like vaginal discharge that was pink in color prior to your arrival. The patient is experiencing abdominal cramping intermittently. You suspect this is: A) Preeclampsia B) Placenta previa C) A mucous plug D) A threatened miscarriage

C) A mucous plug

You are called for an elderly man with an apparent broken hip. The patient tells you that he has osteoporosis and type 2 diabetes. He denies falling and cannot remember any trauma. You should suspect: A) Decreased range of motion B) Osteoarthritis C) A pathological fracture D) An associated spinal injury

C) A pathological fracture

You are teaching an EMT class about the role of an EMS response to a terror attack involving weapons of mass destruction (WMD). A student asks you what constitutes a WMD. You should respond by stating that a WMD is any: A) Biological agent that will kill but not destroy property B) Weapon used to create fear and panic among the general public C) Agent that is used to cause indiscriminate death and destruction D) Agent that is intended to kill a specific segment of the population

C) Agent that is used to cause indiscriminate death and destruction

What medication is indicated for reversal of narcotic intoxication? A) Valium B) Thiamine C) Narcan D) Romazicon

C) Narcan

An elderly patient presenting with pneumonia may experience no fever but frequently will complain of: A) Nausea and vomiting B) Profuse sweating C) Pain referred to the abdomen D) Pulmonary edema

C) Pain referred to the abdomen

Heat flowing from a surface of greater temperature to one of lesser temperature by direct contact BEST explains the concept of: A) radiation. B) convection. C) conduction. D) evaporation.

C) conduction.

You are on the scene of an accident where patients are exhibiting the following​ symptoms: dyspnea,​ tachypnea, coughing, pulmonary​ edema, chest​ pain, shock, and more generalized signs and symptoms such as​ headache, nausea and​ vomiting, altered mental​ status, and seizures. What type of chemical weapon might be​ suspected?

Choking agents

Which of the following is MOST likely to provide toxic exposure through inhalation? A) Wasps B) Prescription medications C) Over-the-counter medications D) Chlorine

D) Chlorine

All of the following are common causes of seizures in the elderly EXCEPT: A) Epilepsy B) Stroke C) Brain injury D) Syncope

D) Syncope

All of the following are risk factors for elder abuse EXCEPT: A) The caregiver has other responsibilities besides caring for the elder patient B) The patient is wheelchair bound and incontinent C) The elderly patient is destitute and must depend on family for housing D) The patient is 68-years-old and plays golf every day

D) The patient is 68 years old and plays gold every day

Which of the following physical assessment findings would make your patient a candidate for rapid transport to a trauma center? A) Respiratory rate of 24 B) Pulse rate of 110 C) Systolic blood pressure of 100 D) Unstable pelvic girdle

D) Unstable pelvic girdle

A behavioral emergency is one in which the patient presents a danger to: A) the general public. B) family members. C) himself/herself. D) all of the above

D) all of the above

You are treating a patient who has an intense irrational fear of needles and is refusing consent for IV therapy. You should: A) start the IV and administer a sedative that will calm the patient. B) withhold any further treatment. C) do not transport the patient if consent is not obtained for the IV. D) respect the patient's autonomy and not start the line.

D) respect the patient's autonomy and not start the line.

he body's attempt at maintaining a core temperature is known as: A) compensation. B) metabolism. C) aerobic metabolism. D) thermoregulation.

D) thermoregulation.

The patient with hypothermia should be transported: A) in Trendelenburg positioning. B) with the head and shoulders elevated to 45-degrees. C) rapidly to a Level III trauma facility. D) with gentle handling to prevent ventricular fibrillation from occurring.

D) with gentle handling to prevent ventricular fibrillation from occurring.

very flammable

If the red diamond on a National Fire Protection Association placard contains a "4," the EMT should recognize that the material contained within is:

health hazard.

On the National Fire Protection Association hazardous materials classification placard, the blue diamond represents:

Hazmat specialist

On the scene of a hazardous materials emergency, which one of the following individuals is best suited to provide command of the incident?

You are participating in a training exercise for an attack by a weapon of mass destruction involving the release of a nerve agent. In the triage​ area, you are presented with a patient who responds to painful stimuli and has an open airway. He is breathing eight times a minute and has a weak radial pulse. He also has wheezing and pinpoint pupils. In his pocket he has a​ metered-dose inhaler containing albuterol. When caring for this​ patient, you should​ FIRST:

administer positive pressure ventilation.

.

.

.

..

In general, the use of ________ restraints is discouraged. A) hard B) chemical C) soft D) leather

A) hard

Why do vomiting agents cause​ vomiting? A. because of the accompanying coughing and gagging B. because they directly affect the digestive track C. because they effect the areas of the nervous system that cause vomiting D. because of the accompanying parasympathetic stimulation

A.

The current treatment for fibromylgia is: A) Antidepressants. B) Allopurinol. C) Sulfonamide antibiotics. D) Anti-inflammatory drugs.

Answer: A - Antidepressants.

What is the purpose of the windshield​ assessment? A. to identify potential scene hazards B. to safely identify hazardous materials C. to safely direct​ on-scene operations of an MCI D. to triage multiple patients at once

A. The purpose of the windshield assessment is to identify potential scene hazards and to be able to relay that information to dispatch.

The neurotransmitter ________ is responsible for contraction of the muscle. A) Acetylcholinesterase B) Myoglobin C) Acetylcholine D) Phosphokinase

Answer: C - Acetylcholine

Four-digit identification number

An intoxicated driver drove his car into the side of a slow-moving train, hitting the tanker car. To get information on what is contained in the tanker, you are going to call CHEMTREC, Inc. What piece of information is it critical to provide this agency so they can help identify the material inside the tanker car?

For a normal pregnancy, within what structure MUST the sperm and egg come together for fertilization? A) Vagina B) Fallopian tube C) Ovary D) Upper uterus

Answer: B - Fallopian tube

Which of the following is a biologic​ agent?

Anthrax

Triage, vital signs, and medical histories

At the scene of a hazardous materials emergency, you have been assigned to the cold zone. In this zone, which one of the following activities will you be performing or assisting with?

A foreign substance that elicits the response of the immune system is called: A) histamine. B) antigen. C) antibody. D) stimulant.

B

Which of the following body response is aimed at limiting exposure to an antigen? A) Hives and urticaria B) Slight bronchoconstriction C) Increased circulation to the brain D) Diaphoresis and anxiety

B

Which of the following is an example of an implanted venous access port? A) Broviac B) MediPort C) Hickman D) Groshong

B) MediPort

Associated factors in determining the rate of heat exchange in the environment include: A) ambient temperature of the body. B) humidity and wind. C) percentage of carbon dioxide in the atmosphere. D) carbon emissions from vehicles.

B) humidity and wind.

Which medication would be the BEST choice for reducing the bronchospasm and laryngeal edema associated with anaphylaxis? A) Solu-Medrol B) Oxygen C) Albuterol D) Dexamethasone

C

Immediately following fertilization in the fallopian tube, the ovum will: A) Rupture from the ovary B) Immediately implant in the uterus C) Begin division immediately D) Be discharged with menstruation

C) Begin division immediately

All of the following are examples of narcotics EXCEPT: A) Heroin B) Codeine C) Crack D) Morphine

C) Crack

According to the​ CDC, which category of bioagents is Staphylococcal enterotoxin​ B?

Category B

Vesicants and asphyxiants are examples of which type of​ WMD?

Chemical weapons

Car bombs or pipe bombs are examples of which type of​ WMD?

Conventional explosives

You are responding to a patient who has a weepy rash that itches intensely. She states that a couple of days ago she cleaned out a wooded area behind her home. How should you manage this apparent allergic reaction? A) Provide oxygen via NRB and 100 mg of Benadryl IM. B) Benadryl IV and 0.3 mg of 1:1000 epinephrine. C) Benadryl and 0.5 mg of 1:10,000 epinephrine. D) None of the above are appropriate treatments.

D

Panic attacks MOST closely resemble: A) dementia. B) schizophrenia. C) delirium. D) cardiac and respiratory conditions.

D) cardiac and respiratory conditions.

A mnemonic to help remember the effects associated with nerve agent poisoning​ is:

DUMBELS.

In chemical​ events, what must you do​ first?

Make sure patients are decontaminated.

You are on the scene of an accident where patients are exhibiting the following​ symptoms: diarrhea,​ urination, miosis,​ bradycardia, emesis,​ lacrimation, and salivation. What type of chemical weapon might be​ suspected?

Nerve agents

A technique that the terrorist may use which involves both conventional and WMD characteristics​ is/are:

Radiological dispersion device

Instruct the patient to remain still and wait for the radiation safety officer.

The driver of a truck carrying a radioactive substance has accidentally come into contact with the material. You are first on the scene and a quick observation indicates that he still has the powder on his arm. Which one of the following should you do immediately?

What chemical agent mostly causes irritation of the upper respiratory tract and the​ eyes? Vomiting agents

Vomiting agents

Chemicals that cause a disruption in the metabolism of oxygen by the cell like cyanide are​ called:

blood agents.

For the patient exposed to a nerve​ agent, which one of the following signs should the AEMT​ expect?

increased salivation

A chemical agent that interferes with normal functioning of nervous transmission is known as​ a(n):

nerve agent.

Biological weapons are often hard to identify because​ they:

often do not have any immediate symptoms.

Chemical agents that cause airway​ irritation, inflammation, and pulmonary edema​ include:

phosgene.

As your rescue unit arrives at the scene of a possible terrorist​ explosion, one of your primary concerns should​ be:

the possibility of a secondary explosive device.

Which of the following would be your likely initial medication treatment when a patient has been exposed to a small amount of an experimental nerve​ agent?

2 mg of atropine and 600 mg of pralidoxime chloride

Diphenhydramine is administered in anaphylaxis because it: A) blocks histamine receptors. B) causes vasoconstriction. C) treats bronchospasm. D) relaxes smooth vessels.

A

What cells release histamine when they recognize antigen-antibody complexes? A) Mast cells B) Helper T cells C) Antigens D) Lymphocytes

A

Which of the following is CORRECT regarding corticosteroids? A) They impair the body's ability to respond to disease. B) They increase the number of red blood cells produced. C) They suppress cancer cell reproduction. D) They lower the body's risk for infection.

A

Your patient is having a reaction after taking a new ACE inhibitor for hypertension. Which of the following would you MOST likely see? A) Angioedema B) Hypertension C) Diaphoresis D) Hyperthermia

A

You have been called to a residence for a 31-year-old patient complaining of confusion, nausea, vomiting, and a headache. He states that this started this morning and has been getting worse all day. Despite lying in bed and resting all day, he is more fatigued than before. Which of the following statements made by the patient would the AEMT recognize as MOST important in relation to the patient's present complaint? A) "I just turned the furnace on last night." B) "I have not been sleeping well the past few nights." C) "I tried some raw seafood last week." D) "My wife is taking penicillin, to which I am allergic."

A) "I just turned the furnace on last night."

The AEMT correctly defines a multiple-casualty incident (MCI) when he states: A) "It is any event that places an excessive demand on rescue personnel and equipment." B) "It occurs any time that five or more people are injured and require transport to different hospitals." C) "It occurs any time the incident requires the medical director to leave the hospital and direct activities on-scene." D) "It occurs when the number of patients requiring care is greater than 10."

A) "It is any event that places an excessive demand on rescue personnel and equipment."

Which one of the following statements shows that the AEMT has a proper understanding of basic operations at the scene of a hazardous materials emergency? A) "Life or health should never be risked if the threat is only to the environment." B) "If you cannot see or smell a chemical, the chance of one being present is very low." C) "It is critical to remove the patients as quickly as possible from the incident site." D) "Identification of the material is not important, since the approach is the same for all materials."

A) "Life or health should never be risked if the threat is only to the environment."

You must assess the pupils of a five-year-old who fell off a bed while playing. Which of the following statements would be MOST appropriate prior to performing this assessment? A) "Open your eyes so that I can look at your pretty eyes." B) "I am going to look at your pupils with my light." C) "I am going to use this light to look into your eyes." D) "I need to test your visual acuity by shining a light in your eyes."

A) "Open your eyes so that I can look at your pretty eyes."

An alert and oriented 18-year-old woman has threatened to hurt herself, but is refusing transport to the hospital. She is calm and states that she knows her rights and taking her would constitute false imprisonment. Her family is present and wants her to go to the hospital for help. Given the situation and the patient's threat, what is your BEST course of action? A) Have the patient sign a refusal that is witnessed by family. B) Physically restrain the patient for transport. C) Contact law enforcement for assistance. D) Transport the patient on her father's consent.

C) Contact law enforcement for assistance.

With a core temperature of 28°C (82.4°F) a patient will display all of the following signs or symptoms EXCEPT: A) Loss of consciousness B) Decline in oxygen consumption and pulse C) Decreased muscle rigidity D) Marked bradypnea

C) Decreased muscle rigidity

Which of the following is NOT a route of ingestion that may result in a toxicological emergency? A) Ingestion B) Absorption C) Diffusion D) Inhalation

C) Diffusion

You respond to a patient who has sustained a fall. In assessing the scene, you MUST include the pathway of transmission, or the: A) Distance of the fall B) Surface the patient fell on C) Direction in which kinetic energy is transmitted through the body D) Path of the patient's body over the course of the fall

C) Direction in which kinetic energy is transmitted through the body

You are treating a patient with an abdominal evisceration. You should: A) Replace the protruding organs, cover with a sterile dressing moistened with sterile saline, and an occlusive dressing B) Do not replace the protruding organs, cover with a dry sterile dressing, and an occlusive dressing C) Do not replace the protruding organs, cover with a sterile dressing moistened with sterile saline, and an occlusive dressing D) Replace the protruding organs, cover with a dry sterile dressing, and an occlusive dressing

C) Do not replace the protruding organs, cover with a sterile dressing moistened with sterile saline, and an occlusive dressing

You are called for an elderly woman who fell. She called 911 saying she hurt her head when she fell. When you arrive, her husband tells you that she is fine and doesn't need you. You can see her in the background and she appears to be bleeding from a head wound and is crying. You should suspect: A) Psychological abuse B) She is not really injured C) Domestic abuse D) Sexual abuse

C) Domestic abuse

You should assume any female of childbearing age who is suffering from lower abdominal pain and a missed period to have: A) Pelvic inflammatory disease B) Acute appendicitis C) Ectopic pregnancy D) Spontaneous abortion

C) Ectopic pregnancy

A small truck has gone over an embankment. The driver has suffered a head injury and is confused, but is able to tell you that he is hauling N-Acetyl-para-Aminophenol. Which one of the following resources provides the AEMT with the BEST initial information regarding this chemical? A) TREC-CHEM EMS Guidebook B) Truck driver C) Emergency Resource Guidebook D) Medical direction

C) Emergency Resource Guidebook

If the patient in labor complains of the urge to push, you should: A) Transport immediately as delivery is imminent B) Cover the vaginal opening with a bulky dressing and transport C) Examine the vaginal area for crowning D) Position the patient in a left-lateral recumbent position

C) Examine the vaginal area for crowning

Weapons of mass destruction include all of the following EXCEPT: A) Biological B) Chemical C) Experimental D) Nuclear

C) Experimental

Rescue workers are preparing to remove the windshield and roof from a car to extricate a patient who is trapped. Before the rescue commences, which one of the following should the AEMT do FIRST? A) Cover the patient with a heavy tarp for protection B) Wet the car down with a fire hose to prevent an accidental fire C) Explain to the patient what is taking place D) Lock all four doors and roll up the windows

C) Explain to the patient what is taking place

A concerned parent asks you which vaccine will decrease the risk of her child getting epiglottitis. Which vaccine would you tell her? A) Influenza B) MMR C) Hib D) Hepatitis

C) Hib

What is the MOST common complication associated with a tracheostomy tube? A) Air leakage around the cuff B) Tube dislodgement C) Mucus obstructing the tube D) Infection at the stoma

C) Mucos obstructing the tube

All of the following are signs of preeclampsia EXCEPT: A) Visual disturbances B) Epigastric pain C) Muscle tetany D) Peripheral edema

C) Muscle tetany

You are called for an elderly man who "is not acting right". The man's son says that they were just leaving for a short vacation but grandpa is sick and needs to go to the hospital. During your assessment, you notice numerous bed sores on the patient's backside and bruises on his arms and legs. Other than his physical condition, he appears to be okay. You should: A) Tell the son the you know he is trying to dump his father so he can leave on vacation B) Ask the son about the bruises on grandpa and how they got there C) Package the patient for transport to the hospital and notify the staff of your suspicions D) Treat the patient's bedsores and turn him so they won't get any worse

C) Package the patient for transport to the hospital and notify the staff of your suspicions

A 78-year-old male pedestrian was struck by a car at a convenience store. The patient is conscious but confused and is suffering from multisystem trauma. Which one of the following represents the BEST sequence of care that can be provided? A) Primary assessment, oxygen, secondary assessment, and all other care en route to the hospital B) Primary assessment, oxygen, secondary assessment, cervical collar, and all other care en route to the hospital C) Primary assessment, oxygen, secondary assessment immobilization, and all other care en route to the hospital D) Primary assessment, immobilization, and all other care en route to the hospital

C) Primary assessment, oxygen, secondary assessment, immobilization, and all other care en route to the hospital

Personal protection against MOST common biological agents includes all of the following EXCEPT: A) Rigorous and frequent hand washing B) Disposable gloves C) Properly fitting surgical mask D) Properly fitting HEPA mask

C) Properly fitting surgical mask

You are called for an elderly man with what appears to be pleuritic chest pain. Further assessment shows he has pain in the lower leg with increased calf pain when he flexes his toes upwards. You should suspect: A) Acute pulmonary edema B) Referred pain from appendicitis C) Pulmonary embolism D) Aortic aneurysm

C) Pulmonary embolism

As you observe a chemical emergency from a distance, the chemical identified is said to have a "highly volatile" state. Based on this information, you know that the liquid will: A) Slowly change from liquid to gas over hours B) Be able to rapidly disperse because of the low surface tension C) Rapidly change to a gaseous state D) Explode when exposed to open flame

C) Rapidly change to a gaseous state

You are caring for a two-year-old child who has fallen from the second story deck. The child is crying and you are trying to complete an assessment on the baby. The father is worried sick about his son. Which of the following should you do in this situation? A) Call for law enforcement assistance in case the father becomes violent B) Have the father leave the room so you can complete your assessment C) Reassure him and have him get the child's favorite toy to take to the hospital D) Tell the father to calm down that the boy will be just fine

C) Reassure him and have him get the child's favorite toy to take to the hospital

When dealing with a patient with a mental and emotional impairment, it is important that you: A) Have law enforcement personnel standing by B) Restrain the patient before they become aggressive C) Recognize that these patients can be a threat to your safety D) Are prepared to chemically sedate the patient if necessary

C) Recognize that these patients can be a threat to your safety

The rescue captain has asked that you disconnect a car's battery to shut down all power to the vehicle completely. How should you do this safely? A) Cut the positive battery cable, and then remove the negative cable B) Disconnect the positive battery cable first C) Remove the negative battery cable first D) Remove the negative and positive cables simultaneously

C) Remove the negative battery cable first

You are on a scene with a patient who up until a few seconds ago was calm and compliant. Now he is brandishing a knife and threatening to kill you and your partner if you do not immediately leave. The man is intoxicated and police have been notified but have not yet arrived. Which one of the following is your BEST course of action? A) Attempt to "talk down" the patient. B) Encourage the man's wife to calm him down. C) Remove yourself, your partner, and family from the house. D) Immediately physically restrain the patient.

C) Remove yourself, your partner, and family from the house.

You are on-scene with a two-year-old girl. Which of the following would MOST likely indicate neglect or abuse? A) She cries as you attempt to assess her B) She is dirty and has bad breath C) She has bruises on her legs and torso D) She is wearing a dirty diaper

C) She has bruises on her legs and torso

You respond for a 66-year-old man complaining of severe pain on the back of his neck and right shoulder. You examine the area and see fluid filled blisters in a horizontal line across his shoulder. You should suspect: A) Kyphosis B) Spondylosis C) Shingles D) Chicken pox

C) Shingles

Pain associated with an ectopic pregnancy may be referred to the: A) Pelvic girdle B) Lower back C) Shoulder D) Umbilicus

C) Shoulder

Exposure to chemical vesicants (mustard) agents results in: A) Immediate death B) Pulmonary edema within five minutes of exposure C) Slowly developing symptoms D) Rapid blistering action to exposed surfaces

C) Slowly developing symptoms

While palpating your patient's neck, you notice a "rice crispy" feel. This is known as: A) Subcutaneous edema B) Rebound tenderness C) Subcutaneous emphysema D) Crepitation

C) Subcutaneous emphysema

The grandfather of an alert and oriented two-year-old girl found her on the floor playing with the pills from his open bottle of digoxin (heart medication that slows the rate). He cannot remember how many pills were in the bottle, but he thinks that some are missing and she may have ingested them thinking that they were candy. Assessment reveals her airway to be open and breathing adequate. Her pulse is within normal limits for a child her age and skin warm and dry. Her grandfather is not sure of how long ago she may have taken the pills. Given the assessment findings, which of the following should the AEMT assume? A) The child did not take any Digoxin. B) Not enough Digoxin was taken to harm the child. C) The Digoxin has yet to absorb into the body. D) The child spit the pills out prior to swallowing.

C) The Digoxin has yet to absorb into the body.

All of the following is correct regarding premature infants EXCEPT: A) The thermoregulatory mechanism is immature B) Glycogen stores may be inadequate C) The cranium protects the fragile brain tissue D) The lungs are underdeveloped lacking surfactant

C) The cranium protects the fragile brain tissue

A train carrying radioactive materials has derailed. The radiation safety officer informs you that you will be receiving a 31-year-old man who was exposed to the radioactive material. As such, you should recognize that: A) The patient will be vomiting and will have lost his hair B) The patient will require immediate decontamination C) The patient represents no contamination threat to you D) You will need special clothing to protect yourself from radiation

C) The patient represents no contamination threat to you

All of the following communication techniques are appropriate when dealing with the emotionally or behaviorally disturbed patient EXCEPT: A) Being supportive and empathetic B) Encouraging the patient to tell his story C) Using yes-or-no style questions D) Maintaining a safe distance

C) Using yes-or-no style questions

The muscular organ in which a baby develops during pregnancy is called the: A) Fetal membrane B) Placenta C) Uterus D) Amniotic sac

C) Uterus

A ________ is a chemical agent that causes blisters to form where exposure occurs. A) Nerve agent B) Phosphorus C) Vesicant D) Surfactant

C) Vesicant

A class of diseases that includes the ebola virus are known as: A) Pneumonic agents B) A febrile hemolytic agents C) Viral hemmorhagic fever D) Asphyxiating agents

C) Viral hemmorhagic fever

Croup is a: A) Bacterial infection of the upper airway B) Bacterial infection of the lower airway C) Viral infection of the lower airway D) Viral infection of the upper airway

C) Viral infection of the lower airway

You are caring for a three-year-old girl who was involved in a serious motor vehicle crash. Which of the following is TRUE regarding care for this child? A) Transport the child in the car seat with padding around the head and torso B) If no sign of spinal injury, remove the child from the car seat and transport normally C) You should remove the child from the car seat maintaining spinal immobilization D) Have the parent hold the child while both are immobilized on a long spine board

C) You should remove the child from the car seat maintaining spinal immobilization

Management of aspirin (salicylate) overdose should include administration of: A) albuterol. B) sodium bicarbonate. C) activated charcoal. D) syrup of ipecac.

C) activated charcoal.

A compulsive and overwhelming dependence on a drug is known as: A) habituation. B) tolerance. C) addiction. D) withdrawal.

C) addiction.

A substance that will neutralize a specific toxin or counteract its effects is known as: A) absorbents. B) emetic. C) antidotes. D) cathartic.

C) antidotes.

You are providing a "stand-by" service at the county fair. About midmorning, a 50-year-old male patient approaches your tent and states that he was bitten in the right shoulder by some sort of insect and the pain is tremendous. Assessment reveals his shoulder to have a large reddened area with a bite mark in the middle. The skin is warm to the touch. Your care of this patient should include: A) massaging the site gently. B) scrubbing the site with soap and sterile water. C) applying a cold pack to the site. D) elevating the right hand.

C) applying a cold pack to the site.

While examining a patient experiencing a behavioral emergency, a family member arrives and the patient becomes very agitated at their presence. You should: A) allow the patient and family time to become acquainted. B) restrain the patient to prevent possible injury to the family member. C) ask the family member to leave and complete your exam. D) allow the family member to remain at a distance.

C) ask the family member to leave and complete your exam.

A situation in which a person's behavior is so unusual that it alarms another person or requires intervention is a(n): A) psychological crisis. B) emotional dilemma. C) behavioral emergency. D) critical affective period.

C) behavioral emergency.

A patient presents with inflated self-esteem, decreased need for sleep, and racing and delusional thoughts. This patient MOST likely suffers from: A) conversion disorder. B) schizophrenia. C) bipolar disorder. D) depression.

C) bipolar disorder.

Hyperthermia is a result of: A) heat transfer from the environment to the body. B) excessive thermogenesis. C) both A and B D) none of the above

C) both A and B

The MOST important factor in treating any behavioral problem is: A) collecting evidence for examination at the hospital. B) ensuring all patients are restrained prior to transport. C) ensuring scene safety prior to approaching the patient. D) obtaining a detailed examination of all patients.

C) ensuring scene safety prior to approaching the patient.

You are called to a football game for a 15-year-old boy who is lying on the field, conscious and alert. It is August and the patient is complaining of weakness and cramping to his abdomen. He is breathing well and his skin color is normal and it is warm and moist to touch. Vital signs are normal. Treatment should include: A) rapid cooling with ice packs to major pulse sites. B) rapid fluid infusion of NaCl 0.9 percent to replace lost sodium. C) gentle cooling by removing excess clothing and equipment. D) high flow oxygen and aggressive fluid replacement.

C) gentle cooling by removing excess clothing and equipment.

The pathophysiology of toxic inhalation involves: A) bronchoconstriction and dispersal of surfactant. B) pulmonary hypertension, alveolar atelectasis, and destruction of cilia. C) irritation, edema, and destruction of alveolar tissue. D) bronchodilation and destruction of cilia.

C) irritation, edema, and destruction of alveolar tissue.

You find a patient who is homeless behind a convenience store in the summer months. The patient is flushed, diaphoretic, and warm to touch. Based on this information, you know the patient: A) probably has a history of alcohol and drug abuse. B) is unable to compensate for the exposure. C) is dissipating heat. D) all of the above are true

C) is dissipating heat.

In your response district, you have a subject who walks around in a white plastic suit collecting small pieces of plastic in a shopping cart. You observe that he frequently talks to himself and the things he picks up off the ground. He presents not danger to himself or others as he seems to live in his own world. This patient is probably suffering from: A) delirium. B) bipolar disorder. C) schizophrenia. D) anxiety neurosis.

C) schizophrenia.

A weapon of terrorism that is designed primarily to start a fire is known as​ a(n): A. conventional explosive B. dirty bomb C. incendiary device D. small arms

C.

Exposure to which type of chemical agent used in terrorism produces​ burns, pain, and small vesicles that eventually become larger blisters on the​ skin? A. blood agents B. vomiting agents C. vesicants D. nerve agents

C.

This type of hazard involves splitting or fusing atoms with release of massive amounts of energy along with​ radiation: A. hazardous material hazards B. radiologic hazards C. nuclear hazards D. radioactive emission hazards

C.

Which of the following is considered an ominous sign late in anaphylaxis? A) Hypotension B) Tachycardia C) Hypertension D) Bradycardia

D

Which of the following mediators is responsible for many of the manifestations of anaphylaxis? A) Antibodies B) Basophils C) Macrophages D) Histamine

D

Which of the following situations indicate a potential terrorist​ attack? A. reports of multiple patient casualties in a small neighborhood following an F4 tornado B. a fully involved structure fire involving only a single structure C. a motor vehicle collision involving several automobiles including a tanker truck on the interstate D. reports of simultaneous fires involving important governmental structures throughout the city

D

On the scene of a motor-vehicle collision, the driver is sitting inside the car with all windows up. When you attempt to open the doors, you find that they are locked. Given this situation, which one of the following statements made to the patient is MOST appropriate? A) "Please remain still. I am calling for the rescue truck to remove the door of your car." B) "Very carefully, with as little movement as possible, please roll down the window." C) "Please shield your eyes. I am going to break the window so we can open the door." D) "Without moving your head or neck, please try to unlock the door."

D) "Without moving your head or neck, please try to unlock the door."

You have been called to a residence for a spider bite. On-scene the homeowner states that he was moving some boxes in his garage when he accidentally grabbed a spider and it bit him. He killed the spider and shows you a brown-colored spider with a "violin shape" on its back. He then states that the bite does not hurt and he sees no need to go to the hospital. As a knowledgeable AEMT, you should state: A) "You really need to go because the bite of that spider does not heal very well and will put you at risk for infection." B) "Let us take you to the hospital. They have anti-venom that will allow this to heal and make you resistant to future bites." C) "You should be okay by refusing transport, but make sure to wash the bite site daily with soap and water." D) "Since the bite of that spider generally causes immediate pain and swelling, you probably can go without additional medical attention."

A) "You really need to go because the bite of that spider does not heal very well and will put you at risk for infection."

You are participating in a training exercise for an attack by a weapon of mass destruction involving the release of a nerve agent. In the triage area, you are presented with a patient who responds to painful stimuli and has an open airway. He is breathing eight times a minute and has a weak radial pulse. He also has wheezing and pinpoint pupils. In his pocket he has a metered-dose inhaler containing albuterol. When caring for this patient, you should FIRST: A) Administer positive pressure ventilation B) Obtain vital signs, including the temperature C) Look for an injury causing shock D) Administer albuterol with the metered-dose inhaler

A) Administer positive pressure ventilation

Which one of the following chemical agents can kill a person MOST quickly? A) Aerosolized agent inhaled into the lungs B) Liquid vesicant that is splashed on the arm C) Solid agent that contact the patient's torso D) Liquid agent that is splashed into the eyes

A) Aerosolized agent inhaled into the lungs

A worker has been exposed to beta rays given off by a radioactive material. Which one of the following should you do FIRST? A) Assess his level of consciousness and ABCs B) Determine the exact material he was exposed to C) Decontaminate the patient D) Quickly assess for any radiation burns

A) Assess his level of consciousness and ABCs

The dysrhythmia commonly encountered as a result of the aging process is: A) Atrial fibrillation B) Sinus bradycardia C) Ventricular tachycardia D) Idioventricular

A) Atrial fibrillation

Which of the following would be an example of psychological abuse? A) Berating an elderly patient for incontinence B) Sedating a patient to keep him quiet C) Restraining a patient to prevent his wandering D) Distributing sexually explicit photos of a patient

A) Berating an elderly patient for incontinence

You are assessing a three-year-old child for possible injuries after the child fell down a flight of stairs. Which of the following would raise your suspicion that the child may be a victim of physical abuse? A) Bruises are found on his chest and abdomen B) The child is fearful and appears malnourished C) The child cries when you palpate his arm D) Several bruises are located on his knees and shins

A) Bruises are found on his chest and abdomen

A direct blow to the chest at a vulnerable point in the cardiac cycle, leading to cardiac arrest is known as: A) Commotio cordis B) ALTE C) Livor mortis D) Ventricular hypertrophy

A) Commotio cordis

While he is moving a container of a radioactive liquid, some of the liquid splashed onto the technician's arm. This situation is BEST be described as: A) Contamination B) Exposure C) Exposure with contamination D) Exposure with beta injury

A) Contamination

The AEMT would recognize which one of the following statements as the BEST example of the use of a weapon of mass destruction? A) Death of 49 people in a crowded mall following intentional infection with unknown bacteria B) Death of 57 following nighttime tornadoes that have struck a suburban area C) Accidental release of radioactive gas from a nuclear power plant that has impacted thousands of people D) Death of 25 following an explosion in an industrial plant caused by faulty wiring

A) Death of 49 people in a crowded mall following intentional infection with unknown bacteria

A patient involved in a motor vehicle collision is complaining of neck pain. Which of the following pieces of information should cause the AEMT to have a high index of suspicion that the patient may have suffered more significant injuries beyond the complaint of neck pain? A) Death of the car's driver B) Complaint of head and neck pain C) History of heart problems D) No airbags in the car

A) Death of the car's driver

Which of the following would be the BEST treatment for a four-year-old having an acute asthma attack? A) Deliver humidified oxygen, and a beta2-agonist by small-volume nebulizer B) Provide oxygen and a fluid bolus at 10 mL per kilogram up to 200 cc's C) Deliver oxygen via a nasal cannula and administer racemic epinephrine via nebulizer D) Provide oxygen and administer SQ epinephrine at a dosage of 0.10 mg/kg

A) Deliver humidified oxygen, and a beta 2-agonist by small-volume nebulizer

In preparation for delivery, you should do all of the following EXCEPT: A) Have tepid water available for cleansing the fetus B) Deliver an IV fluid bolus of isotonic fluids C) Drape the mother with sterile sheets if time permits D) Put on sterile gloves, mask, and gown

A) Have tepid water available for cleansing the fetus

An 89-year-old woman with dementia who is being cared for by her daughter after a pipe-bomb explosion and exhibits deformity to her left upper arm. As you attempt to splint her arm, she continually screams and pulls away from you. After several attempts, your BEST course of action should be to: A) Have the daughter help apply the splint B) Gently restrain the patient and apply the splint C) Contact advanced life support for assistance D) Transport without a splint and document that splinting was attempted

A) Have the daughter help apply the splint

You are called for a motor vehicle collisioninvolving two elderly women. You are concerned because you know that elderly patients are often prescribed blood thinner for cardiac problems. This can be a problem because? A) Impaired blood clotting can exacerbate blood loss B) It means they most likely have cardiac problems C) It increases their risk for pulmonary embolism D) It leads to diminished liver and kidney function

A) Impaired blood clotting can exacerbate blood loss

Patients with dementia often have reduced levels of melatonin which can lead to: A) Impairment of the sleep-wake cycle B) Decrease in calcium in the bones C) An increase in the inflammatory process D) Decreased absorption of medications

A) Impairment of the sleep-wake cycle

All of the following lead to an over accumulation of CSF in the brain EXCEPT: A) Increased acidity of CSF B) Overproduction of CSF C) Poor reabsorption of CSF D) Inadequate flow of CSF

A) Increased acidity of CSF

What is the MOST common route of entry for toxic substances? A) Ingestion B) Inhalation C) Injection D) Absorption

A) Ingestion

What method of entry of a toxin results in rapid absorption of the agent through the alveolar-capillary membrane? A) Inhalation B) Injection C) Absorption D) Ingestion

A) Inhalation

Your service has acquired the antidote for a nerve agent attack. As such, you realize that it will be given by what route? A) Injection B) Inhalation C) Topical D) Oral

A) Injection

A patient with kyphosis has fallen and is complaining of numbness in both arms. Regarding immobilization, you should: A) Place pillows in voids between the patient and the long board B) Disregard immobilization and transport using lights and sirens C) Apply a cervical collar and transport in a semi-Fowler's position D) Transport fully immobilized and in a prone position

A) Place pillows in voids between the patient and the long board

Your 23-year-old patient is seven months pregnant and has called 911 for vaginal bleeding. She reports that had bent down to pick up some toys and when she began bleeding. She says it is not painful but she is frightened by the gush of bright red blood. You should suspect: A) Placenta previa B) A ruptured uterus C) Prolapsed cord D) Abruptio placenta

A) Placenta previa

A term referring to the period prior to birth of a child is: A) Prenatal B) Perinatal C) Preterm D) Parturition

A) Prenatal

You respond for an elderly female who has apparently taken the wrong medication. She tells you that she thought she took and aspirin for her headache, but instead she is feeling real tired. You check her medication and see that the aspirin looks like her sleeping pills. Which of the following likely lead to the problem? A) Presbyopia B) Presbycusis C) Menopause D) Proprioception

A) Presbyopia

Approximately 15 trucks and cars have been involved in a "chain-reaction" collision. You are the first ambulance on the scene and begin going from vehicle to vehicle to determine the type and severity of each injury. This would be an example of what kind of triage? A) Primary B) Secondary C) Emergency D) Tertiary

A) Primary

When working in the "warm" zone, the AEMT MUST remember that: A) Protective gear must be worn B) Patient care is not performed C) Contamination is no longer a threat D) Water used does not have to be contained

A) Protective gear must be worn

Prior to restraining a patient, all of the following apply EXCEPT: A) Provide chemical restraint early B) Ensure the safety of your crew C) Use the least amount of restraint necessary D) At least five people should be present

A) Provide chemical restraint early

You have just delivered a healthy infant who is breathing well and has an APGAR of 9 at five minutes. As you are awaiting delivery of the placenta, the mother suddenly experiences severe shortness of breath, JVD, and hypotension. You should suspect: A) Pulmonary embolism B) Autotransfusion C) Hypovolemic shock D) Sepsis

A) Pulmonary embolism

You have been assigned to the triage unit at a multiple-casualty incident. A patient with a yellow triage tag arrives. Your immediate action should be to: A) Re-triage the patient B) Arrange for immediate transport C) Obtain vital signs D) Change the tag to green

A) Re-triage the patient

You have been given an order to cut through the "C" posts of a car with a trapped passenger. As an AEMT with knowledge of vehicle extrication, you cut the posts located at which areas of the roof? A) Rear B) Middle C) Front D) Top

A) Rear

As you approach the scene of a mass casualty situation in a downtown office building, you note the slight odor of almonds as you see many victims in the lobby lying unconscious with no apparent injury. Your BEST actions should be: A) Remain upwind, away from the site, as this is possibly an asphyxiant B) Immediately extricate the nearest victim to an area of safety C) Administer a Mark I kit to all victims within the first five minutes D) Note the odor and continue with triage of all victims

A) Remain upwind, away from the site, as this is possibly an asphyxiant

When you are assessing a three-year-old who is in respiratory distress, which of the following assessment findings should be MOST concerning to you? A) Retractions observed above the clavicles B) Abdominal movement during breathing C) Retraction of the muscles between the ribs D) Respiratory rate of 28 breaths per minute

A) Retractions observed above the clavicles

You are attending a pregnant patient who believes she is in labor. Suddenly, she reports a very large "gush of water" from her vagina. You suspect: A) Ruptured amniotic membrane B) Spontaneous abortion C) Placenta previa D) Pre-term delivery

A) Ruptured amniotic membrane

You are delivering a baby and the head delivers normally, but then retracts back into the birth canal. You should suspect: A) Shoulder dystocia B) Breech delivery C) Precipitous delivery D) Uterine inversion

A) Shoulder dystocia

Which of the following techniques will improve communication with a deaf patient? A) Speak normally while looking directly at the patient B) Exaggerate the pronunciation of your words C) Speak slowly and move your lips carefully D) Raise you voice somewhat and speak more slowly

A) Speak normally while looking directly at the patient

Which of the following would BEST prepare you to care for a special needs patient? A) Spend time working at a camp for special needs adults and children B) Working in the emergency department at the community hospital C) Complete some extra shifts on the ambulance working with a paramedic D) Interview a patient with hearing and speech disabilities

A) Spend time working at a camp for special needs adults and children

Which of the following techniques is preferred when restraining the combative patient who is a threat to himselfand the emergency responders? A) Supine with fourpoint restraint B) Prone with four-point restraint C) Supine, with two-point restraint D) Prone, with two-point restraint

A) Supine with fourpoint restraint

You are mentoring a new EMT. Which of the following could help your protégée understand the importance of treating a patient empathetically? A) Take him to an Assisted Living facility and introduce him to some of your regular patients B) Explain to him the various ways you can get a patient to give you a refusal C) Show him different ways that he can get another unit to take the calls on skid row D) Spend time showing him how to operate the airway equipment on the ambulance

A) Take him to an Assisted Living facility and introduce him to some of your regular patients

When teaching EMRs about airbags, which of the following points would you emphasize? A) The benefit of airbags is decreased in multiple collisions B) Airbags prevent broken glass from striking the occupant C) Properly placed airbags eliminate the need for seatbelts D) Airbags are effective in decreasing injury in a rollover-type collision

A) The benefit of airbags is decreased in multiple collisions

Assessment of a patient involved in a motor vehicle accident reveals him to have crepitus to the left humerus, instability to the left lateral chest wall and flank, and pain on palpation to left hip region. He also complains of pain to the right side of the neck. Based on this injury pattern, the AEMT would recognize the patient was: A) The driver of a car hit on the driver's side B) An unrestrained occupant in a car struck from the side C) An unrestrained driver involved in a head-on collision D) The passenger in a car hit from behind

A) The driver of a car hit on the driver's side

All of the following can lead to urinary retention EXCEPT: A) Urinary incontinence B) Medications C) Neurological problems D) Enlarged prostate

A) Urinary incontinence

During immersion in cold water, heat loss will MOST likely occur as a result of: A) conduction. B) convection. C) radiation. D) evaporation.

A) conduction.

Several weeks after being involved with rescue operations of a tornado that destroyed a small community your partner tells you that he is having relationship problems at​ home, is unable to​ sleep, and is anxious all the time. What do you recommend he should​ do? A. see a mental health professional B. attend a critical incident stress debriefing C. contact his family physician D. take some time off and try to work out his issues

A.

Terrorism that is committed by citizen of a foreign country from the target of the attacks is known​ as: A. international terrorism B. ​lone-wolf terrorism C. patriot terrorism D. domestic terrorism

A.

A medical direction physician gives you a radio order to administer aspirin to a patient with chest pain. How should you respond to the order? A) "Received. I will administer 81 mg aspirin orally." B) "Copy. I will administer the medication." C) "I will re-contact you with any changes in the patient's condition." D) "Affirmative. The patient has no allergies and I will administer the aspirin orally."

Answer: A - "Received. I will administer 81 mg aspirin orally."

Which of the following patients would the AEMT recognize as a drug abuser? A) A 56-year-old man who accidentally took an additional dose of blood pressure medication. B) A 41-year-old man who tried marijuana for the first time. C) A 46-year-old woman who used cocaine for 10 years but has not used it in five years. D) A 39-year-old woman who takes pain medications even though she is not in pain.

D) A 39-year-old woman who takes pain medications even though she is not in pain.

Pulmonary embolism is more common in patients who have had: A) Pelvic inflammatory disease (PID) B) Placenta previa C) Trauma during pregnancy D) A cesarean section

D) A cesarean section

Which one of the following statements about the heart rhythm of ventricular fibrillation is TRUE? A) "When a patient's heart is in ventricular fibrillation, it is unable to pump blood throughout the body." B) "The AED is designed to identify ventricular fibrillation and give a 'No shock advised' message when it is present." C) "The most effective treatment for converting ventricular fibrillation to a healthy heart rhythm is CPR." D) "Ventricular fibrillation occurs when the heart rate is so slow and weak that a pulse cannot be felt."

Answer: A - "When a patient's heart is in ventricular fibrillation, it is unable to pump blood throughout the body."

You are treating a pediatric patients suffering from severe acute anaphylaxis using a solution of epinephrine. The CORRECT dosage and concentration for this patient would be: A) 0.01 mg/kg of a 1:1,000 concentration B) 0.3 mg/kg of a 1:10,000 concentration C) 0.01 mg/kg of a 1:10,000 concentration D) 0.3 mg/kg of a 1:1,000 concentration

Answer: A - 0.01 mg/kg of a 1:1,000 concentration

You are given the order to administer 0.01 mg/kg of epinephrine, 1:1000, IM to a 35-pound pediatric patient. Epinephrine 1:1000 is supplied in a concentration of 1 mg/mL. The correct dosage for this patient would be: A) 0.16 mL B) 16 mL C) 0.35 mL D) 35 mL

Answer: A - 0.16 mL

The CORRECT dosage for activated charcoal for adult and pediatric patients would be: A) 1 gram/kg per os (PO) B) 0.1 gram/kg per rectum (PR) C) 1 gram/kg per rectum (PR) D) 0.1 gram/kg per os (PO)

Answer: A - 1 gram/kg per os (PO)

The normal amount of blood loss during menstruation is around: A) 30 mL. B) 50 mL. C) 100 mL. D) 150 mL.

Answer: A - 30 mL.

Which one of the following patients should the AEMT recognize as having a normal skin temperature? A) 36-year-old man complaining of nausea with warm skin B) 25-year-old woman who is dizzy with cool skin C) 88-year-old man who is weak with skin that is cool and dry D) 47-year-old woman with chest pain and warm-to-hot skin

Answer: A - 36-year-old man complaining of nausea with warm skin

Which of the following would be considered a significant exposure? A) A needle stick while starting an IV on a patient with hepatitis B. B) Wearing gloves while caring for a patient with a skin rash. C) Standing in the same room with a patient with tuberculosis. D) Touching a cloth with blood from an elderly cardiac patient.

Answer: A - A needle stick while starting an IV on a patient with hepatitis B.

Drugs delivered by inhalation typically require the use: A) A small volume nebulizer B) 100-percent oxygen C) A saline lock D) Endotracheal intubation

Answer: A - A small volume nebulizer

Which one of the following statements about different musculoskeletal injuries is TRUE? A) A sprain is an injury to a joint with possible damage to or tearing of ligaments B) A dislocation occurs when the bone within a joint is broken C) A fracture occurs when a tendon connecting muscle to a bone is overstretched and injured D) Strains are injuries that occur to ligaments and the joints to which they are attached

Answer: A - A sprain is an injury to a joint with possible damage to or tearing of ligaments

What sign or symptom of a stroke is defined as the inability of a patient to speak? A) Aphasia B) Dysarthria C) Hemiparesis D) Ataxia

Answer: A - Aphasia

Which of the following conditions will put the elderly patient at risk for drug toxicity? A) Loss of total fluid volume B) Decreased fat stores C) Polypharmacy D) All of the above

D) All of the above

Which of the following is TRUE regarding severe burns in very young pediatric patients? A) They have less subcutaneous fat than adults, which results in increased heat loss when the skin is damaged B) They have thin skin that can be burned at a much lower temperature and shorter duration of exposure to heat than in an adult C) Their high body-surface-area-to-volume ratio causes them to lose fluids more quickly when the skin is damaged D) All of the above are true

D) All of the above

You are in the home of a 17-year-old teenager complaining of abdominal pain. Her mother and father are hovering near her and answering most of the questions you ask her. You sense that the patient is uncomfortable answering questions in front of her parents. Which of the following is the BEST way to handle this situation? A) Defer the history to the triage nurse at the emergency department so as not to cause undue discomfort to the patient B) Tell the parents that their daughter is a big girl and probably doesn't want to talk about her problems in front of them C) Tell the patient you know she might be embarrassed but that it is imperative that she give you the whole story so you know what to do for her D) Ask the parents to go to the kitchen with your partner so he can get some information for your paperwork

D) Ask the parents to go to the kitchen with your partner so he can get some information for your paperwork

The branch of medicine that deals with the management of obese patients is called: A) Adipositics B) Lipidenosis C) Steatpheresis D) Bariatrics

D) Bariatrics

All of the following are symptoms of a panic attack EXCEPT: A) Dizziness B) Paresthesia C) Nausea D) Bradycardia

D) Bradycardia

Which of the following is another term for an articulation disorder? A) Dysrhythmias B) Dysphasia C) Dysphoria D) Dysarthria

D) Dysarthria

A type of biological agent is: A) Vesicants B) Nerve agents C) Blister agents D) Encephalitis-like agents

D) Encephalitis-like agents

You are treating a patient who has been stabbed in the abdomen and now has abdominal contents protruding from the site of the wound. The BEST way to treat this injury is to: A) Cover contents with moist gauze and use an occlusive dressing such as plastic from a trauma dressing package and tape in place B) Elevate patient's shoulders and cover abdominal contents with bulky, wet dressing C) Carefully replace abdominal contents after rinsing with saline and cover with bulky, dry dressing D) Dressing and bandage wound as any other, careful transport

Answer: A - Cover contents with moist gauze and use an occlusive dressing such as plastic from a trauma dressing package and tape in place

You are examining a patient who complains of difficulty breathing. Upon auscultation of the lung fields, you notice a light, popping, nonmusical sound. This is known as: A) Crackles B) Rhonchi C) Wheezing D) Stridor

Answer: A - Crackles

A common complaint from patients who are immobile is/are: A) Decubitus ulcers. B) Cellulitis. C) Gangrene. D) Clostridia.

Answer: A - Decubitus ulcers.

You respond for a 52-year-old woman complaining of pain in her right calf. She has been relatively immobile since fracturing her ankle two weeks earlier. You should first suspect: A) Deep vein thrombosis. B) Pathological fracture. C) Multiple myeloma. D) Sickle cell crisis.

Answer: A - Deep vein thrombosis.

The effects of head injury that manifest themselves after the initial injury are referred to as: A) Direct brain injuries. B) Secondary brain injuries. C) Primary insults. D) Indirect brain injuries.

Answer: A - Direct brain injuries

What is the FIRST method that you should use to control external hemorrhage? A) Direct pressure B) Ice C) Elevation D) Tourniquet

Answer: A - Direct pressure

The AEMT understands that when paramedics and AEMTs administer IV fluids as treatment for hemorrhagic shock, the fluids will: A) Do nothing to get more oxygen to the cells B) Enable the lungs to better oxygenate the blood C) Stabilize the shock by restoring needed electrolytes D) Reverse the shock by increasing the blood pressure

Answer: A - Do nothing to get more oxygen to the cells

The potential effects of crush injury syndrome when the object is removed from the patient include all of the following EXCEPT: A) Hypotension B) Metabolic acidosis C) Renal failure D) Cardiac toxicity

Answer: A - Hypotension

Which IV fluids cause a fluid shift of fluid from the intravascular space to the intracellular space? A) Hypotonic crystalloids B) Isotonic crystalloids C) Hypertonic crystalloids D) Isotonic colloids

Answer: A - Hypotonic crystalloids

A 45-year-old woman was at a family reunion and fell off a trampoline. She impacted a picnic table with the left side of her lower rib cage and is very anxious and confused. Her airway is open, and her breathing is adequate. Her radial pulse is rapid and weak and her skin is cool and diaphoretic. Vital signs are: pulse 132, respirations 20, blood pressure 102/78 mmHg, and SpO2 at 97 percent. She has ecchymosis to the left lower rib area as well as tenderness and instability. She also has a superficial abrasion to her forehead. Given these findings, you should suspect and treat her for what condition? A) Hypovolemic shock B) Hypoxia C) Closed head injury D) Spinal shock

Answer: A - Hypovolemic shock

A 48-year-old man is short of breath and confused. His airway is open and clear with a respiratory rate of 28 breaths per minute. His skin is cool and dry. His pulse is 124 beats per minute with a blood pressure of 152/88 mmHg. His Sp02 is 89percent on room air and he has slight wheezing on inspiration. Based on his signs and symptoms is the greatest threat to this patient? A) Hypoxia B) Tachycardia C) Infection D) Elevated blood pressure

Answer: A - Hypoxia

You have an 18-year-old woman who was hit in the arm by a softball. Upon inspection, you observe swelling and tenderness, but no deformity or crepitus. The patient has no external bleeding, but there is tissue discoloration. You suspect internal bleeding, and begin treatment which includes: A) Immobilization B) Pain control C) Controlling hemorrhage D) Preventing introduction of contamination into the wound

Answer: A - Immobilization

The AEMT should recognize the cause of heart failure is: A) Inadequate ejection of blood. B) Poor gas exchange in the lungs. C) Fluid accumulation in the lung tissue. D) Backup of fluid in the lower extremities.

Answer: A - Inadequate ejection of blood.

You are giving report to the nurse at the hospital regarding your patient who is nine months pregnant. You tell her that the patient reports that her doctor told her that she had recently had protein in her urine. You know that this is indicative of: A) Eclampsia B) Gestational diabetes C) Imminent labor D) Preeclampsia

D) Preeclampsia

An elderly man mistakenly presses the gas pedal instead of the brake causing him to crash through convenience store. This scenario is commonly related to: A) Dementia B) Confabulation C) Alzheimer's disease D) Proprioception

D) Proprioception

You find your 18-year-old patient thrown from his vehicle. He is breathing six times per minute. You should: A) Provide supplemental oxygen at 4-6 lpm via nonrebreather mask B) Provide supplemental oxygen at 15 lpm via nonrebreather mask C) Provide supplemental oxygen at 4-6 lpm and consider overdrive ventilation D) Provide supplemental oxygen at 15 lpm and consider overdrive ventilation

D) Provide supplemental oxygen at 15 LPM and consider overdrive ventilation

By which mechanism of action does naloxone work as a narcotic antagonist to reverse respiratory depression associated with narcotic overdose? A) Naloxone has a greater affinity for narcotic receptor sites than opiates, therefore displacing from the receptor B) Naloxone blocks part of the chemical reaction responsible for activating narcotics C) Naloxone increases glucose concentration in the blood for the reversal of acute overdose D) Naloxone is administered in overdose to cause vasoconstriction and relax bronchiolar smooth muscle

Answer: A - Naloxone has a greater affinity for narcotic receptor sites than opiates, therefore displacing from the receptor

The MOST common medication administered sublingually by AEMTs is: A) Nitroglycerin B) Aspirin C) Abuterol D) Nitrous oxide

Answer: A - Nitroglycerin

A three-year-old-has stuck a crayon in his nose. Assessment reveals the crayon to be deeply embedded in the right nostril with some irritation and swelling noted. His vital signs are: pulse 124, respiration 20, and SpO2 100 percent. Which of the following would be MOST appropriate when caring for this child? A) Non-emergent transport to the hospital B) Place a warm pack to the bridge of the nose C) Attempt removal of the crayon with forceps D) Oxygen via a nonrebreather at 5 liters/minute

Answer: A - Non-emergent transport to the hospital

The problem with a Le Fort fracture is that it may: A) Present an airway compromise. B) Frequently be the cause of hypovolemia. C) Have internal bleeding associated with it. D) All of the above

Answer: A - Present an airway compromise.

Which of the following burn classifications describes a first-degree burn? A) Superficial burn B) Full-thickness burn C) Partial-thickness burn D) Second-degree burn

Answer: A - Superficial burn

For which of the following patients is microdrip tubing MOST appropriate? A) The 68-year-old suffering difficulty breathing from CHF B) The 40-year-old man who passed out from hypotension C) The 20-year-old victim of an ATV rollover D) The 34-year-old runner suffering from dehydration

Answer: A - The 68-year-old suffering difficulty breathing from CHF

When determining the mechanism of injury, you will identify: A) The forces involved in the accident B) The number of patients C) The need for additional resources D) The person who caused the accident

Answer: A - The forces involved in the accident

You are assessing the pupils of a patient who hit his head after falling from the top of a tractor trailer. Which one of the following findings would suggest a closed head injury? A) The left pupil constricts to light but the right pupil does not. B) The left pupil dilates in reaction to light while the right does not. C) Both the left and right pupils are constricted and do not react to light. D) The right pupil constricts when light is shined into the left pupil.

Answer: A - The left pupil constricts to light but the right pupil does not

Ringing in the ears is also known as ________. A) Tinnitus B) Vertigo C) Eczema D) Purulence

Answer: A - Tinnitus

You are called for a patient who passed out with no apparent cause. Bystanders report that she was only out for a few seconds. The patient has a cardiac history and diabetes. The patient reports she was reading at the library and simply fell forward. Which of the following would be the MOST likely cause of her syncope? A) Transient cardiac dysrhythmia B) Fright from something she read C) A focal motor seizure of short duration D) An acute neurological disorder

Answer: A - Transient cardiac dysrhythmia

Pain that originates in an organ, such as the intestines, is called ________ pain. A) Visceral B) Referred C) Acute D) Parietal

Answer: A - Visceral

You are called for a 'near' drowning. When you arrive, you find a nine-year-old boy who was pulled from the deep end of the pool just before you arrived. He is conscious and alert, sitting by his mother. He has a shallow cough and his pulse oximetry reading is 98 percent. How should you manage this patient? A) Advise his mother to watch him closely for the next 24 hours B) Recommend that his mother schedule an appointment with his physician C) See that he is dried and warmed before sending him home with his mother D) Recommend that he be transported to the hospital for evaluation

D) Recommend that he be transported to the hospital for evaluation

Among the following, who is the MOST likely to be the abuser of a child? A) The child's kindergarten teacher B) A babysitter who sits regularly C) A brother or sister of the child D) Someone he knows and trusts

D) Someone he knows and trusts

Which of the following is a concern when caring for bariatric patients? A) They often require medical equipment for day-to-day living B) They usually live in nursing homes or other medical facilities C) They require higher concentrations of oxygen D) Supine positioning can lead to respiratory difficulty

D) Supine positioning can lead to respiratory difficulty

Which one of the following questions is BEST asked when attempting to determine the degree of sensation in the hand of a patient who has suffered an elbow injury? A) "Can you tell me if you have parethesia in your hand?" B) "Can you tell me what finger I am touching?" C) "Do you have feeling in your hand?" D) "Does your hand feel as though it is asleep?"

Answer: B - "Can you tell me what finger I am touching?"

A 66-year-old man is complaining of substernal chest pain. The AEMT is considering assisting with the patient's prescribed nitroglycerin. Which one of the following patient statements is MOST relevant to the AEMT at this time? A) "I accidentally took five baby aspirins when the chest pain first started." B) "My urologist just started treating me for problems I am having with sex." C) "I took a nitroglycerin yesterday for the pain and it did not do anything." D) "I forgot to take my high blood pressure medication this morning."

Answer: B - "My urologist just started treating me for problems I am having with sex."

You are treating a neonate for hypoglycemia using a dextrose solution. The CORRECT dosage and concentration for this patient would be: A) 0.1 mg/kg of a 50 percent dextrose solution B) 0.5 mg/kg of a 10 percent dextrose solution C) 0.5 mg/kg of a 25 percent dextrose solution D) 0.5 mg/kg of a 50 percent dextrose solution

Answer: B - 0.5 mg/kg of a 10 percent dextrose solution

Which of the following size of needle gauge is used to draw up medications but NOT to administer them subcutaneously or intravenously? A) 25 gauge B) 18 gauge C) 21 gauge D) 23 gauge

Answer: B - 18 guage

Which one of the following patients has an isolated head injury that should be considered the MOST serious? A) 31-year-old man who is nonverbal and extends his arms and legs when you pinch the muscles on his shoulder B) 21-year-old man wearing a deformed motorcycle helmet who does not respond to verbal or painful stimuli C) 62-year-old woman who fell and hit her head and is becoming very confused and combative D) 53-year-old woman who fell and has a large laceration that is bleeding heavily to the back of her head

Answer: B - 21-year-old man wearing a deformed motorcycle helmet who does not respond to verbal or painful stimuli

Stroke patients who may be candidates for fibrinolytic therapy MUST receive definitive treatment within __________ hours of onset of symptoms. A) 1-2 B) 3-4 ½ C) 5-6 ½ D) 6-8

Answer: B - 3-4 ½

You are dispatched to a structural fire in which a 32-year-old man has been burned in a steam blast. Upon assessment, you note blisters to the anterior chest and circumferential burns to both lower extremities. Using the rules of nines, what percentage of burn does this patient have? A) 54 percent B) 45 percent C) 36 percent D) 27 percent

Answer: B - 45 percent

Macrodrip tubing is indicated for all of the following patients EXCEPT a: A) 68-year-old in anaphylactic shock B) 77-year-old in CHF C) 24-year-old in heat exhaustion D) 17-year-old gunshot victim

Answer: B - 77-year-old in CHF

Severe back pain radiating into the legs is associated with which of the following conditions? A) Acute cholecystitis B) Abdominal aortic aneurysm C) Gastrointestinal blockage D) Appendicitis

Answer: B - Abdominal aortic aneurysm

A nine-year-old boy has fallen from a swing. Assessment findings reveal no threats to the airway, breathing, or circulation, but do indicate deformity, pain, and swelling to his right wrist. The right radial pulse is strong and is accompanied by skin that is pink and warm to the touch. Which one of the following actions indicates that the AEMT is properly caring for this patient? A) Maintaining the wrist below the level of the heart to decrease swelling B) Applying cold packs to the wrist to reduce swelling C) Straightening the wrist to promote blood flow to the hand D) Massaging the wrist gently to decrease the pain

Answer: B - Applying cold packs to the wrist to reduce swelling

A patient goes into cardiac arrest at 11:40 A.M. Which one of the following treatment descriptions gives the patient the BEST chance for recovery? A) Defibrillation at 11:46 A.M. followed by ACLS at 11:51 A.M. B) CPR at 11:41 A.M. and defibrillation at 11:43 A.M. C) CPR at 11:42 A.M. and advanced cardiac drugs at 11:48 A.M. D) Defibrillation at 11:44 A.M. followed by CPR at 11:49 A.M.

Answer: B - CPR at 11:41 A.M. and defibrillation at 11:43 A.M.

A mnemonic to help the AEMT evaluate aspects of the secondary assessment is: A) SAMPLE B) DCAP-BTLS C) OPQRST D) AVPU

Answer: B - DCAP-BTLS

Of the following, which would be the MOST likely dysfunction experienced by a patient with multiple myeloma? A) Decreased T cell production B) Decreased blood cell production C) Increased T cell production D) Increased blood cell production

Answer: B - Decreased blood cell production

The method of medication administration that involves placing a hollow needle into the medullary cavity, where medication is absorbed through the medullary vasculature would be: A) Intravenous (IV) route B) Intraosseous (IO) route C) Intramuscular (IM) route D) Subcutaneous (SC) route

Answer: B - Intraosseous (IO) route

The ________ of the eye works similarly to the shutter of a camera. A) lens B) Iris C) Eyelid D) Cornea

Answer: B - Iris

Lactated Ringer's is considered an: A) Hypertonic colloid B) Isotonic crystalloid C) Isotonic colloid D) Hypotonic crystalloid

Answer: B - Isotonic crystalloid

Your patient is a 40-year-old woman who has been experiencing abdominal pain and vomiting for two days. She is now responsive to verbal stimulus; has cool, dry skin; a heart rate of 116; respirations of 24; and a blood pressure of 100/70. Which of the following is the BEST position for transporting this patient? A) Supine with the knees bent B) Left lateral recumbent with the legs bent C) Sitting up at a 90-degree angle D) Sitting up at a 45-degree angle

Answer: B - Left lateral recumbent with the legs bent

An EMR reports that a patient has bruising to the lumbar area of the back. Based on this statement, the AEMT should expect to find bruising in which area? A) Neck B) Lower back C) Upper back D) Buttocks

Answer: B - Lower back

A method used to calculate the amount of force involved in a collision would be: A) Acceleration × deceleration B) Mass × acceleration (or deceleration) C) Speed × velocity D) None of the above

Answer: B - Mass × acceleration (or deceleration)

The proper site for placement of an intraosseous needle for the pediatric patient is the: A) Lateral aspect of the proximal tibia B) Medial aspect of the proximal tibia C) Medial malleolus D) Lateral malleolus

Answer: B - Medial aspect of the proximal tibia

Handguns as well as some rifles and shotguns are considered: A) Less-than-lethal weapons B) Medium-energy weapons C) Low-energy weapons D) Highly-lethal forces

Answer: B - Medium-energy weapons

A chronic disease of the inner ear marked by a recurring syndrome of vertigo, tinnitus, and progressive hearing loss is called ________. A) Cholesteatoma B) Meniere disease C) Otosclerosis D) Mastoiditis

Answer: B - Meniere Disease

Which of the following is TRUE regarding myocardial infarction in the elderly? A) They are more likely to present with classic symptoms B) They usually complain of more severe symptoms C) They are less likely to have cardiac enzyme elevation D) They are less likely to present with classic symptoms

D) They are less likely to present with classic symptoms

After administering oral glucose to a patient with altered mental status, which one of the following BEST indicates a positive therapeutic effect or response to the medication? A) Drop in BP from 156/90 to 120/80 mmHg B) Mental status that is alert and oriented C) Absence of an allergic reaction D) Pulse oximeter reading of 99 percent on room air

Answer: B - Mental status that is alert and oriented

Your patient is complaining about severe pain from a minor scratch on his arm. His medical history includes type 2 diabetes and hypertension. He is agonizing over the pain in his arm. You should suspect: A) Dry gangrene. B) Necrotizing fasciitis. C) Infectious cellulitis. D) Ankylosing spondylosis.

Answer: B - Necrotizing fasciitis.

Which of the following observations indicate that your patient may have overused his prescription inhaler? A) Decreased level of consciousness B) Nervousness C) Increased secretions from the airway D) Decreased heart rate

Answer: B - Nervousness

The term for an inflammation of the female reproductive organs and genitalia usually caused by STD is: A) Pyelonephritis. B) Pelvic inflammatory disease. C) Chlamydia. D) Gonorrhea.

Answer: B - Pelvic inflammatory disease.

While cleaning a gun, a 44-year-old man accidentally shot himself in the abdomen. On arrival the patient is responsive to painful stimuli and lying on his side with his legs drawn to his chest. Blood is evident on his shirt and pants. Which one of the following shows the CORRECT sequence of events for caring for this patient? A) Perform a primary and rapid secondary assessment, move to the stretcher for immediate transport, and start positive pressure ventilation en route to the hospital B) Perform the primary assessment, administer high-flow oxygen, perform the secondary assessment, transfer to the stretcher, and provide rapid transport C) Transfer to the ambulance, perform the primary assessment, provide oxygen therapy, and provide rapid transport D) Transfer to the ambulance, provide rapid transport, perform primary and secondary assessments, and administer oxygen therapy en route to the hospital

Answer: B - Perform the primary assessment, administer high-flow oxygen, perform the secondary assessment, transfer to the stretcher, and provide rapid transport

When the gallbladder is diseased, the pain is NOT only felt in the RUQ but also in the: A) Epigastric area. B) Right shoulder. C) Left shoulder. D) Right scapula area.

Answer: B - Right shoulder.

When should the AEMT use the pulse oximeter? A) After contacting medical direction for authorization B) Routinely on all patients with a medical or trauma complaint C) On any and all patients one year of age and older D) Only if the AEMT is having difficulty getting a blood pressure

Answer: B - Routinely on all patients with a medical or trauma complaint

Of the following situations, which would LEAST likely to result in infection? A) A highly virulent form of anthrax is inhaled during decontamination. B) Scrubbing your hands after transferring a patient with a rash. C) Blood splatter into the eyes from a patient with hepatitis C. D) A needle stick from a needle containing contaminated blood.

Answer: B - Scrubbing your hands after transferring a patient with a rash.

A 69-year-old man is complaining of weakness to the left arm and leg. He also states that he is nauseated and has a headache. The patient's past medical history includes stroke and diabetes. In relation to this information, which one of the following is TRUE? A) The dizziness is a sign indicating the severity of the chief complaint B) The headache is a symptom related to the chief complaint C) The history of diabetes is a contributing symptom of the chief complaint D) The history of diabetes is a sign related to the chief complaint

Answer: B - The headache is a symptom related to the chief complaint

A patient has been struck in the back by a heavy piece of wood that was being bent to make a form for a concrete arch. He is responsive, but cannot feel or move his legs. He has bruising to his back and is incontinent of urine. The skin below the injury site is red and warm. As a knowledgeable AEMT, you should realize that: A) A cervical collar is unnecessary B) The paralysis may resolve C) The ability to feel will return D) The paralysis is permanent

Answer: B - The paralysis may resolve

A feeling of dizziness and or whirling in space is called ________. A) Purulence B) Vertigo C) Eczema D) Tinnitus -

Answer: B - Vertigo

Which of the following sounds may be heard in lower respiratory obstruction? A) Crowing B) Wheezing C) Stridor D) Snoring

Answer: B - Wheezing

After completion of the initial assessment, you should consider top priority transport for the following patients EXCEPT an adult: A) With a respiratory rate greater than 10 breaths per minute B) With multiple lacerations and controlled bleeding C) With a pulse rate greater than 100 beats per minute D) That is alert but not oriented

Answer: B - With multiple lacerations and controlled bleeding

What is the recommendation of the American Diabetes Association for diabetics' HbA1c levels? A) ≥ 8.0 percent B) ≤ 6.5 percent C) 1.5 percent D) 9 percent

Answer: B - ≤ 6.5 percent

The AEMT understands the primary reason to obtain a medical history using the SAMPLE mnemonic when he states: A) "The SAMPLE mnemonic helps AEMTs to diagnose a patient's medical problem." B) "The SAMPLE mnemonic is important because it will give the AEMT the patient's exact chief complaint." C) "After performing the primary assessment, the SAMPLE mnemonic helps guide the AEMT in further assessment and care." D) "It is important to use the SAMPLE mnemonic because the emergency department will need the information."

Answer: C - "After performing the primary assessment, the SAMPLE mnemonic helps guide the AEMT in further assessment and care."

You have arrived on the scene of an industrial plant where a female worker had a chemical splashed into her eye. She is complaining of severe pain in the eye and is flushing it with tap water over a sink. Which of the following questions is it MOST important that the AEMT ask FIRST? A) "Do you take any medications?" B) "Have you ever had an eye injury before?" C) "Are you wearing contact lenses?" D) "What is your past medical history?"

Answer: C - "Are you wearing contact lenses?"

Which of the following is TRUE regarding febrile seizures? A) The higher the fever, the more likely to have a seizure B) They cause a long postictal state after the seizure C) Cool the patient to the shivering state to decrease temperature D) They are related to the rate of increase in temperature

D) They are related to the rate of increase in temperature

Which one of the following statements made by a patient's family member would cause the AEMT to suspect that a patient is suffering from hypovolemic shock? A) "He has been taking an antibiotic for a chest cold." B) "He got up this morning and was having a hard time breathing." C) "He cannot stop throwing up." D) "He has had a rash for the past three days."

Answer: C - "He cannot stop throwing up."

A vacuum splint has just been applied to the arm of a patient who fell backward from a chair while hanging drapes. Which one of the following statements or questions should the AEMT make or ask next? A) "Can you rate your pain for me?" B) "I need to feel your arm for tenderness." C) "I am going to feel your wrist for a pulse now." D) "I am wrapping the splint with roller gauze to secure it."

Answer: C - "I am going to feel your wrist for a pulse now."

You are having a difficult time controlling bleeding from a small skin avulsion to a patient's ankle. Which one of the following statements made by the patient BEST explains why control has been difficult to achieve? A) "I take steroids for my lung disease." B) "My blood pressure sometimes runs a little high." C) "I take Coumadin for my irregular heartbeat." D) "I drank a lot of alcohol last night."

Answer: C - "I take Coumadin for my irregular heartbeat."

A roofer on top of a two-story house called 911 after experiencing chest pain and dizziness. You are providing care to the patient on top of the roof and the fire department has arrived to help remove the patient from his present location. As the patient is brought down the ladder, your highest priority as an AEMT is: A) Safety of the patient B) Providing patient care C) Safety of the other rescuers D) Your personal safety

D) Your personal safety

Problems related to a patient experiencing hallucinations, phobias, or delusions are related to the patient's: A) sensorium. B) bipolar disorder. C) cognitive processes. D) perceptual processes.

D) perceptual processes.

A patient working at an ice plant was trapped in a room filled with sulfur dioxide. On arrival, you donned a self-contained breathing apparatus and entered the room, finding the patient unresponsive on the floor. The room remains filled with the gas. Your immediate priority in caring for this patient is to: A) assess the airway, breathing, and circulation. B) loosen all tight-fitting clothing. C) start positive pressure ventilation. D) remove him from the immediate environment.

D) remove him from the immediate environment.

An individual who works on his own to plan and carry out an act of terrorism is known as​ a(n): A. Unabomber B. domestic terrorist C. conventional terrorist D. ​lone-wolf terrorist

D.

Before beginning care on the scene of a chemical​ event, what must the AEMT​ assure? A. that the AEMT is properly decontaminated B. that life threats are already managed in the hot zone C. that the patient has been properly triaged D. that the patient is properly decontaminated

D.

Being active in the National Disaster Medical System may require the responder​ to: A. travel overseas and assist with medical management of disaster victims B. assume incident commander duties on all MCIs in his or her jurisdiction C. only respond to local disasters D. become involved in MCIs outside of his or her jurisdiction

D.

Locally based teams that respond jointly where disaster medical assistance is required are usually trained through which​ entity? A. Federal Emergency Management Agency B. National Incident Command System C. National Highway Safety and Traffic Administration D. National Disaster Medical System

D.

Which of the following can best assure smooth scene operations with other jurisdictions during a​ disaster? A. assume that every system operates the same B. insist everyone follow your​ agency's SOPs for disaster management C. allow federal authorities to assume command of the scene D. have disaster management training with other agencies

D.

Which of the following is to be expected when caring for survivors of a​ disaster? A. physical injuries B. emotional injuries C. behavioral issues D. all of the above

D.

You have been called to a public pool for an unresponsive patient. On arrival you find lifeguards performing CPR with a pocket mask and oxygen on a 67-year-old man. They report that the patient was in the water and was seen clutching his chest seconds before going unresponsive. He was immediately pulled from the water and CPR was initiated. They estimate that CPR has been performed for five minutes. Assessment shows the man to be unresponsive, apneic, and pulseless. Which one of the following is your FIRST response? A) "Stop CPR and let us apply the AED." B) "We need to quickly dry him from head to toe before applying the AED." C) "Let us take a towel and dry off his chest." D) "Let us start ventilation with a bag-valve mask and oxygen."

Answer: C - "Let us take a towel and dry off his chest."

Which one of the following statements about traction splints is TRUE? A) "Once the traction splint has been positioned under the leg, and the ankle hitch is applied, manual traction can be released." B) "Once the mechanical traction has been applied, the ischial strap must be released to promote circulation into the affected leg." C) "Mechanical traction should be applied until the mechanical traction is equal to the manual traction and the patient experiences a reduction in pain." D) "Mechanical traction should be applied until the affected leg is approximately one to two inches longer than the unaffected leg."

Answer: C - "Mechanical traction should be applied until the mechanical traction is equal to the manual traction and the patient experiences a reduction in pain."

A 66-year-old man is complaining of substernal chest pain. The AEMT is considering assisting with the patient's prescribed nitroglycerin. Which one of the following patient statements is MOST relevant to the AEMT at this time? A) "I took a nitroglycerin yesterday for the pain and it did not do anything." B) "I forgot to take my high blood pressure medication this morning." C) "My urologist just started treating me for problems I am having with sex." D) "I accidentally took five baby aspirins when the chest pain first started."

Answer: C - "My urologist just started treating me for problems I am having with sex."

Which of the following patients should NOT receive any resuscitation attempts? A) 45-year-old who was pulled from an icy lake after being submerged for 10 minutes B) 5-year-old pulled from swimming pool in summertime C) 26-year-old with a complete midsection dissection from a sheet of metal at a shipyard D) 10-year-old struck by auto at a medium speed collision

Answer: C - 10-year-old struck by auto at a medium speed collision

In the prehospital care report, the AEMT should recognize that a blood pressure has been palpated when she sees: A) 118/78 (palpated) B) Palp: 178/118 mmHg C) 178/P D) P/118

Answer: C - 178/p

Upon arrival on scene, you find a seven-year-old boy struck by a car. He has an altered level of consciousness and is exhibiting signs of shock. How much fluid would you administer initially? A) 10 mL/kg B) 300 mL C) 20 mL/kg D) 15 mL/kg

Answer: C - 20 mL/kg

Which of the following statements about leukemia is TRUE? A) Leukemia is a disease of children and young adults, not older adults. B) Leukemias are cancers of red blood cells. C) A patient with leukemia may bleed excessively. D) Common forms include Hodgkin's lymphoma.

Answer: C - A patient with leukemia may bleed excessively.

Which of the following patients with respiratory distress is the MOST critically ill? A) A patient who speaks in five or six word sentences. B) A patient with wheezing in the lower lobes. C) A patient with no breath sounds on auscultation. D) A patient with a pulse oximetry reading of 93 percent.

Answer: C - A patient with no breath sounds on auscultation.

Your patient just deplaned from a long coast to coast flight and you are called because he has had an acute onset of shortness of breath. You should suspect: A) An acute asthma attack. B) Disseminated intravascular coagulation. C) A pulmonary embolism due to DVT. D) Flash pulmonary edema.

Answer: C - A pulmonary embolism due to DVT.

Which of the following mnemonics is used to help determine the cause of a patient's altered mental status? A) SAMPLE B) AEIOU-TIPS C) AVPU D) OPQRST

Answer: C - AVPU

When assessing a patient who sustained blunt trauma to the chest, which one of the following assessment findings is MOST indicative of an injury to the lung? A) Elevated heart rate and blood pressure B) Respiratory rate of 20 breaths per minute C) Ability to speak a few words and then gasp D) Ecchymotic area to the anterior chest

Answer: C - Ability to speak a few words and then gasp

In the mnemonic AEIOU-TIPS, what does the letter 'A' stand for? A) Arm drift B) Anaphylaxis C) Alcohol D) Abnormal speech

Answer: C - Alcohol

The single-dose breakable glass vessel containing medication is known as a(n): A) Pre-filled syringe B) Vial C) Ampule D) Non-constituted vial

Answer: C - Ampule

Carbon dioxide exchanged for oxygen in the alveolus in the lungs is an example of: A) Internal respiration. B) Anatomic ventilation. C) External respiration. D) Oxyhemoglobin exchange.

Answer: C - External respiration.

The major difference between stable and unstable angina is that unstable angina: A) Indicates the patient's condition is improving. B) Responds more readily to treatment. C) Is generally not relieved by rest. D) Causes cardiac muscle cell death.

Answer: C - Is generally not relieved by rest.

Which fluid solutions given in therapeutic amounts do NOT cause significant fluid or electrolyte shifts in patients with normal fluid status? A) Hypotonic crystalloids B) Hypertonic colloids C) Isotonic crystalloids D) Isotonic colloids

Answer: C - Isotonic crystalloids

Why is an occlusive dressing, taped on three sides, used in a patient with an open pneumothorax? A) It converts an open pneumothorax into a spontaneous pneumothorax, limiting the aspiration of air and enhancing pressure build-up B) It converts an open pneumothorax into a tension pneumothorax, limiting the aspiration of air and relieving pressure build-up C) It converts an open pneumothorax into a simple pneumothorax, reducing the aspiration of air and relieving pressure build-up D) It converts an open pneumothorax into a hemopneumothorax, limiting the aspiration of blood and relieving pressure build-up

Answer: C - It converts an open pneumothorax into a simple pneumothorax, reducing the aspiration of air and relieving pressure build-up

Which of the following is TRUE regarding hemoglobin? A) It contains proteins such as albumin and antibodies. B) It can carry two oxygen molecules at a time. C) It is a molecule that consists of protein and iron. D) It has a greater affinity for oxygen than carbon monoxide.

Answer: C - It is a molecule that consists of protein and iron.

Which of the following is typically seen as a nosocomial infection? A) Impetigo B) Boils C) MRSA D) Ringworm

Answer: C - MRSA

Which of the following shows good EMS provider safety by the AEMT? A) Re-capping of contaminated needles B) Sticking needles into the stretcher mattress C) Making a habit to include disposal of sharps as a step continuous with the procedure D) Lying sharps aside for later disposal

Answer: C - Making a habit to include disposal of sharps as a step continuous with the procedure

In a lateral dislocation of the ankle, the affected limb: A) Presents with a downward pointing foot (plantar flexion) B) Presents with an upward turned foot (dorsiflexion) C) Presents with a foot turned outward from the ankle D) Remains in the normal anatomical position and becomes immediately swollen

Answer: C - Presents with a foot turned outward from the ankle

You are treating a patient with suspected spinal injury following an accident. Your initial actions are: A) Removal from a vehicle to prevent further injury B) Secondary assessment C) Primary assessment while maintaining manual cervical stabilization D) Establishing large-bore IVs and treatment for shock

Answer: C - Primary assessment while maintaining manual cervical stabilization

An AEMT has an accurate understanding of the systolic blood pressure when he tells you that the systolic blood pressure is: A) The pressure in the veins B) Represented by the bottom number C) Produced when the heart contracts D) Caused by constriction of the arteries

Answer: C - Produced when the heart contracts

Which of the following medications is indicated in the treatment of a 52-year-old man with difficulty breathing and a history of emphysema? A) Activated charcoal B) Nitroglycerin C) Proventil D) Oral glucose

Answer: C - Proventil

All of the following are common complications of renal dialysis EXCEPT: A) Decrease in hemoglobin levels B) Bleeding hemodialysis access sites. C) Pulmonary embolism D) Electrolyte abnormalities

Answer: C - Pulmonary embolism

A drop in the systolic pressure of greater than 10 mmHg during inspiration is called: A) Pulsus alternans B) Paradoxical movement C) Pulsus paradoxus D) Mean arterial pressure

Answer: C - Pulsus paradoxus

Light passes to the eye through what opening? A) Iris B) Eyeball C) Pupil D) Lens

Answer: C - Pupil

Friends called 911 for a 37-year-old woman who intentionally overdosed on a narcotic drug. Which one of the following eye findings would reinforce that the patient did indeed take a narcotic? A) Non-reactive pupils that are unequal B) Bloodshot eyes with large pupils C) Pupils that are constricted D) Pupils that constrict to light

Answer: C - Pupils that are constricted

You have been called to assess a conscious and alert five-year-old whose chief complaint is nausea and vomiting for the past two hours. When assessing his pulse, you should FIRST check which pulse? A) Brachial B) Carotid C) Radial D) Pedal

Answer: C - Radial

Of the following, which is the MOST important in caring for a possible stroke patient? A) Establish the time of onset of signs and symptoms. B) Administer oxygen to patients with a SpO2 <94percent. C) Recognize signs and symptoms that may indicate stroke. D) Check the patient's blood glucose level.

Answer: C - Recognize signs and symptoms that may indicate stroke.

Which abdominal quadrant contains the appendix? A) Left lower B) Right upper C) Right lower D) Left upper

Answer: C - Right lower

A female patient struck her face on the steering wheel in a head-on motor vehicle collision and has deformity and crepitus to the bridge of her nose. Further assessment reveals bleeding deep within the nasal passages. The patient has been fully immobilized to a long board. To BEST address the hemorrhage in the nasopharynx, the AEMT would: A) Carefully pack the nasal passages with rolls of sterile gauze. B) Apply ice packs to the bridge of the nose to control hemorrhage. C) Suction blood from the airway as needed. D) Transport fully immobilized with her feet elevated.

Answer: C - Suction blood from the airway as needed

In the mnemonic DCAP-BTLS, "S" stands for: A) Sensation B) Shock C) Swelling D) Symmetry

Answer: C - Swelling

The pulse pressure is the: A) Stroke volume times the heart rate. B) End-systolic volume minus the end-diastolic volume. C) Systolic blood pressure minus the diastolic blood pressure. D) Cardiac output times the systemic vascular resistance.

Answer: C - Systolic blood pressure minus the diastolic blood pressure.

You are treating a patient with a history of asthma. You have administered two albuterol breathing treatments and the patient had used their own inhaler three times before calling you. Which of the following describes an expected side effect of the medication administration? A) Respiratory rate decreasing from 30 to 24 breaths per minute B) Relaxation of bronchial smooth muscle C) Tachycardia D) Bronchospasm

Answer: C - Tachycardia

You have arrived on a scene in which one person has been shot in the head. As you approach the patient, you note that he is sitting up and talking with the police. He is holding a blood-soaked towel over the left temporal area of his head. Which one of the following is your initial action in caring for this patient? A) Determine the need for a nasal or oral airway B) Remove the towel to assess the gunshot wound C) Take manual inline spinal stabilization D) Obtain a pulse rate, respiratory rate, and blood pressure.

Answer: C - Take manual inline spinal stabilization

Correctly assessing orthostatic vital signs involves: A) Determining if the patient's respirations and heart rate are within normal limits B) Determining if the patient's pulse disappears during deep inspiration C) Taking the blood pressure in the supine, seated, and standing positions D) Taking the blood pressure and heart rate three times, two minutes apart

Answer: C - Taking the blood pressure in the supine, seated, and standing positions

Which of the following statements is CORRECT regarding the short immobilization device? A) Immobilization of the spine is complete B) It should be used on all suspected spine injuries C) The device will keep the torso inline while rotating a victim to a backboard D) Once secure, the patient can be hoisted by the straps from a confined space

Answer: C - The device will keep the torso inline while rotating a victim to a backboard

In Lead II, Einthoven's triangle shows: A) The left arm is negative and the right arm is positive B) The left arm is positive and the right arm is negative C) The left leg is positive and the right arm is negative D) The left leg is negative and the right arm is positive

Answer: C - The left leg is positive and the right arm is negative

Which of the following physical assessment findings would make your patient a candidate for rapid transport to a trauma center? A) Respiratory rate of 24 B) Pulse rate of 110 C) Unstable pelvic girdle D) Systolic blood pressure of 100

Answer: C - Unstable pelvic girdle

Airway thermal burns are generally associated with injuries to the: A) Lungs. B) Entire respiratory system. C) Upper airway. D) Lower airway.

Answer: C - Upper airway

Management of the responsive adult patient with a history acute heart failure who presents with respiratory distress with pulmonary edema, respiratory rate of 32, and SpO2 of 88 percent includes: A) Placing the patient supine position with legs slightly raised. B) Administration of aspirin and epinephrine. C) Use of continuous positive airway pressure. D) Establishing an IV and administering a fluid bolus.

Answer: C - Use of continuous positive airway pressure.

A patient with a closed fracture to the forearm has been properly splinted when which of the following are immobilized? A) Elbow and forearm B) Shoulder, elbow, and forearm C) Wrist, forearm, and elbow D) Wrist and forearm

Answer: C - Wrist, forearm, and elbow

A 23-year-old roofer has fallen 20 feet from the roof of a house into shrubs below. Aside from superficial abrasions and complaint of soreness, you detect no obvious injuries. Consequently, the patient states that as long as his legs are not broken, he does not see the need to go to the hospital. As a knowledgeable AEMT, your BEST response would be: A) "Since this is a work-related injury, you should really be evaluated in the hospital. Workman's comp may not cover you if you do not." B) "Although you feel okay now, you will most likely be sore later on. Why don't we go to the hospital and ask about some medications for pain?" C) "Since you did not lose consciousness, I feel better about having you refuse. Just keep an eye on your feet and watch for swelling." D) "From a fall of this height, you may have hurt some internal organs. You really need to be examined."

Answer: D - "From a fall of this height, you may have hurt some internal organs. You really need to be examined."

Which one of the following statement about removing a helmet in the prehospital setting is TRUE? A) "Helmets should only be removed if they are too tight or spinal immobilization will be required." B) "Since helmets should never be removed, the AEMT must be creative in providing care around the obstacle of a helmet." C) "Any patient wearing a helmet should have it removed so the airway and breathing can be properly assessed." D) "It is acceptable to leave the helmet on a patient if the patient has no airway or breathing problems."

Answer: D - "It is acceptable to leave the helmet on a patient if the patient has no airway or breathing problems."

Your patient has an injury to the elbow and forearm. While you are conducting the reassessment, which one of the following statements made by the patient should concern you most? A) "My arm seems to be bruising more." B) "I am still having a hard time moving my arm." C) "It hurts when I move my fingers." D) "My hand feels like it is going to sleep." Answer: D - "My hand feels like it is going to sleep."

Answer: D - "My hand feels like it is going to sleep."

When obtaining a medical history using the SAMPLE mnemonic, which one of the following statements would be obtained when asking about the "P" component? A) "The pain is rated at 8/10." B) "The pulse is 116 beats per minute." C) "The physician is Dr. Coleman." D) "There is a history of pancreatitis."

Answer: D - "There is a history of pancreatitis."

The AEMT is appropriately using the SAMPLE mnemonic when he asks which one of the following questions? A) "Why did you call for the ambulance?" B) "Who is your doctor?" C) "Have you thought about stopping smoking?" D) "When did you eat last?"

Answer: D - "When did you eat last?"

As an AEMT you are ordered by your medical direction physician to administer 0.3 mg of epinephrine 1:1000 by subcutaneous (SC) injection. Your correct needle length and gauge for this administration would to use a: A) 18 gauge needle - 1.5 inches long B) 21 gauge needle - 1.0 inches long C) 23 gauge needle - 1.5 inches long D) 25 gauge needle - 5/8 inches long

Answer: D - 25 guage needle - 5/8 inches long

The dosage of 50 percent dextrose given intravenously to a hypoglycemic patient is: A) 5-10 cc. B) 50-100 cc. C) 0.25-0.5 mL. D) 25-50 mL.

Answer: D - 25-50 mL.

The rule of nines identifies topographical regions that are approximately: A) 1 percent B) 4.5 percent C) 10 percent D) 9 percent

Answer: D - 9 percent

Which of the following is TRUE regarding a patient with severe respiratory distress? A) Cellular metabolism without adequate oxygen creates an alkalotic state. B) Supporting an open airway is usually sufficient to reverse respiratory distress. C) A blocked airway is the most common cause of respiratory emergencies. D) Accessory muscle use increases oxygen demand during a respiratory emergency.

Answer: D - Accessory muscle use increases oxygen demand during a respiratory emergency.

A patient has been splashed in the face with battery acid. He is complaining of being unable to see anything and severe burning to his face. On closer examination, you note him to be holding both eyes tightly shut and significant burns to his cheeks and eyelids. In this situation, the AEMT would assess the eyes: A) After determining the patient's level of consciousness. B) Prior to assessing the airway. C) Following manual inline spinal immobilization. D) After assessing the patient's radial pulse.

Answer: D - After assessing the patient's radial pulse.

A 15-year-old girl was struck in the mouth with a baseball bat. She has lost her front teeth and is spitting blood. The AEMT's priority concern when caring for this patient would be: A) Seizure activity B) Jaw fracture C) Blood loss D) Airway compromise

Answer: D - Airway compromise

AEMT's may only administer drugs on the order of a licensed physician providing EMS medical direction. Which of the following represents how those orders maybe delivered to the AEMT via the physician? A) Standing orders B) Written orders C) Verbal orders D) All of the above

Answer: D - All of the above

The presence of a skull fracture indicates: A) That rapid transport is necessary. B) A significant mechanism of injury. C) Need for specialized care. D) All of the above

Answer: D - All of the above

What occurs when the body's need for oxygen exceeds the available oxygen supply? A) Anaerobic metabolism B) Hypoxia C) Cyanosis D) All of the above

Answer: D - All of the above

When evaluating the MOI of a sudden deceleration injury in a motor vehicle, the AEMT should: A) Determine if airbag restraints deployed B) Assess the damage to the vehicle itself C) Note if any restraint devices are in use D) All of the above

Answer: D - All of the above

Which of the following are considered peripheral veins that can be used for cannulation? A) External jugular veins B) Hand veins C) Forearm veins D) All of the above

Answer: D - All of the above

Which of the following is helpful in determining the severity of abdominal injury? A) Clinical exam findings B) Mechanism of injury C) Patients complaint D) All of the above

Answer: D - All of the above

Which of the following is important information to gather from a patient suspected of pneumonia? A) Acute onset of chills and fever B) History of recent bout of bronchitis C) Dyspnea that is progressively worsening D) All of the above

Answer: D - All of the above

What anti-inflammatory analgesic is indicated for patients with suspected myocardial infarction? A) Versed B) Nitroglycerin C) Morphine D) Aspirin

Answer: D - Aspirin

f left untreated, glaucoma can lead to what? A) Presbyopia B) Hyperopia C) Myopia D) Blindness

Answer: D - Blindness

A patient states that he is short of breath, lightheaded, and has chest pain that worsens when he takes a deep breath. Assessment reveals an open airway, adequate breathing, and a rapid pulse of 120 beats per minute. His skin is hot to the touch, and he has a blood pressure of 116/84 mmHg. The patient informs you that he was diagnosed with pneumonia three days ago and has not been taking the prescribed antibiotics. Which one of these assessment findings would the AEMT BEST recognize as a sign related to the patient's chief complaint? A) Recent diagnosis of pneumonia B) Complaint of lightheadedness C) Chest pain that worsens with inspiration D) Blood pressure of 116/84 mmHg

Answer: D - Blood pressure of 116.84 mmHg

A patient complaining of substernal chest pain tells you that it feels the same as when he had a heart attack two years ago. He is diaphoretic, nauseated, vomiting, and has a blood pressure of 78/50 mmHg and heart rate of 84. He also is allergic to aspirin. He asks you to assist him in taking one of his nitroglycerin pills. Which one of the assessment findings that follows represents a contraindication to the administration of the nitroglycerin? A) Chest pain rated less than 5 on a scale of 1 to 10 B) Heart rate below 100 C) Nausea and vomiting D) Blood pressure of 78/50 mmHg

Answer: D - Blood pressure of 78/50 mmHg

A patient complaining of substernal chest pain tells you that it feels the same as when he had a heart attack two years ago. He is diaphoretic, nauseated, vomiting, and has a blood pressure of 78/50 mmHg and heart rate of 84. He also is allergic to aspirin. He asks you to assist him in taking one of his nitroglycerin pills. Which one of the assessment findings that follows represents a contraindication to the administration of the nitroglycerin? A) Nausea and vomiting B) Heart rate below 100 C) Chest pain rated less than 5 on a scale of 1 to 10 D) Blood pressure of 78/50 mmHg

Answer: D - Blood pressure of 78/50 mmHg

By echoing the orders from medical direction you: A) Confirm your reception of the order B) Confirm your understanding of the order C) Question the authority of the medical director D) Both A and B are correct

Answer: D - Both A and B are correct

In the mnemonic DCAP-BTLS, "B" stands for: A) Breaks B) Bleeding C) Bruises D) Burns

Answer: D - Burns

Your patient has a history of viscous mucus that blocks the ducts of the exocrine glands that secrete digestive enzymes into the digestive tract. These thick secretions also obstruct the airways. What is his diagnosis? A) COPD B) Pneumonia C) Emphysema D) Cystic fibrosis

Answer: D - Cystic fibrosis

A patient involved in a motor vehicle collision is complaining of neck pain. Which of the following pieces of information should cause the AEMT to have a high index of suspicion that the patient may have suffered more significant injuries beyond the complaint of neck pain? A) No airbags in the car B) History of heart problems C) Complaint of head and neck pain D) Death of the car's driver

Answer: D - Death of the car's driver

When assessing a patient's pupils, the AEMT should: A) Instruct the patient to blink several times and observe the size of the pupils B) Cover one eye and shine a light in the other, watching for changes to the size of the pupil C) Shine a light in one eye and watch for the pupil to dilate D) Determine the size of the pupil and then look for a change in pupillary size as you shine a light in the eye

Answer: D - Determine the size of the pupil and then look for a change in pupillary size as you shine a light in the eye

Pulse pressure is the: A) Difference between systolic blood pressure and pulse rate B) Sum of diastolic blood pressure and pulse rate C) Sum of systolic and diastolic blood pressures D) Difference between systolic and diastolic blood pressures

Answer: D - Difference between systolic and diastolic blood pressures

Which of the following statements regarding eye contact with a patient is NOT true? A) It shows you are interested in the patient B) It shows you are confident C) It shows you are attentive D) Eye contact is always appropriate

Answer: D - Eye contact is always appropriate

The Cincinnati Prehospital Stroke Scale evaluates possible stroke patients on what three parameters? A) Age, duration of symptoms, abnormal speech B) Arm drift, blood glucose levels, history seizures C) Headache, arm drift, onset of symptoms D) Facial droop, arm drift, abnormal speech

Answer: D - Facial droop, arm drift, abnormal speech

The pancreas secretes two major hormones, including: A) Prolactin. B) Calcitonin. C) Aldosterone. D) Glucagon.

Answer: D - Glucagon.

A patient is being extricated from a car using a vest-type short immobilization device. After the patient has been extricated, the AEMT should: A) Secure the patient and vest-type short immobilization device in a supine position on the stretcher with the feet elevated B) Place the patient in a semi-Fowler's position on the stretcher for transport to the hospital C) Remove the vest-type short immobilization device and secure the patient to a long backboard D) Immobilize the patient with the vest-type short immobilization device to a long backboard

Answer: D - Immobilize the patient with the vest-type short immobilization device to a long backboard

Spinal injury care steps performed during the primary assessment include: A) Apply a cervical collar independent of spinal stabilization. B) Administer medications to prevent further paralysis. C) Perform a rapid rollout independent of manual spinal stabilization. D) Initiate proper manual spinal stabilization followed by mechanical immobilization.

Answer: D - Initiate proper manual spinal stabilization followed by mechanical immobilization.

The bony labyrinth is another name for the ________. A) Outer ear B) Auditory nerve C) Middle ear D) Inner ear

Answer: D - Inner Ear

What are the two endocrine hormones released by the pancreas? A) Epinephrine and norepinephrine B) Glucose and glycogen C) Dopamine and neurontin D) Insulin and glucagon

Answer: D - Insulin and glucagon

Which of the following MOST accurately depicts the chest wall movement in a patient with flail segment? A) Inward with both inspiration and expiration B) Inward with expiration and outward with inspiration C) Outward with both inspiration and expiration D) Inward with inspiration and outward with expiration

Answer: D - Inward with inspiration and outward with expiration

A patient with pancreatic cancer exhibits a yellow discoloration to his entire body. The AEMT would correctly document this condition as: A) Pallor B) Flushed C) Cyanotic D) Jaundice

Answer: D - Jaundice

You male patient has a history of mental illness and has inserted a cotton-tipped swab into his urethra. He is complaining of dysuria. How will you manage this patient? A) Remove the swab from his penis. B) Apply pressure to the area. C) Deliver a fluid bolus via IV. D) Leave the object in place.

Answer: D - Leave the object in place.

Which of the following would be MOST typical of a patient suffering from diverticulitis? A) Diffuse abdominal pain B) Dark, tarry stools C) Constipation D) Left lower quadrant pain

Answer: D - Left lower quadrant pain

For which one of the following conditions would the AEMT MOST likely get an inaccurate pulse oximeter reading despite proper application of the sensor? A) Increased heart rate B) Elevated body temperature C) Unresponsiveness D) Low blood pressure

Answer: D - Low blood pressure

Which one of the following is the BEST indication for the administration of oral glucose to a patient who has diabetes? A) Chest pain B) History of diabetes C) Stroke D) Low blood sugar

Answer: D - Low blood sugar

You are evaluating a patient who is experiencing pain, deformity, and a loss of pulse to his left upper extremity following a riding accident. You suspect a dislocated shoulder. Based on this information, you should: A) Provide oxygen and paralytics to reduce the fracture site B) Make multiple attempts at resetting the joint into socket C) Splint the shoulder in position found and transport rapidly D) Make one attempt to manipulate the joint while checking distal pulse

Answer: D - Make one attempt to manipulate the joint while checking distal pulse

You are caring for a 21-year-old woman who is apparently suffering from food poisoning. She has been forcefully vomiting for several hours and has begun to vomit a large amount of blood. What should you suspect? A) Esophageal varices B) Peptic ulcer C) Esophagitis D) Mallory-Weiss tear

Answer: D - Mallory-Weiss tear

When a victim at a MVC is found to be standing outside of the vehicle and requires immobilization, the AEMT should: A) Place cervical collar, stabilize the cervical spine, seat on backboard and secure B) Manually stabilize the c-spine, apply cervical collar, and carefully seat patient on board C) Stabilize cervical spine, place KED, move to board and secure D) Manually stabilize the c-spine, place collar, and perform standing takedown

Answer: D - Manually stabilize the c-spine, place collar, and perform standing takedown

An analgesic is a pain reliever. Which of the following is NOT an analgesic potentially administered by the AEMT? A) Aspirin B) Nitrous oxide C) Acetaminophen D) Morphine

Answer: D - Morphine

In which of the following disorders are pathological fractures MOST commonly seen? A) Lymphadenopathy B) Sickle cell C) Anemia D) Multiple myeloma

Answer: D - Multiple myeloma

A test may be run to check for creatine phosphokinase in the blood. This test measures: A) Acid levels. B) Protein levels. C) Jaundice. D) Muscle damage.

Answer: D - Muscle damage.

Skin color is BEST evaluated at the: A) Wrists and neck B) Limbs and torso C) Cheeks and nose D) Nail beds and conjunctiva

Answer: D - Nail beds and conjunctiva

Your patient has been impaled on a metal fence. The object is through the medial thigh and the patient is still on the fence. You should: A) Contact medical direction regarding tourniquet placement for bleeding control. B) Carefully lift the patient off the fence and control hemorrhage and treat for shock. C) Cut the fence below the patient and then remove the post while controlling bleeding. D) None of the above

Answer: D - None of the above

A mnemonic for remembering the steps for determining the present illness is: A) SAMPLE B) HEENT C) ABCDE D) OPQRST

Answer: D - OPQRST

A 14-year-old boy fell 10 feet from a retaining wall and hit his head on a metal post. He is responsive to verbal stimuli with incomprehensible speech. The secondary assessment indicates blood and fluid coming from inside the patient's left ear. You should do which one of the following? A) Suction the blood from the ear so you can determine the cause of bleeding. B) Tilt the backboard to the left to allow the ear to drain freely. C) Place a folded piece of gauze in the ear canal to stop the drainage. D) Place a piece of sterile gauze over the ear.

Answer: D - Place a piece of sterile gauze over the ear.

You are assessing a patient who had a previous head injury in which a portion of the cerebellum was destroyed. Which one of the following signs and symptoms would the AEMT expect to find in relation to this injury? A) Inability to move or feel sensations in one side of his body B) Inability to remember information such as a past medical history C) Problems with the regulation of heart rate and blood pressure D) Poor coordination when signing his name to the prehospital care report

Answer: D - Poor coordination when signing his name to the prehospital care report

Which of the following is the MOST appropriate prehospital management for a patient with flail segment? A) Placing IV fluid bags on the chest B) Placing a sandbag that weights at least 15 pounds on the injured side C) Needle decompression D) Positive pressure ventilation

Answer: D - Positive pressure ventilation

You have arrived on the scene of an explosion in a warehouse used to store fertilizers. A paramedic, who has already triaged the nine patients involved, directs you to a young man sitting under a tree. The man states that he is having excruciating ear pain. The primary assessment and secondary assessment reveal no obvious injuries. Given this presentation, the AEMT would recognize which of the following? A) Inner ear damage caused by the noise of the explosion B) Ear damage caused by exposure to superheated air C) Secondary blast injury to the inner ear D) Primary phase blast injury to the inner ear -

Answer: D - Primary phase blast injury to the inner ear

You have arrived by the side of a cyanotic patient who is responsive only to painful stimuli and exhibiting stridorous respirations. Which one of the following would be MOST appropriate? A) Prepare and apply the pulse oximeter before administering oxygen B) Contact medical direction for instructions on how to manage the patient C) Obtain a full set of vital signs excluding the patient's temperature D) Provide positive pressure ventilation with supplemental oxygen

Answer: D - Provide positive pressure ventilation with supplemental oxygen

As you are treating a patient who has had blunt trauma to the chest, during transport, oxygen saturation is falling and the patient is experiencing hemoptysis. Lung sounds are clear and equal bilaterally, but diminished due to painful breathing. You suspect: A) Simple pnuemothorax B) Cardiac tamponade C) Tension pnuemothorax D) Pulmonary contusion

Answer: D - Pulmonary contusion

Signs of hypoperfusion in infants include all of the following EXCEPT: A) Pale, ashen, or cyanotic skin B) Altered mental status C) Capillary refill time greater than two seconds D) Pulse rate greater than 100 beats per minute `

Answer: D - Pulse rate greater than 100 beats per minute

The purpose of the secondary assessment is to: A) Identify lifethreats to the ABCs B) Assess neurological status C) Determine all injuries present on a victim D) Quickly assess areas of the body where serious life threats may occur

Answer: D - Quickly assess areas of the body where serious life threats may occur

Your patient opens his eyes only when you pinch his shoulder. He has no verbal response and withdraws from pain. His Glasgow Coma score total is: A) Two. B) Four. C) Three. D) Seven.

Answer: D - Seven

Significant injuries to the head and neck require: A) Detailed neurological evaluation on scene. B) Intubation. C) Only BLS intervention. D) Spinal immobilization.

Answer: D - Spinal immobilization

Which of the following is often a result of the underlying problem of benign prostatic hypertrophy? A) Renal calculi B) Polycystic renal disease C) Uremic frost D) Urinary retention

Answer: D - Urinary retention

Vomiting agents effect the human body​ by: A. binding with acetylcholine causing continuous stimulation of the parasympathetic nervous system B. increasing gastric acid production leading to metabolic acidosis and vomiting C. targeting the hypothalamus of the brain leading to increased emesis D. causing irritation to the upper respiratory tract and eyes

D. Vomiting agents are aerosolized and cause signs and symptoms when inhaled. They are potent and only a small dose is needed to cause distress. Contrary to the​ name, vomiting agents mostly cause irritation of the upper respiratory tract and the​ eyes, but the accompanying coughing and gagging can lead to vomiting.

What biologic agent is in the most virulent​ category, category​ A?

Smallpox

The AEMT understands the importance of evaluating the mechanism of injury when he states: A) "The mechanism of injury is useful in determining the exact injuries the patient has sustained." B) "The mechanism of injury can be used to guide your assessment and treatment of the patient." C) "The mechanism of injury is a useful tool in determining if the patient's outcome will be good or bad." D) "Evaluating the mechanism of injury is important since it determines whether emergency transport to the hospital is needed."

B) "The mechanism of injury can be used to guide your assessment and treatment of the patient."

The typical dose for nebulized albuterol in pediatric patients is: A) 0.10 mg/kg B) 0.15 to 0.3 mg/kg C) 1-2 mg/kg D) 0.3-0.5 mg/kg

B) 0.15 to 0.3 mg/kg

A very young pediatric patient who loses _____percent of their circulating blood volume can go into compensated shock. A) 10 B) 15 C) 20 D) 25

B) 15

__________ are abrupt, high-dose single exposures. A) Toxicants B) Acute exposures C) Toxidromes D) Chronic exposures

B) Acute exposures

You are called for an elderly man with chest pain but you have difficulty understanding him. His wife tells you that he has Bell's palsy which causes paralysis of facial nerves. You should suspect: A) Physical abuse B) An articulation disorder C) A language disorder D) A fluency disorder

B) An articulation disorder

Your 18-year-old patient was involved in an altercation and has a puncture wound to his right mid-thorax. You should: A) Apply a moist dressing and seal on all four sides B) Apply an occlusive dressing and seal on only three sides C) Apply a moist dressing and seal on only three sides D) Apply an occlusive dressing and seal on all four sides

B) Apply an occlusive dressing and seal on only three sides

A mnemonic to help remember the effects associated with nerve agent poisoning is: A) WMD. B) DUMBELS. C) CBRNE. D) SAMPLE.

B) DUMBELS

All of the following are complications that can result from preeclampsia EXCEPT: A) Hemorrhagic stroke B) Decreased deep tendon reflexes C) Sudden placental abruption D) Acute kidney failure

B) Decreased deep tendon reflexes

Alzheimer's is the MOST recognizable form of ________ in the elderly. A) Delirium B) Dementia C) Dysarthia D) Bipolar disorder

B) Dementia

The cumulative effects of vascular disease and inflammatory processes can lead to: A) Alzheimer's disease B) Dementia C) Proprioception D) Delirium

B) Dementia

As a member of the emergency medical services team, an AEMTs role includes all of the following EXCEPT to: A) Triage trauma patients B) Dispatch equipment C) Transport patients to the appropriate hospital D) Provide rapid assessment

B) Dispatch equipment

An 83-year-old woman has fallen down the basement stairs. She complains of left hip pain and rates it 10/10. Since the patient is downstairs, you immediately recognize the need for assistance in extricating her to the ambulance. As you start the primary assessment, you note a dog at the top of the stairs growling. At this time, what is your primary concern? A) Pain B) Dog C) Need for additional help D) Airway

B) Dog

An ectopic pregnancy MOST commonly occurs in the: A) Pelvic cavity B) Fallopian tube C) Uterus D) Abdominal cavity

B) Fallopian tube

You have initiate chest compressions for a neonate with a pulse rate of 50 and he is improving steadily. You should next: A) Begin ventilations at 30 breaths per minute B) Gain IV access and deliver IV fluids C) Continue chest compressions at 80 per minute D) Cut the umbilical cord and tie it off

B) Gain IV access and deliver IV fluids

Which of the following would be an example of child neglect? A) Burning a child with a cigarette lighter B) Leaving a two-year-old child in the tub alone C) Spanking a child repeatedly with a cord D) Locking a child in a closet at night

B) Leaving a two-year-old child in the tub alone

A car has been hit head-on by another vehicle at a moderate rate of speed. Seatbelts were in place and the airbags deployed. The patient is complaining of chest pain. Quick observation reveals a bruise to his sternum. The AEMT would immediately: A) Deflate the airbag to get breath sounds B) Lift the airbag and check the steering wheel C) Suspect an injury related to the airbag D) Check the windshield for starring

B) Lift the airbag and check the steering wheel

Biological weapons are often hard to identify because they: A) Change their form upon contact with the host B) Often do not have any immediate symptoms C) Do not vaporize readily D) Are vaporized with the initial explosions

B) Often do not have any immediate symptoms

At the scene of a multiple-casualty incident, you are triaging a patient using the Simple Triage and Rapid Transport (START) system. You are presented with a patient who is not breathing. Which one of the following should you do next? A) Place a black tag on the patient B) Open the airway C) Start positive pressure ventilation D) Place a red tag on the patient

B) Open the airway

When spinal immobilizing a pediatric patient to a long spine board, which of the following is acceptable? A) Immobilize the child to the stretcher if you do not have a pediatric board B) Padding under the shoulders will maintain cervical spine alignment C) Cervical collars are not required since their necks are so short D) If the child is too upset, it is better to not spinal immobilize them

B) Padding under the shoulders will maintain cervical spine alignment

Which of the following statements about pneumonia in the elderly patient is FALSE? A) It may have a bacterial or viral cause B) Pain may be referred to the umbilicus C) It may present without any fever D) It may be community acquired or nosocomial

B) Pain may be referred to the umbilicus

A 10-year-old boy was struck by a car while riding his bike. He has blood on his face and presents as unresponsive with snoring respirations. Respirations are approximately 10 per minute, with cyanosis noted to his extremities. Manual inline spinal stabilization is being held by EMRs. Which one of the following should you do next? A) Look for life-threatening injuries B) Perform a jaw-thrust maneuver C) Suction the airway D) Administer positive pressure ventilation

B) Perform a jaw-thrust maneuver

The trauma system is based on the principle that serious trauma is a: A) Medical disease B) Surgical disease C) Trauma disease D) Psychological disease

B) Surgical disease

Following the destruction of a small community by a tornado you are on the scene of a multiple casualty incident. Which of the following injury types would you most likely​ expect? A. complex lower extremity fractures B. blunt and penetrating trauma C. compartment syndrome D. barotrauma to the lungs

B. Due to the extreme forces and destructive nature of​ tornados, the responders should expect penetrating trauma from flying projectiles and blunt trauma from contact with flying heavy debris.

An example of a Category A biologic agent​ is: A. salmonella B. anthrax C. hantavirus D. cryptosporidium

B. Examples of Class A biologic agents include​ anthrax, botulism,​ plague, smallpox,​ tularemia, and viral hemorrhagic fevers.

You are caring for a young woman who has been stung by a wasp. There is redness and a hard knot where she was stung. Her vital signs are stable and she denies respiratory distress. Which of the following is MOST appropriate for this patient? A) Administer 0.3 mg of epinephrine SQ. B) Assist her with her EpiPen™. C) Continue with your focused history. D) Administer nebulized albuterol.

C

You respond for a patient showing signs of anaphylaxis. During your history, you discover that she has just starting taking Vasotec for hypertension. Which of the following should you suspect? A) An immediate hypersensitivity response B) Angioneurotic edema C) An anaphylactoid reaction D) An IgE mediated allergic reaction

C

A car swerved to miss a deer in the road and hit a utility pole at a low rate of speed. Damage to the car was minimal, but the 82-year-old male passenger was not wearing a seatbelt and hit his head on the dashboard. He presents as confused and mildly diaphoretic. His daughter, the driver, states that he has a history of dementia, high blood pressure, and diabetes, as well as coronary artery disease. She adds that he did take his insulin about two hours ago. There were two other passengers in the rear seats of the vehicle. Which one of the following questions to the daughter would be MOST important at this time? A) "Does he take high blood pressure pills?" B) "How far along is the dementia?" C) "Did he eat anything today?" D) "He seems sweaty. Was he feeling okay this morning?"

C) "Did he eat anything today?"

A technique that the terrorist may use which involves both conventional and WMD characteristics is/are: A) Nuclear detonation B) Biological attack C) "Dirty bombs" D) Chemical release

C) "Dirty bombs"

You are ventilating a neonate but he doesn't seem to be responding. You know that you MUST begin chest compressions the heart rate drops below ________ beats per minute. A) 100 B) 80 C) 60 D) 40

C) 60

You are transporting an elderly woman who fell in a nursing home and has outward rotation of her left leg. She has a lengthy medical history, including dementia. Currently, she is very quiet and receiving oxygen through a nasal cannula. She also is immobilized to a long spine board. During your reassessment, you note her to be alert but confused, with slightly accelerated respirations and a strong radial pulse. You also note that her SpO2 has dropped from 97 percent on scene to 91 percent. You should: A) Request ALS assistance B) Perform a head-tilt/chin-lift maneuver C) Apply a nonrebreather D) Start positive pressure ventilation

C) Apply a nonrebreather

A 31-year-old groundskeeper has been stung by wasps three times in his right arm and left ankle. After performing the primary and secondary assessments, what should you do next? A) Place a constricting band above the sting sites. B) Apply ice packs to the sting site for pain relief. C) Attempt removal of any embedded stingers. D) Wash the sting sites with sterile water and soap.

C) Attempt removal of any embedded stingers.

Which of the following is NOT part of the pediatric assessment triangle when forming your general impression of the patient's condition? A) Appearance B) Work of breathing C) Breath sounds D) Circulation to the skin

C) Breath sounds

You are working at a multiple-casualty incident (MCI) involving the use of an incendiary device that was thrown into a large group of people. The victims could be expected to have: A) Contamination from biological agents B) Fractures from the initial explosion C) Burn injuries D) Possible radiation exposure

C) Burn injuries

You are working the scene of a collision involving a tractor trailer that is hauling an unknown chemical. After it has been determined that no material has been spilled, the rescue chief asks you to get the truck's shipping papers. Where would these MOST likely be located? A) Rear compartment on the tanker B) Electrical box between the tractor and trailer C) Cab of the truck D) Shipping vault on the trailer

C) Cab of the truck

You are dispatched to an auto collision. Because of very heavy traffic, your response time is 25 minutes. The closest hospital is 40 miles from the accident site. You should: A) Request that law enforcement escort you to a hospital B) Request additional ground units C) Call for air transport D) Have state law enforcement stop traffic while you transport

C) Call for air transport

A common condition that affects the elderly and causes an impairment of vision is/are: A) Chemical conjunctivitis B) Astigmatism C) Cataracts D) Blindness

C) Cataracts

As an EMS​ provider, if you are first on the scene and see signs that the event is an MCI as a result of natural disaster or​ terrorism, what should you​ do?

Stay in your vehicle and perform a windshield assessment.

As you approach the scene of a mass casualty situation in a downtown office​ building, you note the slight odor of almonds as you see many victims in the lobby lying unconscious with no apparent injury. Your BEST actions should​ be:

remain​ upwind, away from the​ site, as this is possibly an asphyxiant.

Which of the following is usually associated with an anaphylactoid reaction? A) Radiological dyes B) Shellfish C) Tree nuts D) Latex exposure

A

Which of the following is your FIRST priority when responding to the scene of a patient complaining of anaphylaxis? A) Scene safety B) Respiratory support C) Airway maintenance D) Fluid resuscitation

A

Which sign associated with an allergic reaction should concern a caregiver the MOST? A) Wheezing B) Warm, flushed skin C) GI distress D) Urticaria

A

Emergency Resource Guidebook

A small truck has gone over an embankment. The driver has suffered a head injury and is confused, but is able to tell you that he is hauling N-acetyl-para-aminophenol. Which one of the following resources provides the EMT with the best initial information regarding this chemical?

the patient represents no contamination threat to you.

A train carrying radioactive materials has derailed. The radiation safety officer informs you that you will be receiving a 31-year-old male who was exposed to the radioactive material. As such, you should recognize that:

Assess his level of consciousness and ABCs.

A worker has been exposed to beta rays given off by a radioactive material. Which one of the following should you do first?

After determining that a patient intentionally took an overdose of blood pressure medications, which of the following questions by the AEMT would be MOST appropriate? A) "How many did you take?" B) "Are you on other medications?" C) "Why did you take so many?" D) "Were you confused about the proper dose?"

A) "How many did you take?"

A patient has been bitten by an unidentified snake. Which one of the following statements made by the patient would lead the AEMT to believe that the snake was poisonous? A) "Its head was triangular." B) "Its eyes were round." C) "It was about two feet long." D) "It had a stripe down its back."

A) "Its head was triangular."

You have been summoned to a beach for a young male who has been stung by jellyfish on both forearms. Which one of the following instructions to other AEMTs shows proper care of this patient? A) "Let us soak the arms in warm water." B) "Let us put a very tight constricting band on his wrists." C) "Make sure to keep the arms below the heart." D) "We need to rinse the sting sites with peroxide."

A) "Let us soak the arms in warm water."

A car has rolled several times and is on its roof. Closer inspection reveals the unrestrained driver lying in her back with the back portion of the roof collapsed. At this time, which one of the following instructions should you give FIRST? A) "We need to take the weight off of the posts." B) "Let us open the doors and use them for support." C) "We need to climb in to assess the patient." D) "Let us forcibly remove the back door."

A) "We need to take the weight off of the posts."

A group of AEMTs is preparing to restrain a large and violent patient. Which one of the following instructions given by the team leader is MOST appropriate? A) "You distract him by coming at him from the front and the rest of us will come at him from different directions, each taking a separate limb." B) "You grab his head and the rest of us will grab his legs and chest, and then let's get him face down on the stretcher." C) "Since you are trained in martial arts, try to give him a kick to his stomach, then when he goes down, the rest of us will grab his arms and legs." D) "Two of us will get him from the front and the other two from the back. Let us do this slowly so no one gets hurt."

A) "You distract him by coming at him from the front and the rest of us will come at him from different directions, each taking a separate limb."

You have been dispatched to a motor-vehicle collision. On-scene you find that a minivan has completely sheared a wooden utility pole. The electrical wires are draped over the hood of the minivan and the driver, a woman in her 20s, is screaming that she is not hurt and wants to get out of the vehicle. Further observation of the wires reveals that they are not moving, arcing, or making any noise. Which one of the following instructions is MOST appropriate at this time? A) "You need to remain still and calm in your seat and do not touch anything in the car." B) "Carefully get out of the car and shuffle away with small steps while keeping your feet together." C) "I need you to jump clear of the car and shuffle away in small steps keeping your feet together." D) "The electrical wires are not charged anymore, but stay where you are until the fire department gets here."

A) "You need to remain still and calm in your seat and do not touch anything in the car."

Following a large wedding, health authorities announce that some of the food was contaminated with salmonella. Consequently, your EMS service receives notice that there may be a significant number of calls involving food poisoning. A new AEMT asks you how to treat those with food poisoning. You would reply: A) "You should treat the patient based on his signs and symptoms, just like any other ingested poison." B) "After assessing the patient, the administration of activated charcoal can be effective if there are no obvious contraindications." C) "The primary role of EMS in this situation is merely to transport to the hospital." D) "If the patient has a fever, we can administer a medication to make him vomit to get the food out of his system."

A) "You should treat the patient based on his signs and symptoms, just like any other ingested poison."

Medical direction has ordered the administration of activated charcoal to a three-year-old boy. What dose would be MOST appropriate? A) 1 gram/kilogram B) 0.5 mg/kilogram C) 50 grams D) 100 mg

A) 1 gram/kilogram

Body core temperature in a patient suffering from heatstroke will usually be at least: A) 104°F. B) 102°F. C) 107°F. D) 100°F.

A) 104°F.

Which of the following would be your likely initial medication treatment when a patient has been exposed to a small amount of an experimental nerve agent? A) 2 mg of atropine and 600 mg of pralidoxime chloride B) 0.3 mg of epinephrine (1:1000) and 25 grams of dextrose 50% C) 0.4 mg of nitroglycerine and 325 mg of aspirin D) 10 mL of sodium chloride and 2 mg of naloxone

A) 2 mg of atropine and 600 mg of pralidoxime chloride

Which of the following is TRUE regarding the elderly? A) A decrease in estrogen leads to increased cardiovascular disease in postmenopausal women B) Cardiac output remains constant, but hypercholesterolemia and hypertension increase C) There is a marked increase in obesity which is associated with the onset of type 1 diabetes D) Osteoporosis can occur in both genders, but is especially common in men

A) A decrease in estrogen leads to increased cardiovascular disease in postmenopausal women

All of the following increase the risk for neonatal resuscitation EXCEPT: A) A mother who is over the age of 25 B) The mother has taken cocaine during pregnancy C) A baby who is born at 36 weeks of gestation D) The mother has type 1 diabetes

A) A mother who is over the age of 25

________ is the premature separation of the placenta from the uterine wall. A) Abruptio placenta B) Cervical dilation C) Eclampsia D) Placenta previa

A) Abruptio placenta

You are transporting a 54-year-old man who was working in his yard spraying a pesticide called Parathion. Although he was using a particulate respirator, he suddenly started experiencing dizziness, severe abdominal cramps, and vomiting. What is MOST likely route of toxic exposure? A) Absorption B) Injection C) Inhalation D) Ingestion

A) Absorption

A patient has taken a friend's prescription of Restoril and has become extremely lethargic, exhibits slurred speech, and is semiconscious. Which medication might medical direction advise you to administer? A) Activated charcoal B) Thiamine C) Diazepam D) Naloxone

A) Activated charcoal

Your patient is a three-year-old lethargic child whom you have determined to be hypoglycemic. Her mother tells you that the child weighs 44 pounds. Which of the following is the BEST treatment for this patient? A) Administer 20 mL of 25 percent dextrose IV B) Deliver 25 mL of 10 percent dextrose IV C) Administer 1 mg of glucagon IM D) Give her a sucker to quickly raise her blood sugar

A) Administer 20 mL of 25 percent dextrose IV

An elderly patient has fallen in her home and activated the 911 system by pressing a medical alarm worn around her neck. On arrival you can see the patient on the floor, but she cannot come to the door to open it to let you in. In this situation, your BEST course of action is to: A) Break a window in a room away from where the patient is located B) Open the storm door in the front and kick open the front door C) Use a crowbar to pry open a window or door D) Contact a locksmith to pick the locks on the front door

A) Break a window in a room away from where the patient is located

Which of the following is an example of a chemical that may be employed as a weapon of mass destruction? A) Chlorine B) Gamma radiation C) Botulism toxin D) Anthrax

A) Chlorine

You are by the side of a young male patient with a history of behavioral problems. Friends called 911 after the patient experienced several violent outbursts this evening. Which one of the following findings would cause the AEMT to suspect a physical cause for this behavior, as opposed to a behavioral one? A) Dilated pupils that are sluggish to react B) Fear that friends are out to steal his money C) Hearing his mother's voice in his head D) Difficulty making eye contact with the AEMT

A) Dilated pupils that are sluggish to react

As a member of the emergency medical services team, an AEMT role includes all of the following EXCEPT to: A) Dispatch equipment B) Triage trauma patients C) Transport patients to the appropriate hospital D) Provide rapid assessment

A) Dispatch equipment

For your safety and that of your crew, you should: A) Evaluate every scene for dangers before entering B) Request police assistance on all calls C) Ensure the availability of a paramedic for all medical calls D) Search each and every patient for weapons

A) Evaluate every scene for dangers before entering

Fertilization normally will occur in the: A) Fallopian tube B) Ovaries C) Vagina D) Uterus

A) Fallopian tube

You have arrived on the scene of a chemical leak at an industrial park. The placard has a "1" in the blue diamond and a "4" in the red diamond, which means the material is: A) Flammable but represents a low health hazard B) A flammable liquid with radioactive properties C) Explosive with a high health hazard associated D) A solid that is very flammable

A) Flammable but represents a low health hazard

Speech impairment chiefly occurs because of four reasons: articulation disorders, voice production disorders, language disorders, and: A) Fluency disorders B) Mentation processing disorders C) Neurovascular disorders D) Glossal function disorders

A) Fluency disorders

Your patient has been pregnant three times with two live deliveries. You should document this as: A) Gravida 3, para 2 B) Gravidity 2, para 3 C) Primagravida D) Para 3, gravida2

A) Gravida 3, para 2

You have arrived on the scene to help a child who is short of breath. The mother is screaming hysterically that her three-year-old son cannot breathe, making assessment of the patient very difficult. What should be your initial action in managing this situation? A) Have your partner talk to the mother while you assess the child B) Summon the police to the residence to subdue the mother C) Quickly remove the child to the ambulance and assess while transporting D) Inform the mother that you will call the police if she does not calm down

A) Have your partner talk to the mother while you assess the child

You are providing care for a three-year-old boy who has fallen down a set of stairs in his home. He is crying as his mother tries to console him. You are concerned about the possibility of internal bleeding. Which of the following is CORRECT regarding this patient? A) He may not show signs or symptoms of shock until very late B) He will not likely go into shock because of his ability to compensate C) A blood pressure of 90 mmHg would indicate shock D) A capillary refill time of two seconds would rule out shock

A) He may not show signs or symptoms of shock until very late

You are on the scene of a motor-vehicle collision involving two cars. To remove the driver of the first car, rescuers must break the windshield and roll back the roof. While he is performing this operation, which of the following sets of personal protective equipment will afford the AEMT the BEST protection? A) Helmet, goggles, turnout gear, leather gloves, and steel-toed boots B) Goggles, latex exam gloves, steel-toed boots, and fluorescent vest C) Sunglasses, steel-toed boots, and fluorescent identification vest D) Latex exam gloves, fluorescent identification vest, and helmet

A) Helmet, goggles, turnout gear, leather gloves, and steel-toed boots

You respond to a call of an explosion at a local chemical company. Upon arrival, the company's hazmat team is decontaminating patients so you can easily treat them. The team extricates a 35-year-old man who is experiencing slight respiratory distress. The care that you should provide FIRST for this patient is: A) High-flow oxygen B) An immediate head-to-toe trauma assessment C) Transport to the local trauma center D) Insertion of a supraglottic airway

A) High-flow oxygen

You are transporting a patient who was bitten on her hand by a spider about 20 minutes ago. On-scene the primary assessment revealed no life threats and vital signs were stable. While conducting your ongoing assessment, which one of the following would be of MOST concern to the AEMT? A) Hives to the chest and abdomen B) Swelling to the area of the bite C) Complaint of a headache D) Increased pain to the area of the bite

A) Hives to the chest and abdomen

Which of the following is the leading cause of cardiac arrest in pediatric patients? A) Hypoxia B) Cardiac dysrhythmias C) Trauma D) Congenital defects

A) Hypoxia

A localized terrorist attack involves pepper spray being released at a high school dance. As you respond, the incident commander reports approximately 50 people have been exposed and are calling for the county disaster plan to be implemented. Given the nature of the weapon, which one of the following should you anticipate doing on scene? A) Irrigating eyes with saline B) Administering an aerosolized antidote C) Providing positive pressure ventilation D) Decontaminating patients with diluted bleach

A) Irrigating eyes with saline

In relation to an adult's head, the pediatric patient will have a: A) Larger head B) Smaller occipital bone C) Relatively smaller head size D) Larger facial area

A) Larger head

Which level of trauma center commits resources to address all types of specialty trauma 24 hours a day, 7 days a week? A) Level I B) Level II C) Level III D) Level IV

A) Level I

ou are called to a long-term care facility for a patient with altered mental status and a fever. The staff reports that the patient has an indwelling catheter and he has been passing dark colored urine. He is hypotensive with an SpO2 saturation of 87 percent. You should suspect: A) Life-threatening sepsis B) A severe spinal infection C) A blocked intraventricular shunt D) A contaminated ostomy bag

A) Life-threatening sepsis

You have been asked to deliver a talk about EMS and the geriatric population. Which one of the following points would you emphasize in your presentation? A) Many of the elderly have a combination of different diseases in various stages B) Assessment can be difficult since most persons older than 70 suffer from dementia C) Dementia is an inevitable part of the aging process D) Geriatrics actually comprises a small number of EMS calls and transports

A) Many of the elderly have a combination of different diseases in various stages

A young child involved in a multiple-casualty incident is found lying on the ground with obvious deformity to the left thigh. Using the Jump START system for triage, you note that he is not breathing. Which one of the following should you do next? A) Open the airway, and then check for a radial pulse B) Provide 30 seconds of positive pressure ventilation C) Tag the patient as red, and then move him to the treatment area D) Tag the patient as black, and then move to the next one

A) Open the airway, and then check for a radial pulse

You are the first to arrive on the scene of a motor-vehicle collision. Your scene size-up reveals a tanker truck on its side with an unresponsive driver in the cab. Multiple bystanders are around the cab waving for you to come over and help the driver. Your immediate action should be to: A) Order the bystanders away from the truck B) Assess and extricate the patient C) Determine what material the truck is carrying D) Make certain the tanker is not leaking

A) Order the bystanders away from the truck

Which one of the following patients is the AEMT justified in restraining? A) Patient threatening to kill his wife and trying to leave the scene. B) Patient stating that he wants to kill himself and desires transport to the hospital. C) Patient with depression refusing transport. D) Patient with violent outbursts but at the present time is calm.

A) Patient threatening to kill his wife and trying to leave the scene.

Chemical agents that cause airway irritation, inflammation, and pulmonary edema include: A) Phosgene and chlorine B) Ammonia and carbon monoxide C) Acetylcholine and chlorine D) All of the above

A) Phosphogene and chlorine

A natural gas explosion has destroyed an apartment building. At least five occupants have been killed and another 12 injured. You are in charge of triage and are presented with a young woman who is breathing at a rate of 40 breaths per minute. Using the Simple Triage and Rapid Transport (START) system for triaging, you should immediately: A) Place a red tag on the patient and move on to the next patient B) Apply oxygen with a nonrebreather face mask C) Check for a radial pulse and capillary refill D) Start positive pressure ventilation

A) Place a red tag on the patient and move on to the next patient

A vehicle has been struck in its front end by another car. The driver is entrapped and the roof must be rolled back to allow extrication. The car has been appropriately stabilized and you are inside with the patient. The primary assessment is complete and a cervical collar applied. Rescue personnel are ready to begin cutting the vehicle. At this time, the appropriate action is to: A) Place a tarp over the patient B) Stabilize the vehicle C) Apply a short vest-type extrication device D) Remove the windshield

A) Place a tarp over the patient

As you are evaluating a pregnant patient who is supine on a couch, your partner informs you that the blood pressure is 78/50, pulse of 100, and respirations of 16. You should: A) Place her on her left side and retake vitals B) Give rapid IV fluid boluses and transport immediately C) Start chest compressions to keep BP above 80 D) Give dextrose 5 percent by IV titrated to 100 mmHg systolic

A) Place her on her left side and retake vitals

The MOST common biologic agents thought to be used in a terrorism incident are: A) Plague and anthrax B) Chlorine and phosgene C) Anthrax and phosgene D) Botulinum and gamma radiation

A) Plague and anthrax

A vehicle has rolled onto its passenger side after running a red light at an intersection and striking another car. The restrained driver remains in the car, screaming in pain. To manage this scene, which one of the following should you do FIRST? A) Stabilize the vehicle B) Hold manual inline stabilization C) Remove the roof D) Determine vital signs

A) Stabilize the vehicle

Pregnant patients are more susceptible to life-threatening injury because of: A) The position and increased vascularity of the uterus B) Decreased cardiac output associated with pregnancy C) Decreased vascular volume during pregnancy D) Higher rate of infections associated with pregnancy

A) The position and increased vascularity of the uterus

As your rescue unit arrives at the scene of a possible terrorist explosion, one of your primary concerns should be: A) The possibility of a secondary explosive device B) Rapid transport of as many patients as possible to the nearest hospital C) Searching for clues to the perpetrator D) Identifying the type of explosive device

A) The possibility of a secondary explosive device

What medication is indicated for the presence of alcohol intoxication to address possible encephalopathy? A) Thiamine B) Romazicon C) Valium D) Narcan

A) Thiamine

A poisonous substance NOT derived from the metabolism of an organism which is essentially a foreign substance is referred to as: A) Toxicant B) Toxicology C) Toxidrome D) Toxicodynamics

A) Toxicant

A 62-year-old woman with severe kyphosis has fallen and hit her head, which is oozing blood. Family members state that she suddenly complained of a "very bad headache" before collapsing to the kitchen floor. She has snoring respirations and appears cyanotic in the extremities. Of the following interventions, which one should you do FIRST? A) Use a jaw-thrust maneuver B) Apply a cervical collar C) Cover the laceration D) Start positive pressure ventilation

A) Use a jaw-thrust maneuver

If the red diamond on a National Fire Protection Association placard contains a "4," the EMT should recognize that the material contained within is: A) Very flammable B) Liquid C) Radioactive D) A gas

A) Very flammable

Your 40-year-old male patient was involved in an explosion at a local refinery. During transport, he develops increased respiratory problems. How would you ventilate this patient? A) With caution due to the mechanism of injury B) With demand valve, high-flow oxygen C) By hyperventilating with high-flow oxygen D) Vigorously with high-flow oxygen

A) With caution due to the mechanism of injury

You are using the Simple Triage and Rapid Transport (START) system of triage. Which one of the following patients should have a yellow tag applied prior to being moved to the treatment area? A) Woman with a respiratory rate of 22, palpable radial pulse, and squeezes your fingers when instructed B) Man with a respiratory rate of 40, palpable radial pulse, and squeezes your fingers when instructed C) Woman with agonal respirations, no radial pulse, and unresponsive mental status D) Woman with a respiratory rate of 8, no radial pulse, and unresponsive mental status

A) Woman with a respiratory rate of 22, palpable radial pulse, and squeezes your fingers when instructed

The National Incident Management System (NIMS) is beneficial in an emergency response to a disaster or terror attack because it provides for: A) a consistent approach to managing a disaster by many different responders and agencies. B) specific codes and terms to enhance communications between EMS, fire, and law enforcement personnel. C) increased funding and reimbursement for any EMS agency involved in a disaster response. D) federal government control of any disaster scene regardless of location.

A) a consistent approach to managing a disaster by many different responders and agencies.

While it is normal for MOST people to experience some mild depression such as that experienced when a loved one dies, if depression is prolonged and severe, the patient is said to be having: A) a major depressive episode. B) delirium. C) chronic anxiety. D) PTSD.

A) a major depressive episode.

Hyperthermia results in: A) an abnormally high core temperature. B) shivering to release heat. C) thermogenesis. D) vasoconstriction.

A) an abnormally high core temperature.

A patient who has suffered a serious head injury in the past and now exhibits symptoms related to this disorder is MOST likely suffering from a(n): A) cognitive disorder. B) anxiety disorder. C) substance abuse disorder. D) mood disorder.

A) cognitive disorder.

If the temperature regulating functions of the ________ are NOT working, the result is an abnormally high or low temperature. A) hypothalamus B) adrenal glands C) kidneys D) pancreas

A) hypothalamus

ou find an elderly patient who is unconscious and appears in poor condition. The patient was last seen three days ago. The house is hot as the area has been experiencing a heat wave with temperatures in excess of 100 degrees for the past two weeks. The patient is hot, dry, and breathing deeply and rapidly. Pulses are present and are full and bounding. Your immediate attention should be directed to: A) immediate cooling to reduce body core temp. B) gentle cooling to avoid hypothermia. C) insertion of a supraglottic airway D) administering saline to combat the electrolyte loss.

A) immediate cooling to reduce body core temp.

Entry of a substance through a break in the skin is known as: A) injection. B) inhalation. C) ingestion. D) surface absorption.

A) injection.

In the treatment of behavioral emergencies, ________ is/are important for the AEMT. A) interpersonal skills B) threepoint restraints C) pulse oximetry monitoring D) vital signs

A) interpersonal skills

When the core temperature is greater than that of the surrounding environment, the body will: A) lose heat (thermolysis). B) elevate the core temperature (thermogenesis). C) increase the basal metabolic rate. D) utilize the thermal gradient to exchange heat from the atmosphere to the body.

A) lose heat (thermolysis).

When managing a patient that has been exposed to a poisonous substance, the FIRST priority of the AEMT is to: A) perform a primary assessment. B) induce vomiting. C) administer activated charcoal. D) identify the poison.

A) perform a primary assessment.

A patient at a mountain ski resort presents to you with notable shortness of breath. She denies any past medical history and takes no medications. Your assessment reveals crackles to the bases of both lungs. After applying oxygen, you realize that the key to helping this patient improve is to: A) remove to a lower altitude. B) administer albuterol by a metered-dose inhaler. C) place her in a supine position with legs elevated. D) position her in a left lateral recumbent position.

A) remove to a lower altitude.

An organic brain disease that is characterized by a significant change in behavior and loss of contact with reality is: A) schizophrenia. B) bipolar disorder. C) depression. D) anxiety.

A) schizophrenia.

The fifth leading cause of death in persons of all age groups is: A) unintentional injury B) cardiovascular disease C) heart attack D) cancer

A) unintentional injury

If you are the first to arrive on scene and see signs of terrorism and that the scene is an MCI what should you​ do? A. Perform a windshield assessment from a safe distance. B. Immediately begin care of the most critically wounded. C. Stage far away until law enforcement arrive. D. Assume incident command while triaging all patients.

A.

The delayed response that responders may experience when subjected to traumatic events such as terrorist attacks and natural disasters is​ called: A. ​post-traumatic stress disorder B. acute emotional response C. an acute stress reaction D. nightmares

A.

Weapons that are designed to start fires using gasoline or other fuels are​ called: A. incendiary devices B. burn agents C. conventional explosives D. bombs

A.

What is the purpose of being trained with the National Disaster Medical​ System? A. to receive joint training with other teams to respond to disasters outside your jurisdiction B. to understand how to contact federal authorities during a disaster C. to receive training on how to care for patients during a hazardous materials incident D. to understand how other jurisdictions respond to mass casualty incidents

A.

Which of the following would be considered a natural​ disaster? A. A category F5 tornado destroys a rural town. B. An incendiary device is detonated at an abortion clinic. C. A nerve agent is released in a subway. D. Anthrax is mailed in envelopes to certain residences.

A.

Which type of weapon of mass destruction uses the toxic properties of chemicals to cause illness or​ injury? A. chemical weapons B. radiologic dispersion devices C. incendiary devices D. biologic weapons

A.

While on scene of a terrorist attack involving a nerve​ agent, you and your partner have taken necessary precautions to protect against exposure. As you are assessing your patients another responder begins experiencing SLUDGE symptoms. What medication do you anticipate that the responder will​ receive? A. atropine and pralidoxime B. pralidoxime and lidocaine C. naloxone and pralidoxime D. atropine and naloxone

A.

You are on the scene of a small neighborhood that has been destroyed by a tornado. You are the only transport unit on scene and there are approximately 10 patients. What should you​ do? A. Contact dispatch and request more transport units. B. Utilize other emergency vehicles on scene as transport units. C. Inform the triage team to send you only the least critical patients. D. Inform the triage team to send you only the most critical patients.

A.

Entry into the scene of a nuclear incident should​ be: A. upwind and laterally to the upper air movement B. delayed for an hour to allow the fallout to dissipate C. made in full turnout gear for protection and a SCBA D. delayed until a dosimeter shows the radiation has dissipated

A. Entry into the scene is made from upwind and laterally to upper air movement in order to limit radioactive fallout exposure to rescuers.

A patient has cut his arm with a table saw. Assessment reveals dark red blood flowing steadily from the wound. As an AEMT, you should recognize this as what type of bleeding? A) Venous B) Lymphatic C) Capillary D) Arterial

Answer: - Venous

Which one of the following statements made by an AEMT shows an understanding of chest trauma? A) "Closed chest injuries are caused by blunt trauma, and can be just as serious as open chest injuries." B) "Open chest injuries are more serious than closed chest injuries, because air and bacteria can enter the chest through the open wound." C) "In general, an open chest injury caused by blunt trauma is more life-threatening than a closed one due increased blood loss." D) "Closed chest injuries are caused by penetrating trauma and cause more life-threatening injuries than do open chest injuries."

Answer: A - "Closed chest injuries are caused by blunt trauma, and can be just as serious as open chest injuries."

Assessment of a72-year-old man reveals absent lung sounds to the left lung accompanied by sharp chest pain on the same side. When obtaining the history, which of the following statements made by his wife is MOST important? A) "He has chronic obstructive pulmonary disease." B) "He has had a fever since last night." C) "He did not take his medications today." D) "He just saw his doctor yesterday."

Answer: A - "He has chronic obstructive pulmonary disease."

A two-year-old boy fell down a flight of stairs. Which instruction would you provide to other AEMTs who are immobilizing the patient? A) "Let us place a folded towel under his shoulders to help maintain head alignment." B) "It is better if the collar is a little too big; that is more comfortable for him." C) "After he is on the board, place a towel behind his head to keep the airway open." D) "Just apply a cervical collar and then place him directly on the stretcher."

Answer: A - "Let us place a folded towel under his shoulders to help maintain head alignment."

Which one of the following statements about the care and treatment of a patient with a spinal injury in the prehospital setting is TRUE? A) "Prehospital care for the patient with a spine injury involves immobilization and the identification of life-threatening conditions." B) "If a patient has a possible spine injury but is in shock, it is permissible to forego immobilization since this takes time from definitive care in the hospital." C) "Before transporting the critically injured patient with a spinal injury, the EMT must perform a detailed neurological assessment from head to toe." D) "It is important to identify the site of spinal injury so the proper prehospital care can be rendered."

Answer: A - "Prehospital care for the patient with a spine injury involves immobilization and the identification of life-threatening conditions."

Which one of the following statements shows that an AEMT understands field care of a patient with a possible joint dislocation? A) "The care for a patient with a joint injury is identical to that of a fracture." B) "If a distal pulse is absent, three attempts to straighten the joint can be made." C) "If a joint injury is suspected, ice packs and not splints are indicated." D) "Warm packs, not cold packs, are indicated for a patient with a possible joint injury."

Answer: A - "The care for a patient with a joint injury is identical to that of a fracture."

The customary sublingual dosage of nitroglycerin is: A) 0.4 mg B) 1.5 mg C) 0.04 mg D) 1.0 mg

Answer: A - 0.4 mg

An unrestrained male struck the steering wheel with his abdomen. Currently, he is complaining of generalized abdominal pain and rates it a 3/10. Assessment indicates red marks across his lower abdomen caused by the steering wheel. His pulse is 128, respirations 18, blood pressure 128/92, and SpO2 at 98 percent. Which one of the following is MOST appropriate when providing oxygen to this patient? A) 15 liters per minute through nonrebreather B) Oxygen can be withheld due to the SpO2 reading C) 2 liters per minute through a nasal cannula D) 8 liters per minute through a simple face mask

Answer: A - 15 liters per minute through nonrebreather

Which one of the following indicates the CORRECT documentation of a pulse oximetry reading on the prehospital care report? A) 97percent SpO2 B) 97percent O2 C) 97percent PO2 D) 97percent PO

Answer: A - 97percent SpO2

Which of the following would be considered an opportunistic infection? A) A cancer patient contracts pneumonia after completing radiation treatments. B) A young child comes down with chicken pox after exposure at school. C) A patient get a skin infection while healing from a full thickness burn. D) An unimmunized carpenter contracts tetanus after stepping on a rusty nail.

Answer: A - A cancer patient contracts pneumonia after completing radiation treatments.

Which of the following fits the definition for atelectasis? A) A state in which the alveoli are collapsed B) An involuntary contraction of the muscles of the chest C) A condition in which ventilation exceeds metabolic needs D) Non-cardiogenic pulmonary edema

Answer: A - A state in which the alveoli are collapsed

An injury that occurs as a result of bleeding intracranially is: A) A subdural hematoma. B) A spinal cord transaction. C) A comminuted fracture. D) Hypovolemia.

Answer: A - A subdural hematoma

Which of the following is NOT a common sign or symptom of stroke? A) Acute chest pain B) Facial numbness C) Visual disturbances. D) Loss of coordination.

Answer: A - Acute chest pain

A construction foreman has been stabbed in the right anterior chest with a screwdriver by an angry worker. He is conscious and oriented and complaining of pain to the injury site. Prior to your arrival, he removed the screwdriver. Your primary assessment reveals a patent airway, adequate breathing, and strong radial pulse. Based on these findings, you should: A) Administer oxygen through a nonrebreather mask B) Take manual inline spinal stabilization C) Insert a nasopharyngeal airway D) Open the airway using the jaw-thrust maneuver.

Answer: A - Administer oxygen through a nonrebreather mask

Which of the following is CORRECT regarding asthma? A) After initial treatment for an attack, a secondary reaction may occur hours later. B) If treated by beta2-agonists, the underlying inflammation is eliminated. C) Beta2-agonists act to reduce mucus production in the lungs. D) When an asthmatic stops wheezing, the medication if working well.

Answer: A - After initial treatment for an attack, a secondary reaction may occur hours later.

When documenting fetal development in a location other than the uterus, you would identify it as: A) An ectopic pregnancy. B) Placenta previa. C) Pyelonephritis. D) An ovarian cyst.

Answer: A - An ectopic pregnancy.

With the exception of the ________, MOST abdominal organs are NOT able to sense tearing sensations. A) Aorta B) Ovaries C) Liver D) Colon

Answer: A - Aorta

You are called to a 25-year-old man complaining of RLQ pain. His other symptoms are nausea and vomiting, fever, and decreasing pain in the umbilicus area. You should suspect: A) Appendicitis. B) Pancreatitis. C) Cholecystitis. D) Peritonitis.

Answer: A - Appendicitis.

A patient with a suspected injury to the liver should be treated: A) As any patient in shock B) With oxygen by cannula and positioning comfortably C) Conservatively as the liver is capable of self healing D) In the field with hypotonic fluid administration

Answer: A - As any patient in shock

Which one of the following medications should the AEMT administer orally? A) Aspirin B) Epinephrine C) Albuterol D) Nitroglycerin

Answer: A - Aspirin

The presence of insulin in the bloodstream will: A) Assist glucose to enter the cells. B) Allow for the production of glucose. C) Stimulate the CNS activity. D) Inhibit the use of glucose by the cells.

Answer: A - Assist glucose to enter the cells.

The ongoing assessment in the critically injured trauma patient should be done: A) At least every five minutes. B) Every 15 minutes. C) Every minute. D) Once on the way to the hospital.

Answer: A - At least every five minutes

Which of the following statements regarding meningitis is TRUE? A) Bacterial meningitis is more serious and can be fatal. B) Viral meningitis occurs more often during the winter months. C) The incidence of childhood meningitis has increased recently. D) Viral meningitis is spread through contact with nasal and oral secretions.

Answer: A - Bacterial meningitis is more serious and can be fatal.

Despite the visual appearance of soft-tissue injuries like burns, it is important that the AEMT: A) Be aware of other potential lifethreats B) Realize that blood loss with soft-tissue injuries is severe and may lead to shock C) Cover soft-tissue injuries with sterile gauze and a secure dressing D) Assess distal circulation from a soft-tissue injury

Answer: A - Be aware of other potential lifethreats

Which one of the following medical emergencies would MOST likely result in the use of epinephrine by the AEMT? A) Bee sting B) Cardiac arrest C) Slow heart rate D) Asthmatic episode

Answer: A - Bee sting

Which one of the following is the location of the popliteal pulse? A) Behind the knee B) Top of foot C) Posterior ankle D) Upper arm

Answer: A - Behind the knee

A sign of basal skull fracture in the trauma patient would be the presence of: A) Bilateral periorbital ecchymosis. B) Tachycardia. C) Confusion. D) Hypotension.

Answer: A - Bilateral periorbital ecchymosis

Which of the following are common complications of diabetes? A) Blindness, peripheral neuropathy, and kidney failure B) Autoimmune disease, adrenal disease, and sepsis. C) Myxedema, cardiac disease, stroke. D) Gangrene, hemorrhagic stroke, high cholesterol.

Answer: A - Blindness, peripheral neuropathy, and kidney failure

You can differentiate a partial-thickness burn from a superficial burn by the presence of: A) Blisters. B) Pain. C) Inflammation. D) Redness.

Answer: A - Blisters

You have documented that you patient has hematochezia. Which of the following BEST describes this problem? A) Bloody stools B) Vomiting blood C) Coughing up blood D) Occult blood

Answer: A - Bloody stools

An injury that results from the transmission of energy of an object rather than the object itself BEST describes: A) Blunt trauma B) Penetrating trauma C) Trauma D) Newton's first law

Answer: A - Blunt trauma

When obtaining a blood pressure, the AEMT listens for a pulse over what blood vessel? A) Brachial artery B) Radial artery C) Carotid vessels D) Antecubital vein

Answer: A - Brachial artery

You determine a patient's heart rate to be 48 beats per minute. That heart rate would be categorized as: A) Bradycardic B) Normal C) Tachypneic D) Tachycardic

Answer: A - Bradycardic

A 68-year-old female patient is complaining of a headache and generalized weakness. Her husband informs you that she was with a friend yesterday and was involved in a motor-vehicle collision. She did strike her face on the dashboard, but refused treatment on scene. Her past medical history includes a stroke with right arm weakness and high blood pressure for which she takes medications. When assessing this patient, which one of the following findings should concern the AEMT MOST? A) Bruising behind her left ear B) Complaint of pain when she moves her jaw C) Contusion to her left cheek area D) Weakness to the right-hand grip

Answer: A - Bruising behind her left ear

The amount of blood pumped by the heart in one minute is referred to as: A) Cardiac output B) Blood pressure C) Preload D) Stroke volume

Answer: A - Cardiac output

Which of the following BEST describes the effects of a pericardial tamponade on cardiac output and venous pressure? A) Cardiac output is low, and central venous pressure rises B) Cardiac output rises, and venous pressure is not affected C) Cardiac output is not affected, and central venous pressure rises D) Cardiac output is high, and central venous pressure falls

Answer: A - Cardiac output is low, and central venous pressure rises

A construction worker was accidentally shot with a nail gun. The nail penetrated the skull approximately one inch through the temporal region. Before your arrival, coworkers pulled the nail free. Based on the mechanism of injury, which one of the following signs or symptoms would provide the BEST evidence that the meningeal layers have been broken? A) Cerebral spinal fluid coming from the puncture wound B) Crepitus of the skull at the injury site C) Swelling and deformity to the tissues at the injury site D) Blood coming from the puncture wound

Answer: A - Cerebral spinal fluid coming from the puncture wound

As you are examining a victim involved in a motor vehicle collision, you note the presence of a clear fluid draining from the nose. You suspect this is: A) Cerebrospinal fluid. B) Excessive mucous. C) Potential airway obstruction. D) Pleural fluid.

Answer: A - Cerebrospinal fluid

As you approach a motorcyclist who was thrown from his bike, you can hear him saying that he cannot feel or move his legs. You note that he has obvious deformity to both femurs. EMRs are holding manual inline spinal stabilization. Of the following, which one should be done FIRST? A) Check his rate and effort of breathing B) Place a cervical collar and administer oxygen C) Check for motor function and sensation in the feet D) Expose the legs and look for bleeding

Answer: A - Check his rate and effort of breathing

A patient has been assaulted and sustained blunt trauma to the abdomen. As you start the primary assessment, you note that he is vomiting blood and you begin to suction him immediately. After the airway has been suctioned, you should: A) Check the patient's respirations B) Obtain a full set of vital signs C) Examine the abdomen for specific injuries D) Administer oxygen with a nonrebreather face mask.

Answer: A - Check the patient's respirations

What term describes the pain usually caused by an occlusion of the gallbladder duct? A) Cholecystitis B) Appendicitis C) Kidney stone D) Pancreatitis

Answer: A - Cholecystitis

You respond to a motor vehicle collision in which a 22-year-old woman has suffered several cuts. You determine that the airway is patent and the patient is breathing adequately. The patient's blood pressure is maintaining at a stable level, and you estimate that she has lost approximately 10 percent of her circulating blood volume. She is alert and oriented but seems nervous. From these findings, you would determine that your patient is in ________ hemorrhage. A) Class I B) Class II C) Class III D) Class IV

Answer: A - Class I

Which of the following is the major cause of pain for the sickle cell patient? A) Clumps of sickle cells obstruct capillary beds. B) Blood pools in the spleen causing pain. C) Hemolysis of the cells leading to gallstones. D) Blood clots in the lungs.

Answer: A - Clumps of sickle cells obstruct capillary beds.

When the skin comes into contact with acids, it forms a thick, insoluble mass at the point of contact. This process is called: A) Coagulation necrosis B) Liquefaction necrosis C) The hypermetabolic phase D) The emergent phase

Answer: A - Coagulation necrosis

An IV fluid solution that contains proteins or large starch molecules would be Referenceerred to as a(n): A) Colloid B) Analgesic C) Crystalloid D) Collusion

Answer: A - Colloid

The stage of shock in which the body is still able to meet its critical metabolic needs through a progressive series of actions is called: A) Compensated shock B) Irreversible shock C) Decompensated shock D) Septic shock

Answer: A - Compensated

An example of a diffuse brain injury that has a global injury pattern is a: A) Concussion. B) Epidural hematoma. C) Subdural hematoma. D) All of the above

Answer: A - Concussion

You are assessing a patient who was burned when she dropped a hot iron on her bare foot. Her foot is red and has a small partial-thickness burn near the ankle. When completing the prehospital care report, what cause should you document? A) Contact burn B) Electrical burn C) Scald burn D) Flame burn

Answer: A - Contact burn

In the mnemonic DCAP-BTLS, "C" stands for: A) Contusions B) Cuts C) Concussions D) Crepitus

Answer: A - Contusions

You are dispatched to a beach where a patient has been sunburned. Upon arrival, you find a 23-year-old woman who was sunbathing and fell asleep. She is alert and oriented, in moderate pain, and has blisters covering all four extremities. She also has blisters on her abdomen, face, and chest. You should consider this patient to have ________ burns. A) Critical B) Moderate C) Superficial D) Minor

Answer: A - Critical

Assessment of a patient's skin color reveals a bluish discoloration. As a knowledgeable AEMT, you should identify: A) Cyanosis caused by inadequate oxygenation of the tissues B) Pallor caused by a decrease in blood flow to the tissues C) Pallor caused by a decrease in body temperature D) Jaundice caused by an increase in blood flow to the extremities

Answer: A - Cyanosis caused by inadequate oxygenation of the tissues

Which IV fluid solution is a hypotonic solution of sterile water containing 5 percent dextrose by volume and is useful in patients who are prone to fluid overload? A) D5W B) Normal saline C) D10W D) Lactated Ringer's

Answer: A - D5W

Your patient is a young man who was the unrestrained driver of a car that struck a telephone pole. He is unconscious, has no external bleeding, and is breathing erratically. His blood pressure is 76/50, pulse rate present only at his carotid is 130 and weak. What stage of shock is the patient in? A) Decompensated shock B) Hypovolemic shock C) Compensated shock D) Irreversible shock

Answer: A - Decompensated shock

You have been called to a New Year's party for a male patient who was stabbed in the anterior chest. According to partygoers, the patient went outside with another male and was stabbed. Fifteen minutes later, he was found in the snow and 911 was called. When assessing this patient, which finding should the AEMT consider MOST serious and warrant immediate attention? A) Decreased breath sounds to the left lung B) Capillary refill of three seconds C) Complaint of a worsening headache D) Pain and bruising to the left thigh

Answer: A - Decreased breath sounds to the left lung

A patient fell and sustained an open fracture to the left humerus. However, assessment reveals the bone to have pulled back into the arm. Bleeding from the site is controlled. How will splinting the left arm benefit this patient? A) Decreases the opportunity for further injury to nerves and blood vessels B) Converts the open fracture to a closed fracture and maintains it as such C) Eliminates the possibility of infection through the open wound D) Starts the process of healing by aligning and connecting the bone ends

Answer: A - Decreases the opportunity for further injury to nerves and blood vessels

You are documenting that your patient has pain when he urinates. Which of the following terms will you use to correctly identify this problem? A) Dysuria B) Uremia C) Uritis D) Hematuria

Answer: A - Dysuria

Asking a patient, "What brought this on?" represents what part of the SAMPLE question format? A) E B) L C) M D) P

Answer: A - E

Which of the following is usually the FIRST sign of extravasation at an IV site? A) Edema at the IV site B) Bleeding at the IV site C) Warmth of the IV site D) Redness of the IV site

Answer: A - Edema at the IV site

The mechanism of injury that suggests transport to a trauma center is: A) Ejection from a vehicle B) Vehicle impact at a speed greater than 20 mph C) Extrication time of less than 20 minutes D) A fall from a distance twice the patient's height

Answer: A - Ejection from a vehicle

A teenage female patient has been ejected from a car as it rolled down a hill. She is responsive, but with signs of early shock. She has bruising to her anterior abdomen, a laceration to her left temple, and deformity to he left wrist. At what point should the wrist injury be splinted? A) En route to the hospital B) Before transferring to the stretcher C) Just before leaving the scene D) Prior to immobilizing the patient

Answer: A - En route to the hospital

When a patient suffers penetrating trauma to the chest, as the diaphragm moves downward, air will: A) Enter the pleural cavity from outside B) Not move into the chest cavity C) Be forced from the chest cavity D) Accumulate in the lungs

Answer: A - Enter the pleural cavity from outside

Your patient is a 64-year-old man with a recurrent case of herpes. He has called 911 because he has raised, red lesions scattered around his stomach and back. His medical history includes taking medications for gout. Which of the following should you suspect? A) Erythema multiforme B) Stevens-Johnson syndrome C) Toxic epidermal necrolysis D) Cellulitis

Answer: A - Erythema multiforme

You have arrived on the scene of a stabbing. As you approach the patient, you note that he has removed his shirt and has a single stab wound to the right lower quadrant of the abdomen. At the patient's side, your FIRST action should be to: A) Examine the patient's airway B) Quickly look for additional stab wounds C) Provide oxygen via nonrebreather face mask D) Cover the wound with a nonporous dressing

Answer: A - Examine the patient's airway

If the blood sugar level is very high, which of the following may result? A) Excessive urination, excessive thirst, and excessive hunger B) Polyuria, polydipsia, polyphagia, and hyperactivity C) Excessive insulin, excessive glucose, and excessive urination D) Hyperactivity, excessive thirst, and polyuria

Answer: A - Excessive urination, excessive thirst, and excessive hunger

When cells are unable to utilize glucose for fuel, they will slowly begin to use ________ as an energy source. A) Fats B) Insulin C) Pancreatic enzymes D) Carbohydrates

Answer: A - Fats

During coagulation a circulating substance creates a meshwork to reinforce the blood clot and stabilize it. What is this substance called? A) Fibrin B) Platelets C) Antibodies D) Rh antigen

Answer: A - Fibrin

When using a pulse oximeter on an adult patient, the AEMT knows that the sensor is MOST often attached to the patient's: A) Finger B) Chest C) Ear D) Forehead

Answer: A - Finger

A young girl is complaining of eye pain after having sand thrown in her face by another girl at the park. When assessing her eyes, you note large particles of sand in the right eye. The sand appears to be on the conjunctiva, which also appears to be scratched and irritated. Your initial action would be to: A) flush the eye with clean water from a nearby sink. B) gently wipe the inner eye with sterile pieces of gauze. C) stop the tearing by applying gentle pressure to the eye. D) attempt to remove the larger pieces of sand using a cotton swab.

Answer: A - Flush the eye with clean water from a nearby sink

________ caregivers should be present when logrolling a patient to a spine board. A) Four B) Two C) One D) Six

Answer: A - Four

As you arrive at a metal scrap yard for an unknown medical emergency, you observe a male patient supine on the ground with AED electrodes on his chest. The AED operator has just ordered coworkers to clear the patient because the AED is going to shock. Within seconds, the AED delivers a shock without the operator pressing a "shock" button. The AEMT should recognize what type of AED? A) Fully automated B) Manual C) Biphasic D) Semi-automated

Answer: A - Fully automated

The condition that presents with inflammation of the stomach and small intestines commonly associated with abdominopelvic pain is known as: A) Gastroenteritis. B) Peritonitis. C) Pancreatitis. D) Diverticulitis.

Answer: A - Gastroenteritis.

Which of the following is the BEST way to differentiate between non-life-threatening and life-threatening causes of headache? A) Gather a good history using OPQRST. B) Find out if trauma was involved. C) Ask if the patient has allergies. D) Evaluate the blood glucose level.

Answer: A - Gather a good history using OPQRST.

The FIRST step for treating a patient who has been contaminated with dry lime is to: A) Gently brush off the powder. B) Flush with water. C) Insert a supraglottic airway in the patient. D) Neutralize the lime.

Answer: A - Gently brush off the powder

Which of the following disorders has the characteristic exophthalmos in which swelling of the tissues behind the eyes causes the eyes to protrude? A) Graves' disease B) Cushing's syndrome C) Addison disease D) Diabetic ketoacidosis

Answer: A - Graves' disease

Ecchymosis around the flanks indicative of retroperitineal bleeding is referred to as: A) GreyTurner's sign B) Cullen's sign C) Kehr's sign D) Murphy's sign

Answer: A - GreyTurner's sign

Your patient is complaining of chest pain and is cool, pale, and diaphoretic with a bluish tinge on his nail beds. You place a pulse oximeter probe on his index finger and it shows a reading of 88percent. Based on this information, what will you document in your patient care report when describing his condition? A) He is hypoxic with signs of shock. B) He has chest pain with dyspnea. C) He is sweaty with signs of pneumonia. D) He needs to take his nitroglycerin.

Answer: A - He is hypoxic with signs of shock.

When securing a victim to a 6' spine board, the AEMT should use: A) Head immobilization device B) Sandbags and duct tape C) 2" cloth tape and towel rolls D) One liter IV bags and duct tape

Answer: A - Head immobilization device

The clinical syndrome in which the heart's mechanical performance is compromised so that cardiac output cannot meet the body's needs is called: A) Heart failure. B) Angina pectoris. C) Cardiac tamponade. D) Pneumothorax.

Answer: A - Heart failure.

Which of the following illnesses are spread through the gastrointestinal tract? A) Hepatitis A and E B) Hepatitis B and D C) Hepatitis C and D D) Hepatitis A and C

Answer: A - Hepatitis A and E

As a result of increasing pressure within the cranial cavity, ________ may occur. A) Herniation of brain tissue B) Cerebral concussion C) Hypotension D) All of the above

Answer: A - Herniation of brain tissue

You respond to a call of an explosion at a local chemical company. Upon arrival, the company's hazmat team is decontaminating patients so you can easily treat them. The team extricates a 35-year-old man who is experiencing slight respiratory distress. The care that you should provide FIRST for this patient is: A) High-flow oxygen B) Insertion of a supraglottic airway C) Transport to the local trauma center D) An immediate head-to-toe trauma assessment.

Answer: A - High-flow oxygen

A family has called you for a 41-year-old man they cannot wake up. They state he has been threatening to kill himself and believe that he may have intentionally overdosed on several medications. As you enter the patient's bedroom, you observe him supine on the floor with his eyes closed. As he breathes, you hear snoring respirations. As a knowledgeable AEMT, you recognize which one of the following? A) His tongue is partially blocking the airway B) His respiratory rate must be less than 10 per minute C) He is sleeping and has not overdosed D) He will require immediate suctioning

Answer: A - His tongue is partially blocking the airway

When performing a venipuncture on a patient what is the correct way to hold the IV needle? A) Hold the IV needle at a 45-degree angle or less to the skin with the bevel up B) Hold the IV needle at a 90-degree angle to the skin with the bevel up C) Hold the IV needle at a 45-degree angle or greater to the skin with the bevel down D) Hold the IV needle at a 45-degree angle or less to the skin with the bevel down

Answer: A - Hold the IV needle at a 45-degree angle or less to the skin with the bevel up

The homeostatic function of the endocrine system is regulated by: A) Hormonal release. B) Baroreceptors. C) Nervous system stimulation. D) Voluntary control.

Answer: A - Hormonal release.

The major categories of shock include: A) Hypovolemic, cardiogenic, obstructive, and distributive B) Hypoglycemic, obstructive, distributive, and hypovolemic C) Hemorrhagic, distributive, hypoxic, and obstructive D) Burn, hypovolemic, distributive, and hypoxic

Answer: A - Hypovolemic, cardiogenic, obstructive, and distributive

A 34-year-old man has overdosed on an unknown drug and is unresponsive. His is breathing shallowly at six times per minute. He has cyanosis around his lips and nail beds and he is cool to the touch. His breath sounds are diminished but clear. You should recognize which of the following conditions? A) Hypoxia B) Tachypnea C) Apnea D) Dyspnea

Answer: A - Hypoxia

Management of life-threatening injuries to the pelvic ring MUST include: A) Immobilization of the fracture, hemodynamic support, rapid transport to a trauma center. B) Application of the PASG, large-bore vascular access with two 1000 cc bags of NS or LR, aggressive fluid therapy to maintain systolic blood pressure above 90-100 mmHg, immediate transport to trauma facility. C) Patient immobilization, aggressive fluid therapy to maintain systolic blood pressure above 90 mmHg, transport to trauma center. D) Immobilization of the cervical spine, application of the PASG, large-bore vascular access with appropriate fluid resuscitation to maintain systolic blood pressure above 90-100 mmHg, immediate transport to the trauma center.

Answer: A - Immobilization of the fracture, hemodynamic support, rapid transport to a trauma center.

Which of the following will impair cardiac output? A) Improperly functioning valves B) Increased perfusion to the heart C) Elevated blood pressure D) Increase in mean arterial pressure

Answer: A - Improperly functioning valves

If you are attempting hip reduction, which of the following points MUST you remember? A) In the event that the reduction attempt is unsuccessful, immobilize in the position found and consider rapid transport B) Prehospital hip reduction is never indicated C) In the event of diminished distal circulation, do not stop the reduction attempt until you note femoral head movement D) In the event that the reduction attempt is successful, transport the patient in the fetal position to prevent further dislocation

Answer: A - In the event that the reduction attempt is unsuccessful, immobilize in the position found and consider rapid transport

Which of the following situations would NOT result in anemia? A) Increased production of erythrocytes B) Iron deficiency C) Blood loss D) Destruction of red blood cells

Answer: A - Increased production of erythrocytes

Aspirin is used in the treatment of myocardial ischemia because it: A) Inhibits the aggregation of platelets. B) Is a thrombolytic. C) Has analgesic effects. D) Can cause gastric upset and bleeding.

Answer: A - Inhibits the aggregation of platelets.

Your patient is a 24-year-old woman who was involved in an MVC and struck the steering wheel with her chest. Upon your arrival, she is alert, but restless. Her pulse is rapid and weak, with a respiratory rate of 26 per minute, and blood pressure is 130/64. En route to the hospital, your patient becomes unresponsive, with a heart rate of 40 and blood pressure of 60/30. At this point, in what stage of shock is the patient? A) Irreversible shock B) Compensated shock C) Decompensated shock D) Neurogenic shock

Answer: A - Irreversible shock

Blood loss from a scalp wound: A) Is severe enough to cause shock. B) Requires vasopressor agents to control. C) Is hard to control with pressure. D) Is usually only minor.

Answer: A - Is severe enough to cause shock

Solutions that have a tonicity equal to blood plasma are called: A) Isotonic solutions B) Colloidal solutions C) Hypotonic solutions D) Hypertonic solutions

Answer: A - Isotonic solutions

Which of the following statements regarding angina pectoris is TRUE? A) It can be brought on by exertion or stress. B) It is generally relieved by over-the-counter medications. C) It results in death of myocardial cells. D) It generally lasts 30 to 60 minutes.

Answer: A - It can be brought on by exertion or stress.

Your patient has severe respiratory distress from an acute onset of pulmonary edema. Why do you want to treat her with CPAP? A) It can reduce the amount of fluid that can cross into the alveoli. B) It increases the oxygen concentration of the inspired oxygen. C) It reduces the inflammation of the lower airways. D) It has an analgesic to help reduce the patient's anxiety.

Answer: A - It can reduce the amount of fluid that can cross into the alveoli.

Which of the following is the beneficial action of nitroglycerin in some cardiac emergencies? A) It dilates blood vessels throughout the body. B) It dilates only the coronary arteries. C) It reduces the strength of ventricular contractions. D) It slows down the heart rate.

Answer: A - It dilates blood vessels throughout the body.

You are explaining to your EMT partner how he can differentiate tension pneumothorax from a simple pneumothorax. Which of the following would you tell him indicates tension pneumothorax? A) Jugular venous distension B) Mild to severe shortness of breath C) Decreased lung sounds on affected side D) Tachycardia related to anxiety

Answer: A - Jugular venous distension

Which of the following can cause particular difficulties when spinal immobilizing an elderly patient? A) Kyphosis B) Cellulitis C) Sciatica D) Fibromyalgia

Answer: A - Kyphosis

Asking a patient, "Have you eaten today?" represents what part of the SAMPLE question format? A) L B) P C) A D) M

Answer: A - L

Which of the following IV fluid solutions may NOT be administered with blood products due to potential harmful reactions? A) Lactated Ringer's and D5W B) Normal saline and D5W C) Lactated Ringer's and normal saline D) Normal saline and one-half normal saline

Answer: A - Lactated Ringer's and D5W

Which of the following heart chambers pumps oxygenated blood to the body? A) Left ventricle B) Left atrium C) Right ventricle D) Right atrium

Answer: A - Left ventricle

Which of the following is NOT associated with the retina? A) Lens B) Photopigments C) Rods D) Cones

Answer: A - Lens

The AEMT should gain a general impression of the trauma victim based on: A) Level of responsiveness and ABC evaluation B) Bystander reaction C) MOI and hazard identification D) Dispatch information

Answer: A - Level of responsiveness and ABC evaluation

A patient has been stabbed in the right upper quadrant of the abdomen. Which one of the following organs or structures should the AEMT be concerned about being injured? A) Liver B) Spleen C) Pancreas D) Small intestine

Answer: A - Liver

Glucagon may be ineffective in patients with inadequate: A) Liver glycogen stores B) Renal perfusion pressures C) Oxygen tissue perfusion D) Blood glucose level

Answer: A - Liver glycogen stores

Prior to your arrival on the scene of a motorcycle crash, an EMR contacts you via radio and reports that there is one patient who has an avulsion to her left arm. As a knowledgeable AEMT, you should anticipate which one of the following? A) Loose flap of skin torn on her left arm B) Long and deep laceration with moderate bleeding C) Bruised skin with a portion of bone protruding D) Torn skin with an underlying fracture

Answer: A - Loose flap of skin torn on her left arm

The prehospital stroke scoring system that assesses blood glucose levels, facial droop, grip strength, and arm drift is the: A) Los Angeles Prehospital Stroke Screen. B) New York Prehospital Stroke Screen. C) Cincinnati Prehospital Stroke Scale. D) North American Prehospital Stroke Scale.

Answer: A - Los Angeles Prehospital Stroke Screen

Your 17-year-old patient is presenting with fatigue, fever, chills, headache, muscle and joint pain. Further history reveals a recent camping trip to the mountains. You note a rash on his back that he had missed. You should suspect: A) Lyme disease. B) Tularemia. C) Rocky Mountain spotted fever. D) Rabies.

Answer: A - Lyme disease.

Which of the following statements about immunity is TRUE? A) Lymphocytes are the primary cells involved in the immune response. B) Mast cells are eosinophils that migrate into the tissues. C) Most specific antibodies are present are birth. D) Cell-mediated immunity uses antibody formation to combat pathogens.

Answer: A - Lymphocytes are the primary cells involved in the immune response.

The ______ migrate into the tissues where they release histamine and heparin. A) Mast cells B) Macrophages C) Thrombocytes D) White blood cells

Answer: A - Mast cells

Glucose is measure in: A) Milligrams per deciliter (100 mL) of blood B) Milliliters per liter (1000 mL) of blood C) Grams per deciliter (100 mL) of blood D) Grams per liter (1000 mL) of blood

Answer: A - Milligrams per decileter (100 mL) of blood

The baroreceptors that recognize hypotension due to blood loss and signal the medulla oblongata are located in the: A) Neck B) Vena cava C) Brain D) Heart

Answer: A - Neck

The patient with type 2 diabetes is usually: A) Non-insulin dependent. B) Glucose intolerant. C) Glucose tolerant. D) Insulin dependent.

Answer: A - Non-insulin dependent.

If your patient has type O+ blood, what other blood type(s) can she receive? A) O- B) AB- C) AB- and O- D) AB+

Answer: A - O-

An industrial worker has sustained a laceration to his abdomen. On closer inspection, you note what appear to be fat tissue and a portion of intestine protruding through the wound. Which one of the following is MOST appropriate for treating this injury? A) Occlusive dressing applied over saline soaked gauze B) Dry sterile dressing applied over the wound C) Direct pressure using gloved palms to the injury D) One attempt at replacement using sterile gloves

Answer: A - Occlusive dressing applied over saline soaked gauze

AEMT care for an open chest wound and an abdominal evisceration are similar in that: A) Occlusive dressings are placed over both wounds B) Moist dressings are placed over both wounds C) Air must be periodically released from both wounds D) Both wounds can cause the trapping of air in the body

Answer: A - Occlusive dressings are placed over both wounds

You have been called by family members for their mother who is "not acting right." On-scene the family informs you that they are concerned because their 68-year-old mother has been complaining of a headache and today is very confused. Assessment reveals a bruise to the right side of the head that family states occurred one week ago when she fell in church. Since all other aspects of the assessment are normal, you suspect a head injury. Which condition would make the MOST sense given these findings and history? A) Occult subdural hematoma B) Cerebral contusion C) Scalp laceration D) Concussion

Answer: A - Occult subdural hematoma

You have administered Xopenex to a patient, using a small-volume nebulizer. You then notify medical direction that you have administered the drug. This is an example of which type of medical direction? A) Off-line B) Retrospective C) On-line D) Direct

Answer: A - Off-line

You have arrived on the scene for a patient assaulted with a baseball bat. As you approach, which one of the following positions seems to indicate that he was struck and suffered injury to the abdomen? A) On his side with knees drawn to the chest B) Supine with legs extended and toes pointed C) Prone with hands on the back D) Standing with restless movement and pacing

Answer: A - On his side with knees drawn to the chest

The patient with NKDC is at risk for dehydration due to: A) Osmotic diuresis. B) Vasodilation. C) Sepsis. D) Hemorrhage.

Answer: A - Osmotic diuresis.

A degenerative joint disease in which the articular cartilage is damaged and breaks down is known as: A) Osteoarthritis. B) Osteoporosis. C) Arthritis. D) Bursitis.

Answer: A - Osteoarthritis.

A patient with acute peritonitis may present with rapid, shallow breathing resulting from: A) Pain. B) DKA. C) Hypovolemia. D) Inflammation.

Answer: A - Pain.

GreyTurner's sign is indicative of acute: A) Pancreatitis. B) Kidney stones. C) Cholecystitis. D) Appendicitis.

Answer: A - Pancreatitis.

Your patient was involved in a serious motor-vehicle collision. Which one of the following assessment findings BEST helps to determine that the patient has a flail segment? A) Paradoxical chest wall movement B) Pain with inspiration C) SpO2 reading lower than 90 percent D) Shortness of breath

Answer: A - Paradoxical chest wall movement

A route of drug administration is a way in which the drug gains access to the body. Routes that bypass the gastrointestinal tract are called: A) Parenteral B) Percutaneous C) Enteral D) Aseptic

Answer: A - Parenteral

A patient walks to your location following an accident involving an afternoon barbeque where the patient tried to light the fire with gasoline. He presents with severely burned hands that are white and have areas of charred, black patches. He is complaining of pain to his upper arms and back with some blistering. This patient has: A) Partial-and full-thickness burns B) Partial-thickness burns C) Relative hypovolemia D) Superficial flash burns

Answer: A - Partial and full-thickness burns

A disease of the female reproductive tract that may be caused by bacteria, viral, or fungal infection is: A) Pelvic inflammatory disease. B) Endometriosis. C) Ectopic pregnancy. D) Ruptured ovarian cyst.

Answer: A - Pelvic inflammatory disease.

Rebound tenderness in the abdomen is a sign of potential: A) Peritoneal irritation B) Large bowel obstruction C) Liver perforation D) Spleen rupture

Answer: A - Peritoneal irritation

Symptoms that are absent but would be pertinent to the chief compliant if present are known as: A) Pertinent negatives B) Irrelevant evaluation C) Associated symptoms D) Pertinent positives

Answer: A - Pertinent negatives

The study of the origin, nature, properties, and actions of drugs and their effects on living organisms is collectively known as: A) Pharmacology B) Pharmacodynamics C) Pharmacokinetics D) Pharmacostatics

Answer: A - Pharmacology

You suspect that your 64-year-old patient with a history of COPD has sustained a pneumothorax. He tells you that he has pain chest when he breathes deeply. Which of the following treatments would be MOST appropriate? A) Place the patient in the Fowler's position. B) Administer albuterol via a nebulizer. C) Begin ventilations with a bag-valve mask. D) Administer sublingual nitroglycerin.

Answer: A - Place the patient in the Fowler's position.

Which of the following is associated with sickle cell disease? A) Premature RBC destruction B) Decrease in WBC production C) Abnormal WBC production D) Profuse, abnormal night sweats

Answer: A - Premature RBC destruction

Which of the following is the cause of the majority of Acute Renal Failure cases? A) Prerenal renal failure B) Postrenal renal failure C) Intrinsic renal failure D) End stage renal failure

Answer: A - Prerenal renal failure

After analyzing a cardiac arrest patient's heart rhythm, the AED provides a "Deliver shock" message. After clearing the patient, the AEMT's next step should be to: A) Press the shock button, and then perform CPR for two minutes B) Press the shock button, and then allow the AED to analyze the heart rhythm C) Press the shock button, and then check for the return of a pulse D) Check for a pulse, and then press the shock button if a pulse is not present

Answer: A - Press the shock button, and then perform CPR for two minutes

Care for burns should include: A) Preventing heat loss. B) Oxygen by cannula. C) Hypotonic fluid administration for glucose administration. D) Cool water for pain relief in large burn areas.

Answer: A - Preventing heat loss

You are at the scene of a gunshot victim who has been shot in the head at close range with a 38-caliber pistol. The patient is unconscious, with irregular gasping respirations and bradycardia. Your initial actions are oriented to: A) Primary assessment management and rapid transport. B) Classifying the degree of injury. C) Establishing vital signs. D) Intravenous fluid boluses.

Answer: A - Primary assessment management and rapid transport

You are treating a patient with suspected spinal injury following an accident. Your initial actions are: A) Primary assessment while maintaining manual cervical stabilization B) Removal from a vehicle to prevent further injury C) Establishing large-bore IVs and treatment for shock D) Secondary assessment

Answer: A - Primary assessment while maintaining manual cervical stabilization

Your patient is having extreme difficulty breathing and you note bilateral wheezing in all quadrants. She is unable to hold herself upright and has become lethargic. Which of the following should you immediately do? A) Provide positive pressure ventilation. B) Call for Advanced Life Support backup. C) Administer oxygen via nonrebreather. D) Call for orders to intubate.

Answer: A - Provide positive pressure ventilation.

You find your 18-year-old patient thrown from his vehicle. He is breathing six times per minute. You should: A) Provide supplemental oxygen at 15 lpm and consider overdrive ventilation. B) Provide supplemental oxygen at 4-6 lpm and consider overdrive ventilation. C) Provide supplemental oxygen at 4-6 lpm via nonrebreather mask. D) Provide supplemental oxygen at 15 lpm via nonrebreather mask.

Answer: A - Provide supplemental oxygen at 15 lpm and consider overdrive ventilation.

You are at the scene of a vehicle accident with an unconscious, non-breathing patient who has a pulse. The method used to extricate the victim should be: A) Rapidly extricate the victim B) Perform all spinal precautions prior to moving the patient to protect spine C) Apply short immobilization device and extricate rapidly D) Intubate in place and take full spinal precautions prior to movement

Answer: A - Rapidly extricate the victim

A "whiplash" type neck injury is MOST often observed with what type of collision? A) Rear-end impact B) Frontal impacts C) Rollover D) Ejection

Answer: A - Rear-end impact

911 was dialed by a frantic mother for her three-year-old daughter who stuck a small, dried bean into her ear. Assessment shows the bean to be lodged firmly in the ear canal. When treating this patient, the AEMT would: A) Reassure the mother and transport the patient. B) Make one attempt to remove the bean with tweezers. C) Have the patient follow up with her family physician. D) Gently flush the ear canal with sterile water until the bean is dislodged.

Answer: A - Reassure the mother and transport the patient

Because the burn patient's injured tissue will swell, it is important to: A) Remove restrictive clothing B) Administer high-flow oxygen C) Cover the injury with burn sheet D) Start IV therapy in the field

Answer: A - Remove restrictive clothing

As you are examining a female patient experiencing abdominal pain, she suddenly becomes very quiet and will NOT answer your questions related to reproductive function/dysfunction. You should: A) Respect her wishes, provide supportive care, and transport. B) Obtain a signed refusal and document accordingly. C) Allow the patient the opportunity to see her private physician. D) Proceed with a vaginal exam to check for any discharge.

Answer: A - Respect her wishes, provide supportive care, and transport.

Asking a patient, "Can you rate your pain on a 1-10 scale?" represents what part of the OPQRST question format? A) S B) T C) R D) Q

Answer: A - S

A patient, who was constructing a bomb in his basement, has sustained a secondary phase blast injury. Which of the following presentations would the AEMT MOST likely observe? A) Screwdriver impaled in the arm B) Burns to the head, neck, and chest C) Headache and shortness of breath D) Complaint of nausea after inhaling fumes

Answer: A - Screwdriver impaled in the arm

You respond to a 25-year-old man involved in a farm accident. Upon arrival, you see that your patient has suffered a partial amputation of the leg. The wound is bleeding profusely. The patient is unconscious, pale, and clammy. Vitals are respiration 24, pulse 110, and blood pressure 90/60. You control the bleeding and take spinal immobilization precautions. Your next step should be to treat this patient for: A) Shock B) Respiratory distress C) Kidney failure D) Infection

Answer: A - Shock

Which of the following is the MOST common cause of non-traumatic priapism? A) Sickle cell disease B) Multiple myeloma C) Use of antidepressants D) Carbon monoxide poisoning

Answer: A - Sickle cell disease

Your chest pain patient has a pulse of 48. Which pacemaker do you suspect is malfunctioning? A) Sinoatrial node B) Atrioventricular node C) Bundle of His D) Purkinje fibers

Answer: A - Sinoatrial node

The normal, regular rate of electrical activity for the heart is set by the: A) Sinoatrial node. B) Atrioventricular node. C) Bundle of His. D) Purkinje fibers.

Answer: A - Sinoatrial node.

A patient has been critically hurt in a nighttime motor vehicle collision. She was wearing a seatbelt and hit a tree at 70 mph. The car she was driving weighed two tons and was equipped with airbags that did deploy. Which of the following had the greatest impact on her being injured? A) Speed of the vehicle at impact B) Nighttime conditions C) Deployment of airbags D) Weight of the vehicle

Answer: A - Speed of the vehicle at impact

The basic principle to splinting a suspected fracture is: A) Splint the joint above and below the fracture site B) Always use the traction splint when a femur is fractured C) Splinting takes priority over bandaging to preserve limb function D) All of the above are true

Answer: A - Splint the joint above and below the fracture site

A football player injured his knee during practice and is in pain. His left knee is swollen, ecchymotic, and flexed in an upward position. Your partner reports that the distal skin is warm, and he has located a weak pedal pulse. Which one of the following should you perform immediately? A) Splint the knee in the position found prior to providing transport to the hospital B) Place the patient on a long spine board and apply the traction splint en route to the hospital C) Apply a traction splint and straighten the knee until the pain is decreased D) Realign and straighten the injured knee and then immobilize it by binding it to the other leg

Answer: A - Splint the knee in the position found prior to providing transport to the hospital

Which orders are used when the EMS service medical director is confident that all personnel are able to accurately recognize indications and contraindications for a specific drug and give it safely without consultation? A) Standing orders B) Written orders C) Verbal orders D) Computer orders

Answer: A - Standing orders

You have arrived on the scene of an assault involving a knife. Assessment of the unresponsive 21-year-old man reveals him to have sustained multiple lacerations to the arms and abdomen, with a section of his intestine protruding through a large laceration to the area around the umbilicus. He has lost a significant amount of blood. His airway is open and he is breathing shallowly at a rate of 28. His radial pulse is weak, and his skin is cool and pale. Which one of the following should the AEMT do FIRST? A) Start positive pressure ventilation B) Check the blood pressure C) Cover the intestine with saline soaked gauze D) Immobilize to a long board

Answer: A - Start positive pressure ventilation

Pulse quality refers to: A) Strength B) Rate C) Rhythm D) Output

Answer: A - Strength

A young female patient has been stung by a bee and states that her throat is "closing up." She states that she is allergic to bee stings and the last time this happened, she had to have a "tube put into my windpipe." She is struggling to breathe and can only speak a few words at a time. Knowing that allergic reactions can cause swelling in the pharynx and at the level of the larynx, which one of the following respiratory sounds would indicate that the patient indeed has swelling in this area? A) Stridor B) Snoring C) Wheezing D) Gurgling

Answer: A - Stridor

You determine a radial pulse is readily palpable at the radial artery and has uneven intervals between beats. This should BEST be documented on the prehospital care report as: A) Strong and irregular B) Thready and regular C) Prominent and bounding D) Bounding and regular

Answer: A - Strong and irregular

Of the following parenteral routes of medication administration, the knowledgeable AEMT recognizes the slowest relative rate of absorption would be from: A) Subcutaneous (SC) injection B) Intramuscular (IM) injection C) Intravenous (IV) bolus D) Intraosseous (IO) bolus

Answer: A - Subcutaneous (SC) injection

Assessment reveals a drooling 57-year-old man with slurred speech and right arm paralysis. Which of the following should be your FIRST priority? A) Suction the airway. B) Protect the right arm. C) Determine his risk factors for stroke. D) Closely monitor his blood pressure.

Answer: A - Suction the airway.

A 42-year-old man was involved in a fight and was stabbed in the right lateral chest. The knife is still impaled, and he is complaining of shortness of breath. When he speaks he gurgles and blood drains from his mouth. His respirations are labored and his radial pulse is weak. Which one of the following should you do FIRST? A) Suction the blood from the airway B) Remove the knife and seal the injury with a dressing C) Stabilize the knife with bulky dressings D) Provide positive pressure ventilation

Answer: A - Suction the blood from the airway

You have been called to a hotel swimming pool for a confused adult male. Patrons state that the man has been drinking most of the afternoon and has been in and out of the pool. The man is very confused and noncompliant with your requests. His airway is open and his breathing is adequate. His radial pulse is strong, and his skin is wet with water from the pool. While conducting the secondary assessment, you note a large hematoma to the back of his head. Which one of the following should you do next? A) Take inline spinal stabilization and apply a cervical collar B) Transfer the patient to the stretcher and provide immediate transport C) Check the man's pupils and hand grips for equality D) Apply a cold pack to the hematoma and continue the assessment

Answer: A - Take inline spinal stabilization and apply a cervical collar

You are called for a 78-year-old woman who fell earlier in the day. As you enter her kitchen, you observe her sitting in a chair with ecchymotic areas to her left cheek and left arm. There also appears to be some blood on her pants. She is on oxygen and appears to have slightly labored respirations. She states that she fell a few hours ago and now the pain and swelling in her arm are unbearable. Which one of the following should you do FIRST? A) Take manual in-line spinal stabilization B) Examine her arm for deformity C) Place a cold pack to her cheek D) Assess her respiratory status

Answer: A - Take manual in-line spinal stabilization

ou arrive on the scene of a motor vehicle collision. Walking toward you is the unrestrained driver of the vehicle that sustained moderate front-end damage. The patient complains of some back pain, but was walking around after the crash and does not have any neurological deficits. What is your initial action for managing this patient? A) Take manual inline spinal stabilization B) Look for injuries to the patient's back C) Perform the primary assessment D) Obtain the patient's vital signs

Answer: A - Take manual inline spinal stabilization

Your service has a new AED. During the in-service on the new device, the instructor informs you that it is a semi-automated AED and uses a biphasic wave form, as opposed to the monophasic form used by the previous AED. As a knowledgeable AEMT, you should recognize that: A) The new AED will defibrillate with lower amounts of electrical energy B) Less energy but more shocks will be needed to treat cardiac arrest C) The new AED will defibrillate with higher amounts of electrical energy D) The AEMT will not need to press a "shock" button to shock the patient

Answer: A - The new AED will defibrillate with lower amounts of electrical energy

You are told after delivering your chest pain patient to the emergency department that he is on his way to the cardiac catheterization lab. How does this help the patient? A) The surgeon can place a stent to open an occluded artery. B) The patient has some blood drawn to check for cardiac enzyme levels. C) The patient can have some aberrant conduction pathways cauterized. D) An X-ray is taken to determine if the left ventricle is enlarged.

Answer: A - The surgeon can place a stent to open an occluded artery.

A blood pressure is reported as 116/68 mmHg. Which one of the following is TRUE? A) The systolic blood pressure is 116 mmHg B) The top number reflects the diastolic blood pressure C) The diastolic blood pressure is 116 mmHg D) The systolic pressure is determined by subtracting 68 from 116

Answer: A - The systolic blood pressure is 116 mmHg

Which one of the following statements is TRUE about vital signs obtained on a patient with nausea and vomiting? A) The vital signs will help to determine the relative stability or instability of the patient B) The vital signs will allow the AEMT to diagnose the cause of the nausea and vomiting C) The patient's complaint will be significant only if the vital signs are abnormal D) If the vital signs are within normal limits, it is safe to let the patient refuse transport

Answer: A - The vital signs will help to determine the relative stability or instability of the patient

Which of the following would explain why a person might need an implanted pacemaker device? A) There is a problem with the electrical system in the heart. B) The heart muscle is damaged and is unable to contract. C) Their potassium levels are too low which prevents depolarization. D) They have malfunctioning valves between the cardiac chambers.

Answer: A - There is a problem with the electrical system in the heart.

Seizures characterized by rapid, rhythmic contractions of the muscles that may last for several minutes are called: A) Tonic-clonic. B) Febrile. C) Focal motor. D) Complex partial.

Answer: A - Tonic-clonic.

Which of the following statements regarding the indication and application of traction splints is TRUE? A) Traction splints prevent bone ends from overriding, lessen pain, and may help relax muscular contraction B) If the femur fracture is associated with instability about the ankle, apply the hitch to the lower leg three inches above the site of the fractured malleolus C) If the femur fracture is associated with pelvic instability, application of the PASG prior to the traction splint may minimize the potential bleeding D) Traction splints cannot be utilized in open femur fractures

Answer: A - Traction splints prevent bone ends from overriding, lessen pain, and may help relax muscular contraction

Which of the following is the CORRECT treatment for a patient of sexual assault? A) Treat immediate life threats, treat psychological needs, and protect criminal evidence. B) Maintain scene safety and treat only the secondary injuries. C) Treat immediate life threats but don't worry about protecting criminal evidence. D) Allow the patient to shower to support her psychological needs, and transport.

Answer: A - Treat immediate life threats, treat psychological needs, and protect criminal evidence

In the case of trauma patients, the ideal goal for completing on-scene procedures and initiating transport is: A) Under 10 minutes B) Set by medical direction C) One hour D) Between 20 to 30 minutes

Answer: A - Under 10 minutes

What structures may be affected in a urinary tract infection in a female patient? A) Urethra, bladder, kidney B) Vagina, bladder, urethra C) Bladder, ureters, vagina D) Kidney, bladder, fallopian tubes

Answer: A - Urethra, bladder, kidney

A paramedic has asked that you apply a dressing over a sucking chest wound to the left anterior chest. Which one of the following should you use to cover the injury? A) Vaseline gauze B) Sterile gauze soaked in sterile water C) Dry sterile gauze dressing D) Clean washcloth

Answer: A - Vaseline gauze

Which of the following statements about flail segments is FALSE? A) Ventilation is sufficient if only two ribs are involved in the flail segment B) The chest wall will have paradoxical movement C) The patient will have impaired ventilation as a result of instability to chest wall D) All of the above

Answer: A - Ventilation is sufficient if only two ribs are involved in the flail segment

Which one of the following statements about the four chambers of the heart is correct? A) Ventricles are more muscular than the atria. B) Atria are the inferior chambers of the heart. C) Ventricles receive incoming blood from the body. D) Atria are the two largest chambers of the heart.

Answer: A - Ventricles are more muscular than the atria.

Which one of the following cardiac arrest rhythms is the AED designed to shock? A) Ventricular fibrillation B) Bradycardia C) Asystole D) Pulseless electrical activity

Answer: A - Ventricular fibrillation

The AED should never be applied to a patient who is NOT in cardiac arrest because some patients in: A) Ventricular tachycardia may have a pulse B) Ventricular fibrillation may have a pulse C) Asystole may still have a pulse D) Ventricular fibrillation may be conscious and alert

Answer: A - Ventricular tachycardia may have a pulse

Which of the following would lead to an increase in bone mass? A) Weight bearing exercise. B) Being bedbound. C) Immobilization in a cast. D) Astronauts in weightless conditions.

Answer: A - Weight bearing exercise.

MOST drugs that you will administer to pediatric patients are based on: A) Weight in kilograms B) Body surface area C) The patient's age D) Weight in pounds

Answer: A - Weight in kilograms

For the AEMT, which of the following is the MOST important question to ask of a diabetic patient or his family members? A) When was the last time you ate? B) Do you have a fruity taste in your mouth? C) Does diabetes run in your family? D) What kind of insulin do you take?

Answer: A - When was the last time you ate?

While reading a patient's history, you discover the he has had pertussis. Which of the following BEST defines pertussis? A) Whooping cough B) Croup C) Sore throat D) Common cold

Answer: A - Whooping cough

Impetigo is typically seen in: A) Young children. B) Immunocompromised patients. C) The elderly. D) Patients on ventilators.

Answer: A - Young children.

Your patient with suspected abdominal trauma should be treated with: A) spinal immobilization, oxygen, IV, treatment for shock B) lateral recumbent positioning with knees to chest, oxygen, intravenous therapy C) semi-seated position, oxygen, IV, blanket for warmth D) prone positioning, oxygen, IV, cardiac monitoring, rapid transport

Answer: A - spinal immobilization, oxygen, IV, treatment for shock

Increased resistance to blood flow through the pulmonary vasculature often leads to which of the following? A) Right-sided heart failure B) Pulmonary embolism C) Spontaneous pneumothorax D) Atelectasis

Answer: A -Right-sided heart failure

The only time an impaled object should be removed is when: A) There is a potential airway obstruction B) The patient must be taken off an object C) The patient will need placement on a stretcher D) You should never remove an impaled object

Answer: A There is a potential airway obstruction

The AEMT showed that he understands the difference between a pneumothorax and a tension pneumothorax when he made which one of the following statements? A) "A tension pneumothorax causes blood to accumulate around the lung; a pneumothorax involves the accumulation of air only around the lung." B) "A tension pneumothorax causes cardiac output to decrease; a simple pneumothorax does not affect cardiac output." C) "A pneumothorax is caused by a closed chest injury; a tension pneumothorax is caused by an open chest injury." D) "A pneumothorax describes a collapsed lung; a tension pneumothorax involves both a collapsed lung and blood loss."

Answer: B - "A tension pneumothorax causes cardiac output to decrease; a simple pneumothorax does not affect cardiac output."

Your patient has to catheterize herself and states she thinks she has a bladder infection. Which of the following questions will provide you with useful information about the patient's problem? A) "Do you have a history of kidney stones?" B) "Is it painful when you urinate?" C) "Are you having any vaginal discharge?" D) "Are you sexually active?"

Answer: B - "Is it painful when you urinate?"

A patient who appears to be intoxicated has fallen and is complaining of pain to his right arm. He is loud and using profane language. Which one of the following should be your initial approach to the patient? A) "If you do not quiet down, I will call the police and they will arrest you." B) "Let me put an ice pack on your wrist. It will help it feel better." C) "I cannot help you if you keep yelling like that!" D) "Stop yelling or I will have to strap you to the stretcher."

Answer: B - "Let me put an ice pack on your wrist. It will help it feel better."

When using the SAMPLE mnemonic, which of the following statements made by the patient would be categorized under the letter "M"? A) "I have never had a surgical procedure." B) "I take Zocor, but I am not sure why." C) "My doctor's name is Dr. Hansen." D) "I was diagnosed and treated for breast cancer."

Answer: B - "My doctor's name is Dr. Hansen."

A milliliter equals: A) 1/100 of a liter B) 1/1000 of a liter C) 1/10000 of a liter D) 1/10 of a liter

Answer: B - 1/1000 of a liter

After assessing the following patients, which would the AEMT identify as suffering from multisystems trauma? A) 31-year-old woman; large laceration to her leg from a hunting knife; bleeding controlled B) 61-year-old man; became dizzy and fell down five stairs; laceration to the head; open fracture to the left lower leg C) 89-year-old woman fell in a nursing home; hematoma to the back of her head D) 24-year-old man; slipped on ice; tenderness to the right shoulder

Answer: B - 61-year-old man; became dizzy and fell down five stairs; laceration to the head; open fracture to the left lower leg

You are assessing a patient who has chronic kidney failure. He tells you that his glomerular filtration rate is 40 mL/minute. Which of the following GFR would indicate a normal filtration rate? A) 3 L/hour B) 7 L/hour C) 30 mL/minute D) 75 mL/minute

Answer: B - 7 L/hour

Which of the following blood glucose levels is considered normal for an adult? A) 40 mg/dL B) 80 mg/dL C) 120 mg/dL D) 150 mg/dL

Answer: B - 80 mg/dL

You receive an order to give 4 liters of normal saline over eight hours using a 10 gtts/mL drip set. Calculate the drip rate per minute with this setup. A) 38 gtts/min B) 83 gtts/min C) 166 gtts/min D) 41 gtts/min

Answer: B - 83 gtts/min

Two cars have collided head on. One car was traveling 55 mph and the other 35 mph. The total speed of impact would be: A) 55 mph B) 90 mph C) 20 mph D) 35 mph

Answer: B - 90 mph

An intramuscular injection is given at a: A) 45° angle B) 90° angle C) 15° angle D) 100° angle

Answer: B - 90° angle

Which of the following patients would be the BEST candidate for fibrinolytic treatment? A) A 37-year-old woman diagnosed in the ED with a hemorrhagic stroke two hours after onset of symptoms. B) A 48-year-old man whose symptoms started while watching the 6 P.M. news and 911 was called at 6:10 P.M. C) An 84-year-old man who awoke in the morning with symptoms after sleeping eight hours. D) A 64-year-old woman whose symptoms began at 8 A.M. and resolved 20 minutes later.

Answer: B - A 48-year-old man whose symptoms started while watching the 6 P.M. news and 911 was called at 6:10 P.M.

For which of the following patients should you call for ALS assistance? A) A patient with chest pain that is relieved with nitroglycerin. B) A bradycardic patient with chest pain and a blood pressure of 88/62. C) A teenager with shortness of breath and tingling of the lips and fingers. D) A 72-year-old woman with chest pain that subsides when she rests.

Answer: B - A bradycardic patient with chest pain and a blood pressure of 88/62.

You have been dispatched to a home for a 16-year-old woman with shortness of breath. On scene, you find the patient in bed with her mother at her side. Her mother states that her daughter has a fever with a past medical history of cystic fibrosis and wants her transported to the hospital. Which of the following is the underlying pathophysiology of cystic fibrosis? A) A contagious disease in which the lungs are damaged by bacteria and excessive coughing. B) A condition in which excessive and thick mucus often clogs the airways. C) A disease in which a lifelong antibiotic medications are needed for survival. D) A hereditary disease in which the lungs are malformed and smaller than normal.

Answer: B - A condition in which excessive and thick mucus often clogs the airways.

Which of the following could allow direct transmission of a pathogen? A) Touching a door knob after someone with a cold. B) A couple sharing a passionate kiss. C) Being bitten by an animal with rabies. D) Contracting encephalitis after a mosquito bite.

Answer: B - A couple sharing a passionate kiss.

Which of the following fits the definition for acute respiratory distress syndrome (ARDS)? A) A chronic obstructive pulmonary disease B) A form of non-cardiogenic pulmonary edema C) A viral respiratory disease D) An infection resulting in inflammation of the larynx

Answer: B - A form of non-cardiogenic pulmonary edema

In which of the following patients should you suspect pathological fractures? A) A young boy fell off of his bicycle landing after hitting the curb. B) A little girl broke her arm from pulling her shirt over her head. C) An older man who was struck by a car while crossing the street. D) A woman who fell tripped and fell down a stairwell.

Answer: B - A little girl broke her arm from pulling her shirt over her head.

Your protocol allows the administration of nitroglycerin to patients with chest pain under certain circumstances. To which of the following patients should you administer nitroglycerin? A) A man who has chest pain from a fall from a ladder. B) A patient with chest pain and a blood pressure of 124/86 C) A patient who has taken three of his own nitroglycerin tablets. D) A patient who has chest pain after taking Cialis.

Answer: B - A patient with chest pain and a blood pressure of 124/86

A trauma surgeon informs you that the patient you transported to the emergency department earlier was diagnosed with an epidural hematoma. The patient was taken into surgery and the hematoma was removed. Right now the patient is in guarded condition. Based on this information, the AEMT should recognize that an epidural hematoma is: A) Blood from a ruptured artery forming a hematoma within a section of brain tissue. B) A pocket of arterial blood collected between the skull and dura mater. C) Blood from a ruptured vein collected between the dura mater and brain tissue. D) Cerebral spinal fluid accumulated in a pocket on top of the brain tissue.

Answer: B - A pocket of arterial blood collected between the skull and dura mater

When a moderate or serious MOI is found and the patient does NOT present with the signs and symptoms of shock, you should: A) Rule out the possibility of shock B) Act in anticipation of shock C) Assume the lack of internal bleeding D) Increase the time spent on in-field management

Answer: B - Act in anticipation of shock

The condition in which the heart's reduced stroke volume causes an overload of fluid in the lungs is called: A) Prinzmetal's angina. B) Acute heart failure. C) Angina pectoris. D) Myocardial infarction.

Answer: B - Acute heart failure.

Your patient is a 38-year-old diabetic who took his insulin but he is ill and can't keep anything down. He is so dehydrated you are unable to start an IV line. Which of the following is TRUE regarding glucagon? A) 50 mg of glucagon IM will allow the release of glucose from the liver. B) Adequate glycogen stores must be available in the liver for it to be effective. C) Glucose molecules are synthesized from stores of fat in the spleen. D) Glucagon is metabolized slowly to provide a steady source of energy.

Answer: B - Adequate glycogen stores must be available in the liver for it to be effective.

Which medication administered by the AEMT has minimal beta1 adrenergic effects but may increase heart rate and myocardial oxygen demand, theReferenceore it should be used with caution in patients with heart disease? A) Aspirin B) Albuterol sulfate C) Nitroglycerine D) Naloxone

Answer: B - Albuterol sulfate

Which one of the following descriptions would be obtained when asking about the "A" component of the SAMPLE history? A) Airway open B) Allergy to penicillin C) Abdomen soft D) History of asthma

Answer: B - Allergy to penicillin

You were summoned by a family member for a 64-year-old woman who is having an acute respiratory emergency. You note that her chest looks full and she wheezes on expiration. Her vital signs show she is tachycardic and you are unable to hear air movement upon auscultation. You should suspect: A) A pulmonary embolism. B) An asthma attack. C) Emphysema. D) Pulmonary edema.

Answer: B - An asthma attack.

The form of arthritis that causes the bones that form the joints to become fused is: A) Osteoarthritis. B) Ankylosing spondylosis. C) Osteopenia. D) Septic arthritis.

Answer: B - Ankylosing spondylosis.

What type of anemia occurs when the bone marrow does NOT produce an adequate number of RBCs, WBCs, or platelets? A) Pernicious anemia B) Aplastic anemia C) Sickle cell anemia D) Iron deficiency anemia

Answer: B - Aplastic anemia

The preferred and MOST effective technique for splinting a proximal humeral fracture involves: A) Placing the patient's affected limb against the axillary region with circumferential wrapping of cravats or three-inch cling B) Application of a short padded splint to the lateral forearm, placing the patient's affected limb in a sling, including just the wrist and not the elbow, and gently swathing the arm and forearm to the chest C) Placing the patient's affected limb in the position of function against the chest with circumferential wrapping of cravats or three-inch cling D) Application of a short padded splint to the lateral forearm, placing the patient's affected limb in a sling, including both the wrist and elbow, and gently swathing this to the chest

Answer: B - Application of a short padded splint to the lateral forearm, placing the patient's affected limb in a sling, including just the wrist and not the elbow, and gently swathing the arm and forearm to the chest

Management of a traumatic fracture of the femur should include: A) Stabilization of affected limb using a longboard and padding with pillows and blanket rolls, transport to emergency department B) Application of traction using either the unipolar or bipolar device, supportive fluid therapy, transport to trauma center C) Application of traction using a unipolar device, application of the PASG, supportive fluid therapy, transport to trauma center D) Application of traction device, application of the PASG, immobilization using the longboard, transport to emergency department

Answer: B - Application of traction using either the unipolar or bipolar device, supportive fluid therapy, transport to trauma center

Direct pressure has failed to control an arterial bleed on a patient's leg. Which one of the following should the AEMT do next? A) Splint the extremity B) Apply a tourniquet C) Provide oxygen D) Immediate transport

Answer: B - Apply a tourniquet

You are assessing a 68-year-old woman with shortness of breath. Your assessment reveals crackles in all quadrants of the lungs as well as some swelling of the ankles. As you take her vital signs you see that she is becoming anxious and her breathing is becoming more shallow. Which of the following should you do? A) Increase her oxygen flow to 15 lpm via nonrebreather. B) Apply continuous positive airway pressure (CPAP). C) Give her another albuterol treatment. D) Start an IV and consider giving her a fluid bolus of 250 cc.

Answer: B - Apply continuous positive airway pressure (CPAP).

You have been called for a 49-year-old man with chest pain. The patient states that he usually takes nitroglycerin, but it is in his car in a nearby parking lot. His friend states that he also takes nitroglycerin and hands you his bottle of nitroglycerin. Given the patient's condition, what is your next BEST action? A) Administer half a pill of the friend's nitroglycerin B) Ask the friend to get the patient's nitroglycerin C) Have the patient get his nitroglycerin from the car D) Administer the friend's nitroglycerin

Answer: B - Ask the friend to get the patient's nitroglycerin

Acetylsalicylic acid is better known by what trade name? A) Adenocard B) Aspirin C) Adenosine D) Amiodarone

Answer: B - Aspirin

You are treating a six-year-old boy who has signs and symptoms of influenza including fever. In treating this patient, which of the following would be contraindicated? A) Tylenol B) Aspirin C) Motrin D) Fluid challenge

Answer: B - Aspirin

The ongoing assessment in the critically injured trauma patient with spinal injuries should be done: A) Every 15 minutes B) At least every five minutes C) Every minute D) Once on the way to the hospital.

Answer: B - At least every five minutes

Of the following, which is the MOST common cause of a transient ischemic attack? A) Illegal drug use B) Atherosclerosis C) Prinzmetal's angina D) Aneurism

Answer: B - Atherosclerosis

Many EMS agencies are now carrying nerve agent antidote kits as part of Homeland Security precautions. These kits consist of pReferenceilled injectors of which two antidotes? A) Epinephrine (1:1000) 0.3 mg and atropine 2 mg B) Atropine 2 mg and pralidoxime chloride 600 mg C) Diazepam 10 mg and pralidoxime chloride 600 mg D) Epinephrine (1:1000) 0.3 mg and diazepam 10 mg

Answer: B - Atropine 2 mg and pralidoxime chloride 600 mg

Urinary tract infections (UTI) occur secondary to: A) Inflammation of the urinary bladder. B) Bacterial invasion of the genitourinary system. C) Obstruction of the urethra. D) Erosions in the lining of the ureters.

Answer: B - Bacterial invasion of the genitourinary system.

A lethargic 8-year-old boy is in respiratory distress. His airway is patient and respiratory rate is 12 breaths per minute. His breath sounds are diminished bilaterally and you note accessory muscle use with inspiration and expiration. His pulse is 84 beats per minute and his SpO2 is 88percent. Which of the following should you do next? A) Recheck the SpO2 on another finger. B) Begin positive pressure ventilation. C) Get a history and begin a secondary exam. D) Initiate immediate transport to the hospital.

Answer: B - Begin positive pressure ventilation.

You are called to a doctor's office for a patient with hematuria. You know that hematuria is: A) Blood in the stool. B) Blood in the urine. C) Cloudy urine. D) A blood clot.

Answer: B - Blood in the urine

While responding to a domestic emergency with possible injuries, the dispatcher informs you that the police on-scene report a male patient knife cut and arterial bleeding. Given this information, which one of the following do you expect? A) Dark red blood flowing from the injury that is difficult to control B) Bright red blood that is spurting with each beat of the heart C) Bright red blood oozing from the injury site that is easy to control D) Spurting blood that is darker red in color and difficult to control

Answer: B - Bright red blood that is spurting with each beat of the heart

You have been called to a residence of a diabetic patient who is confused. A family member states she could not reach the patient by telephone, so she came over and found the patient awake but confused. The patient can remember his name and address, but cannot remember the day or year. He refuses to go to the hospital, but consents to an assessment. Of the following physical exam findings, which one is MOST concerning? A) Blood glucose level of 77 mg/dL B) Bruise to the left temple C) Heart rate of 62 beats per minute D) SpO2 at 96 percent

Answer: B - Bruise to the left temple

Which one of the following findings is MOST consistent with a basilar skull fracture? A) Bilateral dilated pupils that do not respond to light B) Bruising behind the ear, which develops several hours after the injury C) Altered mental status D) A hematoma over the occipital area of the head

Answer: B - Bruising behind the ear, which develops several hours after the injury

Parenteral delivery of a medication is more predictable because the medication: A) Solution is measured more accurately than external medications B) Bypasses the digestive tract C) Is absorbed at a slower rate D) Can be given over a longer period of time

Answer: B - Bypasses the digestive tract

Systemic activation of clotting mechanisms can result from all of the following EXCEPT: A) Sepsis B) Calcium overdose C) Transfusion reactions D) Surgery

Answer: B - Calcium overdose

A person who has been exposed to a disease and is capable of transmitting the disease is a: A) Host. B) Carrier. C) Fomite. D) Vector.

Answer: B - Carrier.

Inflammation of the skin, including the dermis and subcutaneous layers, usually caused by bacterial infection would be identified as: A) An abscess. B) Cellulitis. C) Necrotizing fasciitis. D) Rhabdomyolysis.

Answer: B - Cellulitis.

You are securing a patient to the long backboard. Which one of the following indicates the appropriate order for securing the straps? A) Head, legs, chest, pelvis B) Chest, pelvis, legs, head C) Legs, head, chest, pelvis D) Head, chest, pelvis, legs

Answer: B - Chest, pelvis, legs, head

Which of the following structures in the inner ear is the organ of hearing? A) Semicircular canal B) Cochlea C) Vestibule D) Bony labyrinth

Answer: B - Cochlea

Solutions that contain large proteins that cannot pass through the capillary membrane are Referred to as: A) Blood products B) Colloids C) Crystalloids D) Oxygen-carrying fluids

Answer: B - Colloids

Assessment of a young girl who was hit while riding her bike reveals her to be responsive to painful stimuli with flexion of the extremities and in respiratory distress. There is marked deformity to her thoracic spine and bruising noted to her anterior chest and abdomen. She does not move her legs when noxious stimuli are applied to the lower extremities. Manual inline spinal stabilization is being held and a cervical collar has been applied. Given the critical nature of this patient, which one of the following is MOST appropriate for her care? A) Transfer her to the long board already on the stretcher and secure with straps en route to the hospital B) Complete immobilization to the long board on scene prior to rapid transport to the hospital C) Wait for family members to arrive prior to moving her to the stretcher for immediate transport to the hospital D) Rapidly transfer her to the stretcher with immobilization to the long board done en route to the hospital

Answer: B - Complete immobilization to the long board on scene prior to rapid transport to the hospital

A patient with a history of diabetes is slightly confused and irritable. According to family members, he accidentally took too much insulin this morning and did not eat breakfast. Since he is conscious with an intact gag Referencelex, medical direction has ordered you to administer oral glucose. Which one of the following BEST represents the indication for the medication? A) High blood sugar B) Confusion and irritability C) Intact gag Referencelex D) Authorization by medical direction

Answer: B - Confusion and irritability

A patient who is short of breath and has a history of stroke with right arm weakness has asked that you help him with the administration of atrovent by way of his metered-dose inhaler. Since your medical director requires on-line authorization for atrovent, you should: A) Administer the medication B) Contact medical direction for authorization C) Assess the patient for side effects D) Withhold the medication and transport

Answer: B - Contact medical direction for authorization

Always perform a patient assessment and take a history to determine whether a drug is indicated. A known drug allergy to the intended drug is always a(n): A) Indication B) Contraindication C) Side effect D) Precaution

Answer: B - Contraindication

Management of a minor laceration should include: A) Field removal of all debris B) Control of bleeding, dressing, and bandaging C) Intravenous fluid bolus D) Tourniquet if bleeding not controlled by direct pressure

Answer: B - Control of bleeding, dressing, and bandaging

Treatment of all serious burns includes all of the following EXCEPT: A) Maintain warmth B) Cool water for pain relief C) Maintain ABCs D) IV fluid administration

Answer: B - Cool water for pain relief

When assessing a patient's pulse, you note that it is irregular. To get an accurate heart rate, the AEMT should: A) Feel the carotid pulse for 30 seconds and multiply by 2 B) Count the number of beats that occur in one minute C) Listen to the pulse with a stethoscope D) Double the number of beats obtained in 30 seconds

Answer: B - Count the number of beats that occur in one minute

The AEMT is correctly calculating a patient's breathing rate when he: A) Asks the patient to describe how he feels when he breathes B) Counts breaths for 15 seconds and multiplies by four C) Obtains a heart rate and divides it by three D) Assesses the patient for any sign of breathing difficulty

Answer: B - Counts breaths for 15 seconds and multiplies by four

You have been called for a male patient who has been shot. Assessment reveals an entrance wound to the left upper quadrant of the abdomen. You also locate an exit wound to the left upper back. Which one of the following should you do immediately? A) Cover the abdominal wound with saline soaked gauze B) Cover the exit wound with a gloved hand C) Auscultate the patient's breath sounds D) Place a cervical collar on the patient

Answer: B - Cover the exit wound with a gloved hand

Ecchymosis around the umbilicus indicative of retroperitineal bleeding is referred to as: A) GreyTurner's sign B) Cullen's sign C) Kehr's sign D) Murphy's sign

Answer: B - Cullen's sign

Ventricular fibrillation should be treated with immediate: A) Synchronized cardioversion. B) Defibrillation. C) Intubation. D) Antidysrhythmics.

Answer: B - Defibrillation.

Cyanosis, a condition causing a bluish skin color, is caused by increased: A) Oxyhemoglobin B) Deoxyhemoglobin C) Methemoglobin D) Carboxyhemoglobin

Answer: B - Deoxyhemoglobin

Which of the following drug orders is incorrect? A) Aspirin, 325mg, PO B) Dextrose 50 percent, 25g, IM C) NTG, 0.4 mg, SL, every five minutes, up to three doses D) Naloxone, 2 mg, IV push

Answer: B - Dextrose 50 percent, 25g, IM

The AEMT knows that the cause underlying distributive shock is: A) Poor fluid intake B) Dilation of the blood vessels C) Damaged heart with poor contractility D) Loss of blood volume

Answer: B - Dilation of the blood vessels

Injuries to the brain can be classified as: A) Open or closed. B) Direct or indirect. C) Linear or depressed. D) All of the above

Answer: B - Direct or indirect

Which of the following disorders have signs and symptoms similar to appendicitis? A) Mallory-Weiss tear B) Diverticulitis C) Esophageal varices D) Cholecystitis

Answer: B - Diverticulitis

The term that refers to inflammation of small outpouchings in the walls of the large intestines is: A) Peptic ulcer disease. B) Diverticulitis. C) Cholecystitis. D) Pancreatitis.

Answer: B - Diverticulitis.

EMRs have removed an intoxicated and confused patient from a car that struck another car at a high rate of speed. The patient was unrestrained and his car did not have airbags. Assessment reveals deformity bilaterally to the hips and femurs and bruising to the knees. Based on this injury pattern, the AEMT would recognize what type of injury mechanism? A) High velocity B) Down and under C) Up and over D) Lateral impact

Answer: B - Down and under

Once applied, you should NOT remove a ________; otherwise, toxins will be released directly into the patient's bloodstream. A) Dressing B) Tourniquet C) Bandage D) Splint

Answer: B - Dressing

You are treating a patient who complains of having excessive bleeding and pain during her regular menstrual cycle. She states this has been an ongoing problem for the past few years. This condition is referred to as: A) Dysplagia. B) Dysmenorrhea. C) Dyspareunia. D) Endometritis.

Answer: B - Dysmenorrhea.

Of the layers of the skin, which has NO direct blood supply? A) Dermis B) Epidermis C) Subcutaneous D) Adipose

Answer: B - Epidermis

Which of the following would be the treatment of choice for a patient with severe anaphylaxis? A) Oxygen via nonrebreather B) Epinephrine C) Albuterol D) Nitroglycerin

Answer: B - Epinephrine

When auscultating the lungs of a trauma victim, you are listening for: A) Crackling and popping B) Equal, bilateral breath sounds C) Stridor D) Paradoxical motion

Answer: B - Equal, bilateral breath sounds

A Mallory-Weiss tear is a disorder of the: A) Rectum. B) Esophagus. C) Liver. D) Stomach.

Answer: B - Esophagus.

While working to build a house, a construction worker was hit in the head by a 2" × 4" plank that slid from the roof and fell to the ground below. The patient has a sizable laceration to the left side of his face and is spitting blood. He also lost several teeth, two of which he is holding. Your initial action when caring for this patient would be to: A) Apply occlusive dressing to the laceration B) Establish manual inline spinal stabilization C) Obtain and preserve the teeth D) Suction the airway

Answer: B - Establish manual inline spinal stabilization

Which orders may be given in an emergency situation in the hospital, such as during cardiac arrest resuscitation attempts, or in other situations with critical patients? A) Face-to-face written orders B) Face-to face verbal orders C) Face-to-face standing orders D) Face-to-face retrograde orders

Answer: B - Face-to-face verbal orders

Which of the following would you MOST likely see in a patient with an infectious disease? A) Abrasions B) Fever C) Bruising D) Hypertension

Answer: B - Fever

You have been called for a patient with extreme back pain. When you are obtaining a medical history, from the patient, he tells you that he has a history of damaged disks in his vertebrae. As a knowledgeable AEMT, you should recognize that: A) Nerves making up the spinal cord have been damaged. B) Fibrous structures between the vertebrae have been injured. C) Ligaments on the side of the spinal column were overstretched D) Vertebrae making up the spinal column were fractured.

Answer: B - Fibrous structures between the vertebrae have been injured.

A patient exhibits swelling and deformity to the wrist. After splinting, which one of the following positions of the hand is MOST appropriate? A) Hand in a fist with thumb inside fist B) Fingers curled inward C) Wrist flexed with fingers extended D) Fingers extended and spread

Answer: B - Fingers curled inward

The tricuspid valve prevents blood from: A) Moving between the right and left ventricles. B) Flowing from the right ventricle into the right atrium. C) Bypassing the pulmonary vessels and entering the right heart. D) Flowing backward through the left atrium.

Answer: B - Flowing from the right ventricle into the right atrium.

The FIRST step that should be taken when establishing IV access is: A) Explain the procedure to the patient B) Gather and prepare all needed equipment C) Cleanse the site thoroughly with antiseptics D) Placement of the constricting band

Answer: B - Gather and prepare all needed equipment

Which of the following is a form of arthritis that occurs from uric acid crystals being deposited in a joint? A) Bursitis B) Gout C) Osteopenia D) Rickets

Answer: B - Gout

A term referring to the number of times a patient has been pregnant is: A) Pareunia. B) Gravidity. C) Para. D) Parity.

Answer: B - Gravidity.

You are treating a 48-year-old man who has fallen off a ladder. He is complaining of pain to his right elbow and right ankle. His vital signs are: blood pressure 110/74, pulse 48, respirations 18 bpm, blood sugar 88 mg/dL. How should you manage his bradycardia? A) Provide aspirin and IV fluid bolus for cardiac compromise. B) He is asymptomatic therefore supportive treatment is indicated. C) Request ALS intercept to administer cardiac medications. D) Apply transcutaneous pacing pads and begin pacing the patient.

Answer: B - He is asymptomatic therefore supportive treatment is indicated.

A 43-year-old man has fallen from a roof and cannot move or feel his arms or legs. When assessing the patient, which one of the following signs would lead the AEMT to suspect the patient is suffering from spinal shock? A) Cool and moist skin B) Heart rate of 68 beats per minute C) Seizure activity D) Cyanosis to the finger tips

Answer: B - Heart rate of 68 beats per minute

Which of the following statements about hepatitis is TRUE? A) The most common cause of hepatitis is alcohol abuse. B) Hepatitis is caused by a variety of infectious agents and chemicals. C) All types of hepatitis lead to chronic liver disease. D) All types of hepatitis are typically fatal within six months to two years.

Answer: B - Hepatitis is caused by a variety of infectious agents and chemicals.

Which of the following is NOT a modifiable risk factor for type 2 diabetes? A) Obesity B) Heredity C) Poor diet D) Lack of exercise

Answer: B - Heredity

Which of the following information obtained during the history taking would lead you to suspect that emphysema is the cause of the patient's respiratory distress? A) The patient has a history of Marfan's disease. B) History of smoking a pack of cigarettes a day. C) The patient has had a cough and fever for three days. D) You can hear audible stridor upon inhalation.

Answer: B - History of smoking a pack of cigarettes a day.

You have been called to a high school science lab where a male student was splashed in the eye with an alkaline solution. He is complaining of burning to the eye as well as pain. After performing the primary assessment and finding no life-threatening conditions, you would: A) Determine the exact compound the patient was splashed with and the appropriate chemical antidote. B) Hold the patient's eye open and begin flushing with large amounts of tap water and continue throughout transport C) Obtain a set of vital signs and determine if the patient is taking medications or has any past medical problems. D) Cover both eyes with eye shields and move to the ambulance for immediate transport.

Answer: B - Hold the patient's eye open and begin flushing with large amounts of tap water and continue throughout transport.

A common injury referred to as "whiplash" involves: A) Lateral flexion B) Hyperextension C) Hyperextension and hyperflexion D) Hyperflexion

Answer: B - Hyperextension

he spinal injuries MOST commonly associated with rear-end impacts are: A) Compression B) Hyperextension C) Rotation D) Lateral bending

Answer: B - Hyperextension

Which of the following is a cause of Secondary headache syndrome? A) Migraines B) Hypertension C) Cluster headaches D) Photosensitivity

Answer: B - Hypertension

Your patient is extremely hypertensive with a diastolic reading of over 130 mmHg. He complains of a severe headache, vomiting, and dizziness. You should suspect: A) Noncardiogenic pulmonary edema. B) Hypertensive encephalopathy. C) Dissecting aortic aneurysm. D) Spinal meningitis.

Answer: B - Hypertensive encephalopathy.

In which of the following would you be MOST likely to have a patient with paresthesia? A) Cor pulmonale B) Hyperventilation syndrome C) Laryngotracheobronchitis D) Status asthmaticus

Answer: B - Hyperventilation syndrome

The condition in which there is an insufficient amount of sugar in the blood is called: A) Diabetic coma. B) Hypoglycemia. C) Hyperglycemia. D) Diabetic ketoacidosis.

Answer: B - Hypoglycemia.

A partial or complete tear of one of the great vessels such as the aorta or vena cava may result in: A) Insignificant bleeding B) Hypovolemia C) Pulsus paradoxus D) Pnuemothorax.

Answer: B - Hypovolemia

Shoulder injuries without pulse deficit should be managed by: A) Aligning the affected limb into the axillary region and securing it to thorax using cravats or three-inch cling B) Immobilization in position found C) Single reduction attempts followed by sling and swathe D) Repeated reduction attempts until the deformity is aligned

Answer: B - Immobilization in position found

Diabetes mellitus is an endocrine disorder characterized by: A) Inadequate glucose production. B) Inadequate insulin activity in the body. C) Excessive glucose production. D) Excessive catecholamine production.

Answer: B - Inadequate insulin activity in the body.

A 44-year-old woman states that she has a history of endometritis. As such, you would recognize that the patient suffers from: A) Absence of a normal menstrual period. B) Infection of the endometrial lining. C) Growth of endometrial tissue outside of the uterus. D) Infection of the both ovaries.

Answer: B - Infection of the endometrial lining.

You are at the scene of a gunshot victim who has been shot in the head at close range with a 38-caliber pistol. The patient is unconscious, with irregular gasping respirations and bradycardia. Your initial actions are oriented to: A) Intravenous fluid boluses B) Initial assessment management and rapid transport C) Establishing vital signs D) Classifying the degree of injury

Answer: B - Initial assessment management and rapid transport

Type 1 diabetes mellitus is characterized by: A) Excessive insulin production. B) Insufficient insulin production. C) Insufficient glucose production. D) Excessive glucose production.

Answer: B - Insufficient insulin production.

The preferred IV solutions for fluid replacement in the prehospital setting would be: A) Hypotonic crystalloids B) Isotonic crystalloids C) Hypertonic colloids D) Isotonic colloids

Answer: B - Isotonic crystalloids

A patient who has been stabbed is losing blood. You know this will make it difficult for him to maintain his blood pressure because: A) His vessels vasodilate. B) It decreases his preload. C) His tissues become hypoxic. D) It increases his stroke volume.

Answer: B - It decreases his preload.

Which of the following is an action of insulin? A) It increases the transfer of sugar from the stomach and small intestine to the bloodstream. B) It increases the movement of sugar from the bloodstream to the cell. C) It increases the circulating level of glucose in the blood. D) It blocks the uptake of sugar by the body's cells.

Answer: B - It increases the movement of sugar from the bloodstream to the cell.

Which of the following is TRUE concerning sickle cell anemia? A) It is an autoimmune disease in which the body destroys its own hemoglobin. B) It is a hereditary disease involving abnormal hemoglobin. C) It is an acquired disease in which the body cannot absorb iron. D) It is a hereditary disease involving lack of intrinsic factor

Answer: B - It is a hereditary disease involving abnormal hemoglobin.

Which of the following is a characteristic of referred pain? A) It is only felt in hollow organs. B) It is felt in a location other than the organ causing it. C) It is caused by psychological stress. D) It is usually described as "crampy" or "colicky."

Answer: B - It is felt in a location other than the organ causing it.

Which one of the following statements about an abdominal aortic aneurysm is TRUE? A) It is more common in women than in men. B) It is most prevalent above the age of 60. C) It is very painful in the early stages. D) Symptoms include pain in the calf muscles.

Answer: B - It is most prevalent above the age of 60.

Which of the following is TRUE regarding Cushing's syndrome? A) It is the result of increasing intracranial pressure. B) It is often the result of a tumor on the pituitary gland. C) It often results from damage to the adrenal cortex. D) Presents with hypotension that does not respond to fluid therapy.

Answer: B - It is often the result of a tumor on the pituitary gland.

All of the following are TRUE regarding myoglobin EXCEPT? A) It results in dark tea- or cola-colored urine. B) It is toxic to the liver and pancreas. C) It can cause acute renal failure. D) It is released as a result of rhabdomyolysis.

Answer: B - It is toxic to the liver and pancreas.

Auscultation of blood pressure is dependent upon hearing the sounds of blood hitting the arterial walls, known as ________ sounds. A) Kornberg's B) Korotkoff's C) Kobileski's D) Kunzlemann's

Answer: B - Korotkoff's

Which IV fluid solution contains a substance that is involved in the blood buffer system and is useful on patients in acidosis, such as those with diabetic ketoacidosis? A) D5W B) Lactated Ringer's C) One-half normal saline D) Normal saline

Answer: B - Lactated Ringer's

All of the following are TRUE regarding sickle cell disease EXCEPT: A) Patients have inherited an abnormal gene from both parents B) Large clumps of sickle cells obstruct arterial blood flow C) The red blood cells to take on an abnormal curved appearance D) The red blood cells have a very short life span

Answer: B - Large clumps of sickle cells obstruct arterial blood flow

Which of the following patient medications would lead you to suspect your patient suffers from heart failure? A) Aspirin B) Lasix C) Nitroglycerin D) Glucophage

Answer: B - Lasix

The postictal patient with no suspected spinal injury should be placed in which position? A) Semi-Fowler's B) Lateral recumbent C) Supine D) Fowler's

Answer: B - Lateral recumbent

A football player was struck in the head and is complaining of a headache and nausea. He also states that he has pain to his upper back and tingling in his left hand. The AEMT is providing proper care for the patient when he: A) Removes the shoulder pads to apply a cervical collar B) Leaves the helmet in place but removes the face mask C) Does not place the patient on a long board due to the presence of shoulder pads D) Removes the helmet to better assess the airway and head

Answer: B - Leaves the helmet in place but removes the face mask

Which of the following is TRUE regarding the pathophysiology of stroke? A) Ischemic stroke is often a result of an aneurysm. B) Neurological damage and death can begin within four minutes. C) Ventricular fibrillation leads to the formation of blood clots in the atria. D) Ischemic stroke usually presents with a severe headache.

Answer: B - Neurological damage and death can begin within four minutes.

Which of the following drug administration routes is considered parenteral? A) Medications swallowed in pill form B) Nitroglycerine spray under the tongue C) Medications given down an NG tube D) All of the above

Answer: B - Nitroglycerine spray under the tongue

A loud, harsh sound over the trachea suggests: A) Pneumothorax B) Normal air movement C) Bronchoconstriction D) Foreign body obstruction

Answer: B - Normal air movement

Which of the following is the BEST way to prevent or control pathogens inside the body? A) Antibiotic therapy B) Normal flora C) Hand washing D) Anti-fungal therapy

Answer: B - Normal flora

Which IV solution should be administered with blood? A) D5W B) Normal saline C) Lactated Ringer's D) Hypotonic solution

Answer: B - Normal saline

A subdural hematoma will generally: A) Always be lethal. B) Occur gradually. C) Contribute to hypovolemic shock. D) Allow for only small amounts of blood accumulation.

Answer: B - Occur gradually

In patients with blood loss, only about _____ of the fluid is still in the vascular space ______ after it is administered. A) One-half; two hours B) One-third; one hour C) Two-thirds; one hour D) Three-quarters; two hours

Answer: B - One-third; one hour

You are treating a patient who has an open wound directly over the fracture site. This is called a(n): A) Compound fracture B) Open fracture C) Laceration D) All of the above

Answer: B - Open fracture

Which of the following medications is administered orally or applied to the buccal mucosa in hypoglycemic patients who are awake and are NOT at risk of aspiration? A) Glucogon B) Oral glucose C) Dextrose 50 percent D) Glycogen

Answer: B - Oral glucose

Which of the following medications are administered in such a way that they bypass the hepatic portal system? A) Aspirin and nitroglycerin B) Oral glucose and nitroglycerin C) Aspirin and oral glucose D) Acetaminophen and aspirin

Answer: B - Oral glucose and nitroglycerin

An unrestrained driver of a car that has struck a tree at 45 mph has suffered a contusion (bruise) to his heart. The AEMT would recognize that this injury occurred during which impact of the collision? A) Physiologic collision B) Organ collision C) Vehicle collision D) Body collision

Answer: B - Organ collision

The basis of the excessive urination characteristic of untreated diabetes is called: A) Facilitated elimination. B) Osmotic diuresis. C) Gluconeogenesis. D) Facilitated diffusion.

Answer: B - Osmotic diuresis.

Swimmer's ear is also known as ________. A) Otitis media B) Otitis externa C) Otitis interna D) None of the above

Answer: B - Otitis Externa

You are assessing a patient who has fallen from a ladder and is in severe pain from an angulated, midshaft femur. Although angulated, there is good circulation and sensation distal to the injury site. The patient is in severe pain. The MOST correct treatment for this patient is: A) Treat for shock with oxygen, IV fluids, and secure patient to spine board B) Oxygen, IV, traction splinting, and give analgesics for pain as indicated C) Oxygen, IV, ladder splint as found, give analgesics for pain, and transport D) Give morphine, splint the extremity as found, and transport

Answer: B - Oxygen, IV, traction splinting, and give analgesics for pain as indicated

You suspect that your patient has a possible pelvic fracture following a fall. The BEST way to splint this injury is: A) To secure to a six-inch backboard B) PASG application C) To place on padded stretcher and secure for comfort D) Longboard splinting

Answer: B - PASG application

The division of the autonomic nervous system that is responsible for controlling vegetative functions is the ______ nervous system. A) Somatic B) Parasympathetic C) Sympathetic D) Afferent

Answer: B - Parasympathetic

Which of the following burns would be classified as a moderate burn? A) Superficial burns < 50 percent body surface area B) Partial-thickness burns < 30 percent body surface area C) Full-thickness burns < 2 percent body surface area D) Partial-thickness burns > 30 percent body surface area

Answer: B - Partial-thickness burns < 30 percent body surface area

The appearance of pale or ashen skin on the patient suffering from hypoperfusion (shock) is caused by: A) Reduced cardiac output B) Peripheral vasoconstriction C) Peripheral vasodilation D) External or internal blood loss

Answer: B - Peripheral vasoconstriction

You want to accurately document that your patient has a sore throat. Which of the following term would correctly identify a sore throat? A) Laryngitis B) Pharyngitis C) Sinusitis D) Rhinovirus

Answer: B - Pharyngitis

Your patient tells you that he is suffering with a bad migraine. Which of the following would provide the patient with the MOST comfort? A) Provide three adult aspirins per protocol. B) Place a cool compress on his forehead. C) Transport in left lateral recumbent position. D) Apply gentle pressure to the temples.

Answer: B - Place a cool compress on his forehead.

Used IV needles should be: A) Recapped and placed into an approved sharps container B) Placed in an approved sharps container immediately after use C) Broken immediately after use D) Stuck in the seat cushions so as to not stick anyone

Answer: B - Placed in an approved sharps container immediately after use

You are called for a person having a seizure. When you arrive, you note that the patient is postictal and a bystander is providing care for the patient. Which of the following would be considered appropriate care by the bystander? A) Placing a bite block between the patient's teeth. B) Placing the patient in the recovery position. C) Giving the patient small sips of water. D) Sweeping the patient's mouth to remove vomit.

Answer: B - Placing the patient in the recovery position.

Which of the following is responsible for the second step of the clotting process? A) Erythrocytes B) Platelets C) Plasma D) Leukocytes

Answer: B - Platelets

Pressure in the filled ventricle at the end of diastole is called: A) Afterload. B) Preload. C) Cardiac output. D) Stroke volume.

Answer: B - Preload.

What is the purpose of valves in the circulatory system? A) Control the volume of blood moved in each pulse B) Prevent the backflow of blood in the vessels C) Support the bifurcation of the veins D) Allows for expansion and contraction of the veins

Answer: B - Prevent the backflow of blood in the vessels

The ________ is designed to assess limited, isolated injuries. A) Secondary assessment B) Primary trauma assessment C) Detailed body assessment D) Primary assessment

Answer: B - Primary trauma assessment

Which of the following is the only artery that carries deoxygenated blood away from the heart? A) Aorta B) Pulmonary C) Semilunar D) Coronary

Answer: B - Pulmonary

Which of the following is CPAP MOST likely indicated? A) Pneumonia B) Pulmonary edema C) Asthma D) Pneumothorax

Answer: B - Pulmonary edema

You are called for a six-year-old boy who has been very sick for several days. His signs and symptoms include hypersalivation, and hydrophobia. These symptoms are immediately suggestive of: A) Tularemia. B) Rabies. C) Hanta virus. D) Lyme disease.

Answer: B - Rabies.

Your 32-year-old male patient presents with constant, severe flank pain, radiating to his groin, with tenderness over the costovertebral angle, and hematuria. What should be the MOST likely differential diagnosis? A) Nephritis B) Renal calculi C) Chronic renal failure D) Appendicitis

Answer: B - Renal calculi

Your patient has suffered a stroke and is having difficulty speaking. Which of the following is the BEST way to communicate with her? A) Try to use sign language. B) See if she can write. C) Speak more loudly to her. D) Have her nod her head.

Answer: B - See if she can write.

The ________ contain sensors necessary for equilibrium. A) Vestibule B) Semicircular canals C) Incus D) Cochlea

Answer: B - Semicircular canals

Which of the following is the MOST common chief complaint related to acute pancreatitis? A) Diarrhea B) Severe pain C) Blood in the stool D) Headache

Answer: B - Severe pain

You have provided an albuterol treatment for your asthma patient and after a few minutes you reassess her. Which of the following would indicate that she is worsening rather than improving? A) She has become tachycardic. B) She is becoming lethargic. C) Her blood pressure is unchanged. D) She reports increased palpitations.

Answer: B - She is becoming lethargic.

Which of the following is characteristic of a patient with hyperglycemia? A) Use of excessive amounts of insulin or lack of adequate food intake. B) Slow onset, Kussmaul's respirations and acetone odor on breath. C) Cool, moist skin, agitated behavior, and increased heart rate. D) Sudden onset of altered mental status and combativeness.

Answer: B - Slow onset, Kussmaul's respirations and acetone odor on breath.

The amount of oxygen delivered depends on the patient's oxygenation state. Oxygen flow should be titrated to maintain a: A) SpO2 > 90 percent B) SpO2 > 95 percent C) SpO2 < 90 percent D) SpO2 < 95 percent

Answer: B - SpO2 >95 percent

To assess a patient's blood pressure, the AEMT will need a: A) Pulse oximeter B) Sphygmomanometer C) Pair of gloves and stethoscope D) Pulse oximeter and stethoscope

Answer: B - Sphygmamanometer

Significant injuries to the head and neck require: A) Detailed neurological evaluation on scene. B) Spinal immobilization. C) Only BLS intervention. D) Insertion of a supraglottic airway device.

Answer: B - Spinal immobilization.

You are by the side of a young male patient who hit his head after diving from a hillside into a shallow lake. The patient is unresponsive and has a large hematoma to his chin. A paramedic asks you to apply painful stimuli to his arms and legs. You recognize that the paramedic wants you to assess for which one of the following? A) Alcohol intoxication B) Spinal injury C) True unresponsiveness D) Extremity fractures

Answer: B - Spinal injury

Injury to which of the following organs causes greatest concern for hemorrhagic shock? A) Kidneys B) Spleen C) Appendix D) Liver

Answer: B - Spleen

All of the following could be a result of a vaso-occlusive crisis EXCEPT: A) Pulmonary embolism B) Spontaneous bleeding C) Priapism D) Ischemia

Answer: B - Spontaneous bleeding

Which one of the following injuries is MOST likely to cause peritonitis? A) Blunt trauma to the spleen B) Stab wound to the small intestine C) Contusion to the pancreas D) Bullet injury to the liver

Answer: B - Stab wound to the small intestine

Which of the following can produce a toxin that leads to toxic shock syndrome? A) Enterococcus B) Staphylococcus aureus C) Herpes virus D) Cryptosporidium

Answer: B - Staphylococcus aureus

As you are performing a rapid trauma exam of the chest, you note the presence of a crackling, rice-crispy-like sensation beneath the skin under the clavicles. This is MOST likely: A) Crepitus B) Subcutaneous emphysema C) Flail chest D) Cyanosis

Answer: B - Subcutaneous emphysema

________ drugs are typically absorbed through the mucous membranes beneath the tongue A) Subungual B) Sublingual C) Intradermal D) Subcutaneous

Answer: B - Sublingual

Consider a hemorrhagic stroke if the patient describes it as: A) Lasting for more than 18-24 hours. B) Sudden in onset and "the worst headache of my life" C) Being accompanied by nausea and photosensitivity. D) Dull or achy pain with a feeling of forceful pressure.

Answer: B - Sudden in onset and "the worst headache of my life"

Your patient who has been involved in a flash fire tells you he was working on a small engine when volatile chemicals ignited and burned him. He has burned facial hair, and you suspect inhalation injury. Considering their depth, you would classify his burns as: A) Full-thickness B) Superficial C) Minor D) Partial-thickness

Answer: B - Superfical

You are assessing a 34-year-old woman who appears to be in hypovolemic shock. You would expect her pulse to be: A) Bradycardic and irregular B) Tachycardic and weak C) Tachycardic and bounding D) Bradycardic and strong

Answer: B - Tachycardic and weak

A young man riding a motorcycle was hit head on and ejected from the bike. Your scene size-up reveals him to be lying supine in the roadway with obvious deformity to his right thigh and left ankle, with blood noted to his pants in these areas. Patient assessment reveals he is unresponsive with snoring respirations and is breathing 16 times per minute. His radial pulse is moderate in strength, and his skin is warm and dry. Which one of the following actions should be performed FIRST? A) Check for distal pulses in both lower extremities B) Take and maintain manual inline spinal stabilization C) Expose and splint both legs to prevent further injury D) Open the airway using the head-tilt/chin-lift

Answer: B - Take and maintain manual inline spinal stabilization

Air accumulation in the thorax that causes displacement of the mediastinum will result in a condition known as: A) Flail segment B) Tension pnuemothorax C) Sucking chest wound D) Simple pnuemothorax

Answer: B - Tension pnuemothorax

You are called for a patient with respiratory distress. She is only able to speak in one or two word phrases so you check her medication list to give you some history. Which of the following medications would be along acting beta2-agonist? A) Albuterol B) Terbutaline C) Proventil D) Atrovent

Answer: B - Terbutaline

A female patient with osteoporosis stepped sideways on her foot and has suffered an open tibial fracture to the distal portion of her right leg. Which one of the following interventions indicates that appropriate care is being given for this injury? A) The protruding bone is gently replaced back under the skin B) The bone end and soft-tissue wounds are covered with a sterile dressing C) A pressure dressing is tightly applied to the open wound prior to splinting D) The AEMT cleans dirt from within the wound and off the bone prior to splinting

Answer: B - The bone end and soft-tissue wounds are covered with a sterile dressing

A patient complaining of eye pain informs you that he was diagnosed with a corneal abrasion three days ago. He called 911 today because the discomfort has not yet gone away and he desires medical attention. Based on this information, which of the following is TRUE? A) Fluid is leaking from the posterior chamber. B) The clear structure that covers the pupil has been damaged. C) The sclera must be infected. D) The lens of the eye has been abraded.

Answer: B - The clear structure that covers the pupil has been damaged

Which of the following statements is TRUE regarding placement of a constricting band when initiating IV access in the hand or arm? A) The constricting band should be just tight enough to impede arterial blood flow without restricting venous passage B) The constricting band should be just tight enough to impede venous blood flow without restricting arterial passage C) The constricting band should be placed distal to the intended site of puncture D) Never leave the band in place for more than three minutes

Answer: B - The constricting band should be just tight enough to impede venous blood flow without restricting arterial passage

Assessment of a patient involved in a motor vehicle collision reveals him to have crepitus to the left humerus, instability to the left lateral chest wall and flank, and pain on palpation to left hip region. He also complains of pain to the right side of the neck. Based on this injury pattern, the AEMT would recognize the patient was: A) The passenger in a car hit from behind B) The driver of a car hit on the driver's side C) An unrestrained occupant in a car struck from the side D) An unrestrained driver involved in a head-on collision

Answer: B - The driver of a car hit on the driver's side

The AEMT is correct when he identifies baseline vital signs as: A) Any change in two consecutive sets of vital signs B) The first set of vital signs obtained C) A set of vital signs most close to normal D) The most important set of vital signs

Answer: B - The first set of vital signs obtained

Which one of the following shows that the AEMTs are correctly using a vest-type short spine immobilization device? A) The cervical collar is applied after the torso has been secured to the device B) The head is secured to the device after the torso has been secured C) The head is secured to the device immediately after the cervical collar is placed D) The head is secured first followed by the torso and legs

Answer: B - The head is secured to the device after the torso has been secured

In which of the following situations would the administration of aspirin to a cardiac patient be prohibited? A) The patient does not currently take aspirin. B) The patient has a history of asthma. C) The patient has a diastolic blood pressure greater than 100 mmHg. D) None of the above

Answer: B - The patient has a history of asthma.

Which of the following statements regarding the administration of nitroglycerin tablets is TRUE? A) An increase in blood pressure should be expected. B) The patient may complain of a headache after administration. C) If it is fresh it will have no noticeable taste. D) It takes 10 to 15 minutes for nitroglycerin to take effect.

Answer: B - The patient may complain of a headache after administration.

Which one of the following assessment findings BEST shows that the patient's injury should be classified as an abrasion? A) Intact skin with ecchymosis noted; patient states minor pain B) The skin is scraped and red; blood oozing from the injury site C) Irregular break in the skin; bleeding moderate but controllable D) Intact skin with large accumulation of dark blood underneath

Answer: B - The skin is scraped and red; blood oozing from the injury site

Which of the following is TRUE regarding the peripheral circulation? A) Provide blood flow to the internal organs. B) The vessels lay closer to the surface of the body. C) Contains the large vessels of the body. D) Vessels vasodilate during hypovolemic shock.

Answer: B - The vessels lay closer to the surface of the body.

Which one of the following BEST describes why the AEMT administers or assists with the administration of medications? A) To justify a lights-and-siren transport B) To stabilize a patient's medical condition C) To prevent illness and injury D) To expand the scope of the EMT's practice

Answer: B - To stabilize a patient's medical condition

Which of the following BEST describes a flail chest? A) Four or more adjacent ribs, fractured in three or more places B) Two or more adjacent ribs, fractured in two or more places C) Three or more adjacent ribs, fractured in four or more places D) Three or more adjacent ribs, fractured in two or more places

Answer: B - Two or more adjacent ribs, fractured in two or more places

An elderly patient in a nursing home has fallen in a narrow hallway and you suspect spinal injury. After manually stabilizing the cervical spine and applying a cervical collar, one method to move the patient to a spine board would be: A) Drag the patient in a straight line B) Use a scoop (orthopedic) stretcher C) Use a flat sheet and two rescuers D) Use a vest-type immobilization device

Answer: B - Use a scoop (orthopedic) stretcher

Which of the following complaints would raise your suspicion that a female patient has a urinary tract infection (UTI)? A) Pain in the back that spreads into the legs B) Vaginal discharge that is greenish in color C) Burning sensation every time she urinates D) Urinating constantly with associated polydipsia

Answer: B - Vaginal discharge that is greenish in color

Which of the following is a viral disease in the herpes family that causes general malaise and itchy, fluid-filled blisters on the skin that later crust and scab? A) Rubella B) Varicella C) Rubeola D) Mumps

Answer: B - Varicella

The QRS complex reflects: A) Atrial depolarization. B) Ventricular depolarization. C) Ventricular repolarization. D) Atrial repolarization.

Answer: B - Ventricular depolarization.

Which of the following may result in hypoglycemia in the diabetic patient? A) Failure to take insulin B) Vigorous exercise C) High fever D) Overeating

Answer: B - Vigorous exercise

En route to the scene of a shooting victim, dispatch informs you that the suspect has not been located yet. Based on this information, you should: A) Be aware as you arrive on the scene that the patient may have committed suicide B) Wait until police arrive and secure the scene prior to approach C) Document this information D) Recognize the need to rapidly transport the victim

Answer: B - Wait until police arrive and secure the scene prior to approach

What is the MOST common type of medication error resulting in death accounting for over 40 percent of all medication error related deaths? A) Incorrect drug route of the drug B) Wrong dosage of the drug C) Giving the wrong drug D) Improper documentation of the drug

Answer: B - Wrong dosage of the drug

An AEMT has accurately checked the skin color of a patient. Which one of the following indicates correct documentation? A) "Warm skin noted on the upper arm." B) "Thoracic and abdominal skin normal in color." C) "Pale skin color noted to the conjunctiva." D) "Forehead and face show pink skin color."

Answer: C - "Pale skin color noted to the conjunctiva."

While running in a parking lot, a young boy tripped and fell. He hit his face and mouth on a curb, knocking one of his top front teeth from its socket. Bleeding has been controlled and the tooth found. What instructions would you give your partner when caring for the tooth? A) "Wrap the tooth in dry gauze and put it in a plastic bag placed in cool water." B) "Scrub the tooth first to get the dirt off of it and then we will put it back in the socket." C) "Rinse the tooth first then put it in a cup of saline for transport." D) "Make sure to only touch it by its root when scrubbing the dirt off it."

Answer: C - "Rinse the tooth first then put it in a cup of saline for transport."

An adult female is found unresponsive after being shot in the chest. Which one of the following AEMT statements about this patient is of MOST concern? A) "She does not have any jugular venous distention." B) "I see an entry wound but no exit wound anywhere." C) "She is getting more difficult to ventilate." D) "Her SpO2 has gone from 90 percent to 93 percent."

Answer: C - "She is getting more difficult to ventilate."

The AEMT is CORRECT when he makes which one of the following statements about assessment of the pupils? A) "Dilated pupils are less of a concern than pupils that are constricted." B) "Constricted pupils are less of a concern than are dilated pupils." C) "Some people naturally have unequal pupils, but both should react to light." D) "If a patient's pupils are dilated but react to light, the pupillary exam is considered normal."

Answer: C - "Some people naturally have unequal pupils, but both should react to light."

You are treating a four-year-old for hypoglycemia using a dextrose solution. The CORRECT dosage and concentration for this patient would be: A) 0.1 mg/kg of a 50percent dextrose solution B) 0.5 mg/kg of a 10percent dextrose solution C) 0.5 mg/kg of a 25 percent dextrose solution D) 0.5 mg/kg of a 50 percent dextrose solution

Answer: C - 0.5 mg/kg of a 25 percent dextrose solution

Beck's triad is often found in a pericardial tamponade. Which three of the following findings does it include? 1. JVD 2. Hypotension 3. Wheezing 4. Muffled heart sounds 5. Stridor A) 1, 3, 4 B) 2, 3, 5 C) 1, 2, 4 D) 3, 4, 5

Answer: C - 1, 2, 4 (JVD, Hypotension, and Stridor)

Place the following steps to administer fluid using a measured volume administration set in their CORRECT order. 1. Prepare the tubing by closing all clamps and insert the flanged spike into the IV solution bag's spike port. 2. Close the uppermost clamp and open the flow regulator until you reach the desired drip rate. Leave the airway handle open, so that air replaces the displaced fluid. 3. Continue to fill the burette chamber with the designated amount of solution. 4. Open the airway handle. Open the uppermost clamp and fill the burette chamber with approximately 20 mL of fluid. Squeeze the drip chamber until the fluid reaches the fill line. Open the bottom flow regulator to purge air through the tubing. When all air is purged, close the bottom flow regulator. A) 1, 2, 3, 4 B) 2, 1, 4, 3 C) 1, 4, 3, 2 D) 4, 3, 2, 1

Answer: C - 1, 4, 3, 2

One kilogram equals: A) 2 pounds B) 2.3 pounds C) 2.2 pounds D) 2.1 pounds

Answer: C - 2.2 lbs

Symptoms from a transient ischemic attack (TIA) generally resolve within: A) 6 hours. B) 12 hours. C) 24 hours. D) 48 hours

Answer: C - 24 Hours

A patient has been burned to the entire anterior chest, abdomen, neck and face. Using the rule of nines, approximately what percentage of the patient is burned? A) 18 B) 36 C) 27 D) 9

Answer: C - 27

Patients with anemia usually become symptomatic when their hematocrit falls below: A) 10 percent B) 15 percent C) 30 percent D) 40 percent

Answer: C - 30 percent

Of the following, which one BEST illustrates how the AEMT should document the administration of a medication? A) Assist patient with administration of medication via metered-dose inhaler B) Medical direction (Dr. Cole) ordered albuterol at 1245 C) 325 mg of aspirin administered orally at 1325 D) 0.4 of nitroglycerin administered sublingually at 0800

Answer: C - 325 mg of aspirin administered orally at 1325

Which of the following is the Parkland formula for determining fluid resuscitation in patients with moderate to severe burns? A) 8 mL × patient weight in kilograms × BSA involved B) 8 mL× patient weight in pounds × BSA involved C) 4 mL × patient weight in kilograms × BSA involved D) 4 mL × patient weight in pounds × BSA involved

Answer: C - 4 mL × patient weight in kilograms × BSA involved

Current fluid resuscitation guidelines recommend: A) 400 mL × body surface area involved over 4 hours. B) 40 mL/kg × body surface area involved over 24 hours. C) 4 mL/kg × body surface area involved over 24 hours. D) Administer fluid bolus as necessary to keep systolic bp at 100 mmHg.

Answer: C - 4 mL/kg × body surface area involved over 24 hours.

Put the steps for obtaining medication from a glass ampule in the proper order. 1. Snap off top of ampule using gauze to protect fingers. 2. Hold ampule upright and tap to dislodge trapped solution. 3. Withdraw medication into hypodermic needle and administer medication. 4. Confirm medication, allergies, dose, and expiration date. 5. Dispose of needle, syringe, and broken glass ampule. A) 1, 2, 3, 4, 5 B) 3, 2, 1, 5, 4 C) 4, 2, 1, 3, 5 D) 2, 3, 4, 1, 5

Answer: C - 4, 2, 1, 3, 5

The CORRECT pediatric dosage of ibuprofen would be: A) 5 to10 mg/kg every 3 to 4 hours B) 1 to 3 mg/kg every 6 to 8 hours C) 5 to 10 mg/kg every 6 to 8 hours D) 1 to 3 mg/kg every 3 to 4 hours

Answer: C - 5 to 10 mg/kg every 6 to 8 hours

You are giving your child some Children's Motrin for a fever. There are 40 mg/mL. The directions state to give 240 mg for your child's weight. How many milliliters should you give? A) 3 mL B) 40 mL C) 6 mL D) 36 mL

Answer: C - 6 mL

In a microdrip administration set, how many drops usually make up one mL? A) 20 drops B) 10 drops C) 60 drops D) 50 drops

Answer: C - 60 drops

The AV node has an intrinsic rate of self excitation, which is ______ beats per minute. A) 20 to 40 B) 40 to 60 C) 60 to 100 D) 80 to 100

Answer: C - 60 to 100

Which of the following patients show signs of inadequate perfusion? A) A five-year-old child with a capillary refill of two seconds and a seal bark cough. B) A 65-year-old complaining of indigestion and is diaphoretic. C) A 52-year-old woman with congestive heart failure who is very lethargic. D) A teenager with shortness of breath and tingling of the lips and fingers.

Answer: C - A 52-year-old woman with congestive heart failure who is very lethargic.

Which of the following pathogens would be considered a communicable disease? A) A man contracts tetanus after stepping on a nail. B) A family gets food poisoning after eating tainted mushrooms. C) A health care worker gets contracts influenza during an outbreak. D) A woman breaks out with poison ivy after working in her yard.

Answer: C - A health care worker gets contracts influenza during an outbreak.

After asking your partner to check the patient's IV line, she reports to you it is not flowing. She lowers the IV bag below the insertion site, and notes blood flow into the IV tubing. This indicates: A) Extravasation B) A collapsed vein C) A patent IV line D) A clogged catheter

Answer: C - A patent IV line

Which of the following is an example of a patient in the fulminant stage of a disease? A) A patient has been exposed but is asymptomatic. B) The patient is between periods of the disease. C) A patient has an acute onset of the disease. D) The patient has succumbed to the disease.

Answer: C - A patient has an acute onset of the disease.

You are caring for a 68-year-old woman displaying signs of stroke. She has right sided weakness and slurred speech. Her vitals are: pulse 118, respirations 22 with a SpO2 of 92percent, and blood pressure of 188/98. Which of the following would be the LEAST appropriate care? A) Apply high flow oxygen and titrate to effect. B) Place her in the position of comfort. C) Administer nitroglycerin for hypertension. D) Check her blood glucose level.

Answer: C - Administer nitroglycerin for hypertension.

You have been dispatched for a patient whose arm was caught in a grinding machine. Assessment reveals a deformed arm covered with a bloody towel. The patient is alert and anxious, and he has a patent airway. His breathing is adequate. Radial pulse is fast but strong. Which one of the following should you do next? A) Perform a secondary assessment focusing on his arm B) Start positive pressure ventilation C) Administer oxygen through a nonrebreather mask D) Insert an oropharyngeal airway and administer oxygen

Answer: C - Administer oxygen through a nonrebreather mask

In providing fluid therapy for the head injury patient with a delayed capillary refill, diminished level of consciousness, and other signs of shock, the fluid should be administered: A) Slowly so as not to raise ICP any higher. B) Aggressively with two IVs of five percent dextrose and water. C) Aggressively with two large-bore IVs of an isotonic solution. D) Slowly so that the head injury does not bleed more profusely.

Answer: C - Aggressively with two large-bore IVs of an isotonic solution.

Which of the following is TRUE regarding energy production in the body? A) Water and carbon monoxide are formed and are easily eliminated. B) Anaerobic metabolism decreases the accumulation of H+. C) An increase in H+ concentration decreases the body's pH. D) Cells require carbon dioxide in order to bind to oxygen.

Answer: C - An increase in H+ concentration decreases the body's pH.

Which of the following classifications BEST describes atropine? A) Parasympathomimetic B) Neuromuscular blocking agent C) Anticholinergic D) Sympathomimetic

Answer: C - Anticholinergic

The primary treatment for an acute sickle cell crisis includes all of the following EXCEPT: A) IV fluid administration B) Narcotic analgesia C) Anticoagulants D) Oxygen therapy

Answer: C - Anticoagulants

Which of the following vessels has the least ability to constrict? A) Arterioles B) Venules C) Aorta D) Vena cava

Answer: C - Aorta

When interviewing a woman with a suspected gynecological problem, which of the following would be MOST appropriate? A) Ask her if she is having any problems "down there." B) Inquire is she has had any rough sex lately. C) Ask if there is a chance she could be pregnant. D) Transport without asking any personal questions.

Answer: C - Ask if there is a chance she could be pregnant.

Firefighters have pulled a middle-age man from a burning bedroom. The patient is unresponsive and has obvious deformity to his left ankle. His respirations are rapid, stridorous and shallow. Your partner inserts an oropharyngeal airway and begins ventilating the patient with a bag-valvemask. Which one of the following actions would you perform next? A) Check the patient's breath sounds B) Instruct your partner to hyperventilate the patient C) Assess the patient's carotid pulse D) Find out how long the patient was in the burning structure

Answer: C - Assess the patient's carotid pulse

A 56-year-old man is complaining of chest pain without respiratory distress. His vital signs show a blood pressure of 146/88, pulse 88, and pulse oximetry reading of 97percent. How should you treat this patient? A) Administer oxygen via nonrebreather at 15 liters per minute. B) Provide oxygen via bag-valvemask at 15 liters per minute. C) Assure that his airway is secure and monitor his respiratory status. D) Assist the patient with his metered dose inhaler.

Answer: C - Assure that his airway is secure and monitor his respiratory status.

You are first on the scene of a two-car motor vehicle collision. In one car there are two patients entrapped in the vehicle, while in the other, there is a male driver complaining of chest pain. At this time, your primary concern is: A) Triaging the patients B) Calling for additional help C) Assuring personal safety D) Determining severity of injuries

Answer: C - Assuring personal safety

A condition in which fatty plaque builds up in arteries, eventually narrowing the lumen and restricting blood flow is called: A) Arteriosclerosis. B) Claudication. C) Atherosclerosis D) An aneurysm.

Answer: C - Atherosclerosis

Ipratropium bromide is chemically related to: A) Albuterol B) Magnesium C) Atropine D) Epinephrine

Answer: C - Atropine

While on standby at a semi-professional baseball game, you are summoned onto the field for a player complaining of severe leg pain after colliding with the catcher of the opposing team. The primary assessment shows no threats to the airway, breathing, or circulation. The secondary assessment reveals a severely deformed knee that is swollen and ecchymotic. The leg is pale and cool, and the patient cannot move his leg when asked to do so. In addition, you cannot palpate a pedal pulse. Which one of the following is your priority at this time? A) Apply a traction splint and enough traction until a pulse returns B) Check the patient's radial or carotid pulse C) Attempt to straighten the leg until a pulse returns D) Apply cold packs to the knee and transport immediately

Answer: C - Attempt to straighten the leg until a pulse returns

All of the following are correct regarding bacterial meningitis EXCEPT? A) Meningococcal bacteria enter the blood causing damage to blood vessels. B) Seizures and altered mental status can occur in bacterial meningitis. C) Bacterial meningitis is typically less serious than viral meningitis. D) Meningococcal bacterium commonly resides in the nasal passages.

Answer: C - Bacterial meningitis is typically less serious than viral meningitis.

Which one of the following conditions could be responsible for causing obstructive shock? A) Loss of blood in the urine B) Infection throughout the body C) Blood clots in the lungs D) Poor transfer of oxygen at the capillary level

Answer: C - Blood clots in the lungs

When assessing an adult patient with respiratory distress, which of the following signs and symptoms would be MOST alarming? A) Delayed capillary refill B) An SpO2 saturation of 92% C) Blue coloration to the lips D) Temperature of 101.4°F

Answer: C - Blue coloration to the lips

The rule of palms is BEST used when: A) Third-degree burns are present B) Body surface area is large C) Body surface area is smaller D) Superficial burns are present

Answer: C - Body surface area is smaller

Cushing's triad involves: A) Tachycardia, hypotension, and decreased respirations. B) Tachycardia, hypertension and increased respirations. C) Bradycardia, hypertension, and irregular respirations. D) Bradycardia, hypotension, and decreased respirations.

Answer: C - Bradycardia, hypertension, and irregular respirations

When auscultating breath sounds in a patient complaining of shortness of breath reveals wheezing. Which of the following is responsible for this finding? A) Significant hypoxia B) Swelling in the throat C) Bronchiole constriction D) Mucus in the lungs

Answer: C - Bronchiole constriction

What physiologic response will be caused by a beta2-specific agent? A) Increased heart rate B) Constriction of pupils C) Bronchodilation D) Bronchoconstriction

Answer: C - Bronchodilation

You have been called for a patient who had a large bag of lime powder tear as he was unloading it from a truck. On arrival you notice that the patient is wearing shorts and has lime dust covering both lower legs. He is complaining of leg pain from the chemical powder. Which one of the following should be the AEMT's next immediate action? A) Move him to the stretcher for transport. B) Wash his legs off with plenty of water. C) Brush as much of the lime off his legs as possible. D) Cut his clothes off to better assess the extent of the burns.

Answer: C - Brush as much of the lime off his legs as possible.

When performing the primary assessment on a patient with an isolated spinalcord injury, you note that he is in severe respiratory distress and struggling to breathe. Where should you suspect the spinalcord injury has occurred? A) Thoracic spine B) Lumbar spine C) Cervical spine D) Diaphragm

Answer: C - Cervical spine

Which one of the following is the MOST commonly injured area of the spinal cord? A) Sacral spine B) Lumbar spine C) Cervical spine D) Thoracic spine

Answer: C - Cervical spine

You are caring for a patient with significant stroke symptoms. You are completing your assessment when you notice that her Sp02 reading is dropping. Which of the following should you do next? A) Check the batteries in your pulse oximeter. B) Check her skin color and temperature. C) Check her airway and breathing. D) Check the oxygen cylinder pressure.

Answer: C - Check her airway and breathing.

Signs and symptoms of a life-threatening emergency due to asthma include all of the following EXCEPT: A) Cyanosis B) Altered mental status C) Chest pain D) Audible stridor

Answer: C - Chest pain

A sign or symptom that causes a patient or bystander to request medical help is known as the: A) Primary problem B) Present illness C) Chief complaint D) Associated symptom

Answer: C - Chief Complaint

You are called for an 18-year-old woman with dyspnea with sharp chest pain. Your assessment shows she has cramps in her hands and fingers. How should you treat this patient? A) Administer a dose of three baby aspirin. B) Have her breathe slowly into a paper bag. C) Coach her to use her diaphragm to breathe. D) Administer an albuterol treatment.

Answer: C - Coach her to use her diaphragm to breathe.

The phase of blood clotting in which fibrin is released normally takes 7 to 10 minutes to complete. This is called the: A) Platelet phase B) Aggregate phase C) Coagulation phase D) Vascular phase

Answer: C - Coagulation phase

Which of the following is NOT a bone of the middle ear? A) Malleus B) Stapes C) Cochlea D) Incus

Answer: C - Cochlea

Muscle edema, in which the pressure within the fascial compartment of the muscle increases above the capillary perfusion pressure, results in: A) Pathological fractures. B) Rhabdomyolysis. C) Compartment syndrome. D) Dry gangrene.

Answer: C - Compartment syndrome.

Your patient is a 17-year-old teenager involved in a one-vehicle rollover. The patient is complaining of chest pain and, upon inspection, he exhibits bruising and tenderness to the area. You suspect internal hemorrhage and treat appropriately. The patient appears nervous and has increasing respirations and pulse rates. The blood pressure is within normal limits, and the level of consciousness is maintaining itself. This tells you that profusion of the brain is taking place. Which stage of shock would you consider this patient to be in? A) Hypovolemic B) Decompensated C) Compensated D) Irreversible

Answer: C - Compensated

A young intoxicated male patient cannot move his left arm and leg after diving into the shallow end of a pool and hitting the bottom head first. The AEMT should recognize which one of the following mechanisms as MOST likely to be responsible for this injury? A) Rotation B) Distraction C) Compression D) Penetration

Answer: C - Compression

The heart receives its nutrients from the: A) Anterior great cardiac vein. B) Blood within its chambers. C) Coronary arteries. D) Aorta.

Answer: C - Coronary arteries.

Which of the following respiratory disorders presents with a seal bark cough and stridor? A) Epiglottitis B) Bronchiolitis C) Croup D) Pneumonia

Answer: C - Croup

An IV fluid solution such as normal saline that contains water, electrolytes, and in some cases, dextrose would be Referenceerred to as a(n): A) Colloid B) Analgesic C) Crystalloid D) Collusion

Answer: C - Crystalloid

You have a patient with suspected internal bleeding and cool, clammy, ashen skin. Her blood pressure starts to fall, and her level of consciousness is rapidly dropping. Which stage of shock would you consider this patient to be in? A) Irreversible B) Compensated C) Decompensated D) Hypovolemic

Answer: C - Decompensated

A patient with wet gangrene will have infection that causes swelling of the tissues. What can result as a consequence of the gangrene? A) Decubitus ulcers B) Necrotizing fasciitis C) Decreased capillary perfusion D) Metabolic acidosis

Answer: C - Decreased capillary perfusion

In the mnemonic DCAP-BTLS, "D" stands for: A) Drainage B) Debilitation C) Deformities D) Dislocation

Answer: C - Deformities

Late one night, you are called for a patient with an acute onset of altered mental status. You find that the elderly patient is confused and combative. His medical history includes hypertension, diabetes and kidney failure. Which of the following is the MOST likely cause of his altered mental status? A) Alzheimer's B) Dementia C) Delirium D) Neuropathy

Answer: C - Delirium

The BEST way to give the emergency department an idea of the severity of blood loss for a patient with vaginal bleeding is to: A) Compare the total blood loss to that of a normal menses. B) Determine if the patient is hypotensive from excessive blood loss. C) Determine how many sanitary pads are saturated in one hour. D) Compare the blood loss to what is lost during childbirth.

Answer: C - Determine how many sanitary pads are saturated in one hour.

A young boy riding an ATV did not see a chain stretched between two poles and hit it with his neck while traveling forward. Subsequently, he was thrown backward from the bike with significant force. He was not wearing a helmet. EMRs are with the patient and are holding manual inline spinal immobilization as well as administering oxygen. When you are assessing this patient, which of the following signs would indicate the injury that must be addressed and managed FIRST? A) Laceration to the forehead B) Open fracture to the left arm C) Difficulty speaking D) Partially avulsed ear

Answer: C - Difficulty speaking

Injuries to the head that result in the stretching, tearing, and disruption of the CNS are called: A) Subdural hematomas. B) Cerebral contusion. C) Diffuse axonal injuries. D) Basilar skull fractures.

Answer: C - Diffuse axonal injuries

Trauma assessment and management begins with: A) Identifying the MOI B) Evaluating resources at scene C) Dispatch information D) Scene size-up

Answer: C - Dispatch information

Orally administered drugs are largely absorbed through the: A) Stomach B) Colon C) Duodenum D) Spleen

Answer: C - Duodenum

A non-pacemaker heart cell that automatically depolarizes is called a(n) ______ focus. A) Ischemic B) Irritable C) Ectopic D) Reentry

Answer: C - Ectopic

A route of drug administration is a way in which the drug gains access to the body. Routes that allow access through the vasculature of the gastrointestinal system are called: A) Parenteral B) Percutaneous C) Enteral D) Aseptic

Answer: C - Enteral

Your patient is a 23-year-old man who admits that he has chlamydia. He complains of lower abdominal pain and nausea, and says he has swelling in one testicle. He also says the pain has been increasing for the last couple of days. Which of the following should you suspect? A) Paraphimosis B) Testicular torsion C) Epididymitis D) Phimosis

Answer: C - Epididymitis

Your young patient appears ill with a high fever, difficulty swallowing and breathing, and drooling. You see that he is sitting upright and leaning forwarding. What should you suspect? A) Croup B) Pneumonia C) Epiglottitis D) Scarlet fever

Answer: C - Epiglottitis

Which one of the following medications should the EMT administer orally? A) Nitroglycerin B) Albuterol C) Aspirin D) Epinephrine

Answer: C - Epinephrine

Which of the following medications may an AEMT administer by intramuscular injection? A) Dextrose 50 percent and glucagon B) Nitroglycerin and epinephrine 1:1000 C) Epinephrine 1:1000 and glucagon D) Nitroglycerin and dextrose 50 percent

Answer: C - Epinephrine 1:1000 and glucagon

You are caring for a patient who had uncontrolled bleeding from his nose due to a clotting disorder. How would you describe this problem in your documentation? A) Purpura B) Petechiae C) Epistaxis D) Hemolysis

Answer: C - Epistaxis

A 46-year-old woman alcoholic is hypotensive, in severe distress, complaining of dysphagia, and vomiting bright red blood. Of the following, which is the MOST likely cause of this patient's clinical condition? A) Acute gastroenteritis B) Hemorrhagic pancreatitis C) Esophageal varices D) Mallory-Weiss tear

Answer: C - Esophageal varices

When caring for a patient suffering from a nervous system condition or disease, the FIRST priority in patient care is to: A) Administer thrombolytic therapy. B) Assess for any neurological deficits. C) Establish and maintain a patent airway. D) Apply high-concentration oxygen.

Answer: C - Establish and maintain a patent airway.

You have been dispatched to a pool party for a 19-year-old man with shoulder pain. He states that he dove off the diving board and hit the bottom of the pool with his right shoulder and back. Although there is alcohol at the party, he has not been drinking. He is alert and oriented and has obvious redness and abrasions to his right shoulder, neck, and back. He denies pain or discomfort to any other part of the body. Which one of the following should the AEMT do immediately? A) Measure and apply a cervical collar B) Perform a secondary assessment on the patient C) Establish manual inline spinal stabilization D) Start positive pressure ventilation.

Answer: C - Establish manual inline spinal stabilization

A patient has suffered an open crush injury to his elbow. As you approach, you note the patient sitting upright holding a towel to his elbow. Although the towel is soaked with blood, the elbow does not appear to be actively bleeding. Your FIRST action in caring for this patient should be to: A) Place a sterile dressing over the injury B) Apply oxygen through a nonrebreather mask C) Evaluate the patient's airway D) Examine the patient's elbow

Answer: C - Evaluate the patient's airway

Which of the following is associated with cocaine and methamphetamine use, as well as the use of other drugs? A) Dementia B) Meningismus C) Excited delirium D) Photophobia

Answer: C - Excited delirium

Your patient is a 25-year-old woman with a history of diabetes. She is confused, agitated, and verbally abusive to you, and she is very sweaty. Although she refuses to give a history of the present illness, you should suspect which of the following as the likely cause of her presentation? A) Failure to take her insulin. B) A non-diabetic-related problem. C) Failure to consume sufficient sugar D) Overeating foods high in sugar.

Answer: C - Failure to consume sufficient sugar

You receive an order to give 324 mg of aspirin by mouth. You have on hand a bottle of 81 mg tablets of chewable aspirin. How many tablets will need to be administered to this patient? A) Two B) One C) Four D) Three

Answer: C - Four

Your have responded for a 34-year-old male patient who is lethargic and febrile. He has pus weeping from a small gray-colored lesion on his scrotum and a foul odor in the area. You should suspect which of the following? A) Orchitis B) Epididymitis C) Fournier's gangrene D) Paraphimosis

Answer: C - Fournier's gangrene

Bystanders state that a woman complained of chest pain and dizziness just before she fell down a flight of stairs. She is alert but confused and she states that she is nauseated and may vomit. How should you position this patient for transport? A) Left lateral position on a long spine board. B) Supine with her head turned to one side. C) Fully immobilized to a long spine board. D) Semi-Fowler's with a cervical collar in place.

Answer: C - Fully immobilized to a long spine board.

The term automaticity refers to a cell's capability of: A) Responding promptly to electrical stimuli. B) Generating an electrical impulse from one cell to another. C) Generating an electrical impulse on their own. D) Contraction or shortening an impulse

Answer: C - Generating an electrical impulse on their own.

Which medication is administered intramuscularly for emergency treatment of severe hypoglycemia when it is NOT possible to establish an IV line? A) Glucose (D50) B) Glycogen C) Glucagon D) Glucose (D10)

Answer: C - Glucagon

An option to the administration of 50 percent dextrose in the unconscious patient without a suitable intravenous site is: A) Dextrose 25 percent IM. B) Insulin SQ. C) Glucagon IM. D) Insulin IV.

Answer: C - Glucagon IM.

Which of the following is the BEST description of diabetic ketoacidosis? A) A complication of type 2 diabetes without ketoacidosis B) An extreme, life-threatening form of hyperthyroidism C) Hallmark signs include polyphagia, polyuria, and polydipsia D) A disease of glucose regulation and insufficient insulin production

Answer: C - Hallmark signs include polyphagia, polyuria, and polydipsia

A patient has referred pain to her left shoulder. Which one of the following should the AEMT suspect? A) Damage to the gall bladder B) Rupture of the small intestine C) Hemorrhage from the spleen D) Trauma to the right kidney

Answer: C - Hemorrhage from the spleen

A common injury referred to as "whiplash" involves: A) Hyperextension B) Hyperflexion C) Hyperextension and hyperflexion D) Lateral bendinG

Answer: C - Hyperextension and hyperflexion.

Which IV fluids cause water to leave the cells and enter the vascular space? A) Hypotonic crystalloids B) Isotonic crystalloids C) Hypertonic crystalloids D) Isotonic colloids

Answer: C - Hypertonic crystalloids

In which of the following conditions would you find a patient with myxedema? A) Hyperthyroidism B) Graves' disease C) Hypothyroidism D) Addison disease

Answer: C - Hypothyroidism

A patient has been involved in a very serious motor vehicle collision and is in shock. Your assessment findings indicate that he sustained blunt trauma to the abdominal and pelvic areas. From what type of shock would this patient be suffering? A) Cardiogenic B) Obstructive C) Hypovolemic D) Distributive

Answer: C - Hypovolemic

You are managing a patient with a large flail segment to the right lateral chest. As such, you should recognize that the immediate threat to life is: A) Rib fractures B) Infection C) Hypoxia D) Blood loss

Answer: C - Hypoxia

You are called for a man who fell and needs assistance getting up. During your assessment, he tells you that he has missed his last two dialysis appointments. Which of the following would be the MOST appropriate treatment for this patient? A) Start your IV above his shunt. B) Deliver a fluid bolus of 250 cc. C) IV of NS at keep open rate. D) Deliver a fluid challenge of 1000 cc.

Answer: C - IV of NS at keep open rate.

You respond for a patient who tells you he has hypothyroidism. Which of the following signs or symptoms would you expect? A) A patient who is much thinner than normal. B) Eyes that seem to protrude. C) Inappropriately cold hands and feet. D) Evidence of increased thyroid activity.

Answer: C - Inappropriately cold hands and feet.

Puncture wounds have a high risk of: A) Internal bleeding. B) Shock. C) Infection. D) Cavitation.

Answer: C - Infection

Which of the following can cause an increase in white blood cell production? A) Living at high altitudes B) Hypoxia C) Infection or inflammation D) Chemotherapy

Answer: C - Infection or inflammation

A 74-year-old woman is complaining of abdominal pain after falling down five stairs and hitting her abdomen and left hand on a mailbox at the bottom. She also states pain to her left wrist and right ankle, both of which show obvious deformity. She is alert and oriented, and her abdomen is free of bruising but is tender to the left upper and lower quadrant. Her vital signs are: pulse 132, respirations 22, blood pressure 106/86, and SpO2 at 96 percent. Oxygen has been applied, and she is fully immobilized. Advanced life support has been requested and is 12 minutes away. What is your next action? A) Immobilize the wrist and ankle B) Monitor the patient and wait for ALS assistance C) Initiate immediate transport D) Apply ice to the wrist and ankle

Answer: C - Initiate immediate transport

A patient who has been injured in a hunting accident by a shotgun would be expected to have: A) A lethal injury pattern regardless of external signs B) A large amount of energy dissipation to surrounding tissues C) Injuries to areas of impact with little energy dissipation D) Cavitation injuries and tissue damage due to energy dissipation

Answer: C - Injuries to areas of impact with little energy dissipation

You are caring for a 64-year-old woman with chest pain and shortness of breath. She is alert and oriented with an open airway. Her breathing is adequate and you have provided her with oxygen via nonrebreather at 15 lpm. She has a normal pulse and her skin is warm and dry. Which of the following should you do next? A) Obtain her heart rate and blood pressure. B) Prepare for immediate transport. C) Inquire about history of heart problems. D) Hook up the automatic external defibrillator.

Answer: C - Inquire about history of heart problems.

Which of the following is characteristic of end-stage renal disease? A) Painful urination B) Due to use of acetaminophen C) Irreversible kidney damage D) Urinary retention

Answer: C - Irreversible kidney damage

You are transporting a patient who had an episode of chest pain. You administered two doses of nitroglycerin which relieved the patient's pain. He asks you why the nitroglycerin alleviated the pain. Which of the following is the BEST answer to his question? A) It decreases the heart rate which allows the heart muscle to rest more. B) It has a sedative effect on the brain which makes you less aware of the pain. C) It dilates the blood vessels which decreases the workload on the heart. D) It causes the body to relax which decreases the need for oxygen.

Answer: C - It dilates the blood vessels which decreases the workload on the heart.

You have a young male patient who tells you he has Duchenne muscular dystrophy. All of the following are common with this disease EXCEPT: A) The patient will be wheelchair bound by age 12. B) Weakened respiratory muscles can lead to pneumonia. C) It occurs more commonly in females than males. D) Patients usually die from respiratory failure in the early 20s.

Answer: C - It occurs more commonly in females than males.

Referred pain to the left shoulder caused by diaphragmatic irritation, which is caused by the presence of blood and may indicate splenic injury is known as: A) GreyTurner's sign B) Cullen's sign C) Kehr's sign D) Murphy's sign

Answer: C - Kehr's sign

Many of the proteins that make up the remainder of plasma volume, such as albumin and clotting factors, are manufactured in the: A) Bbone marrow. B) Spleen. C) Kidneys. D) Liver.

Answer: C - Kidneys.

The AEMT recognizes that a splint applied too loosely can result in which one of the following? A) Decreased pain to the fracture site B) Conversion of an open fracture to a closed fracture C) Laceration of previously intact blood vessels D) New fractures along the bone

Answer: C - Laceration of previously intact blood vessels

Which IV fluid solution is an isotonic crystalloid that contains sodium, chloride, potassium, and calcium? A) D5W B) Normal saline C) Lactated Ringer's D) One-half normal saline

Answer: C - Lactated Ringer's

Esophageal varices are most associated with: A) Kidney disease. B) Pancreatitis. C) Liver disease. D) Gastroesophageal reflux disease.

Answer: C - Liver disease.

A patient has been shot in the abdomen. Assessment reveals that the bullet entered the body in the right upper quadrant and exited the lower right back. Given this finding, the AEMT should assume which of the following organs may have been injured? A) Stomach, gall bladder B) Liver, gall bladder, left kidney C) Liver, gall bladder, right kidney, small intestine D) Liver, gall bladder, right kidney, small intestine, spleen

Answer: C - Liver, gall bladder, right kidney, small intestine

A patient has been shot in the chest with a rifle. Your assessment reveals a decreasing level of consciousness and inadequate breathing. Positive pressure ventilation is being administered, and you have covered the entrance wound located midclavicular at the second intercostal space on the left side of the chest. Which one of the following should you do next? A) Get a full set of vital signs B) Place a cervical collar C) Look for an exit wound D) Obtain a blood pressure

Answer: C - Look for an exit wound

The MOST common cause of status epilepticus is: A) A sudden drop in blood sugar levels. B) Brain swelling from traumatic head injury. C) Low levels of antiseizure medications. D) Overdose of a hypnotic medication.

Answer: C - Low levels of antiseizure medications.

Following placement of the cervical collar: A) The primary assessment should be done B) Rescuers may focus on establishing intravenous access C) Manual stabilization must be maintained D)The patient is ready for movement

Answer: C - Manual stabilization must be maintained

Following placement of the cervical collar: A) The patient is ready for movement. B) Rescuers may focus on establishing intravenous access. C) Manual stabilization must be maintained. D) The initial assessment should be done.

Answer: C - Manual stabilization must be maintained.

A lifeguard has fallen 10 feet from his chair and has deformity, swelling, and ecchymosis to her right lower leg. The leg also is rotated medially. Another lifeguard has taken and is maintaining manual inline spinal stabilization. As your partner conducts the primary assessment, you should take which one of the following actions? A) Ready the traction splint for application B) Align the leg in a neutral position C) Manually stabilize the lower right leg D) Check the right foot for pulses and motor ability

Answer: C - Manually stabilize the lower right leg

Hyperventilating a brain injury patient: A) Assures good cerebral oxygenation. B) May aggravate cerebral edema by causing cerebral vasodilation. C) May aggravate the head injury by significantly reducing cerebral blood flow. D) Reduces the PaCO2, thus promoting cerebral oxygenation.

Answer: C - May aggravate the head injury by significantly reducing cerebral blood flow

After you apply the pulse oximeter to a patient complaining of chest discomfort, she asks you what you are assessing. You would respond by telling her that the pulse oximeter: A) Can tell how much carbon dioxide she is exhaling B) Is a tool used to see if the airway needs to be opened C) Measures the amount of oxygen circulating in the blood D) Provides the most accurate evaluation of the heart rate

Answer: C - Measures the amount of oxygen circulating in the blood

Which of the following statements regarding chest compressions is TRUE? A) Compress the chest at a rate of at least 120 compressions per minute in an adult B) Use a compression pattern of 15 compressions followed by two ventilations for adult patients C) Mechanical devices can be a useful tool in the case of prolonged CPR or in situations where manpower is scarce D) Compress the adult chest to a depth of one inch and then allow the chest to fully recoil

Answer: C - Mechanical devices can be a useful tool in the case of prolonged CPR or in situations where manpower is scarce

You arrive on the scene of a two-car motor vehicle collision. The patient was the unrestrained driver of a car that struck another from behind at 25 mph. In the course of the collision, the patient flew forward and struck the windshield with his head. No airbags were deployed. The patient extricated himself and is not complaining of any head, neck, or back pain; however, you do note a small cut on his hand, which occurred as he was getting out of the car. When asked about preexisting medical problems he tells you that he did have herniated disks in his lumbar spine repaired several years ago. Which one of the following should provide the AEMT with the strongest reason to immobilize this patient to the long spine board? A) Self-extrication of the patient B) History of back surgery C) Mechanism of injury D) Laceration to the hand

Answer: C - Mechanism of injury

If you suspect significant trauma to an unconscious victim, you should open the airway using: A) Head-tilt/chin lift B) Head-tilt without the chin-lift C) Modified jaw-thrust D) Head-tilt/neck-lift

Answer: C - Modified jaw-thrust

The MOST common cause of spinal injury in individuals between 16 and 35 years of age is: A) Falls B) Penetrating injury C) Motor vehicle crashes D) sports injuries

Answer: C - Motor vehicle crashes

Which of the following is the result of a portion of the heart muscle dying due to a lack of oxygen? A) Angina pectoris B) Cardiac arrest C) Myocardial infarction D) Heart failure

Answer: C - Myocardial infarction

You are caring for a patient complaining of epigastric pain. You are concerned because of the possibility of which of the following? A) Influenza B) Extreme diarrhea with dehydration C) Myocardial infarction D) All of the above

Answer: C - Myocardial infarction

An 86-year-old woman has called 911 for rectal bleeding. There is a large amount of dark red blood and clots in the toilet and on the patient's clothing. The EMR reports a blood pressure of 84/68 mmHg with a heart rate of 124 beats per minute. Given this information, the AEMT should recognize: A) Normal blood pressure since the patient is alert and oriented B) Normal heart rate given the patient's age C) Narrowed pulse pressure and possible shock D) Diastolic hypertension and risk for stroke

Answer: C - Narrowed pulse pressure and possible shock

Which of the following medications the AEMT may administer has a mechanism of action that works as a CNS depressant? A) Ethanol B) Epinephrine C) Nitrous oxide D) Oxygen

Answer: C - Nitrous oxide

Asking a patient, "What were you doing when this problem began?" represents what part of the OPQRST question format? A) P B) Q C) O D) R

Answer: C - O

The universal blood donor type is: A) A. B) B. C) O. D) AB.

Answer: C - O

A male driver was ejected from his vehicle after it rolled several times at a high rate of speed. As you approach the patient you note that he is unresponsive and struggling to breathe. He also has a laceration to the left side of his face and multiple contusions to his legs. After assigning another AEMT to take manual inline spinal stabilization, what should you do? A) Start positive pressure ventilation B) Insert an oropharyngeal airway C) Open the airway using the jaw-thrust maneuver D) Apply a properly sized cervical collar

Answer: C - Open the airway using the jaw-thrust maneuver

Which intravenous catheter is pReferenceerred for peripheral venous access? A) Butterfly catheter B) Hollow-needle catheter C) Over-the-needle catheter D) Catheter inserted through the needle

Answer: C - Over-the-needle catheter

Which of the following statements about defibrillator pad placement is FALSE? A) If the patient is wet, wipe the chest dry before application of the pads B) If possible use a safety razor to shave excessive chest hair before placing the pads C) Pads should be placed directly over internally implanted pacemakers or defibrillators D) A safety razor should be kept with the AED and defibrillator pads

Answer: C - Pads should be placed directly over internally implanted pacemakers or defibrillators

What is considered the hallmark of acute abdominal emergency? A) Dysuria B) Swelling C) Pain D) Bruising

Answer: C - Pain

A patient dove into a shallow pool and struck his head on the bottom. Your assessment findings indicate that he has no motor or sensation ability in his legs, but he can move his arms. The AEMT should recognize which one of the following conditions? A) Quadriplegia B) Hemiplegia C) Paraplegia D) Tetraplegia

Answer: C - Paraplegia

Which of the following would be more likely to have urinary tract infections? A) Patients who take blood pressure medications. B) Men who have multiple sexual partners. C) Patients who have sustained a spinal injury. D) People who eat a lot of grapefruit.

Answer: C - Patients who have sustained a spinal injury.

Which of the following statements about return of spontaneous circulation (ROSC) is TRUE? A) Patients with ROSC always regain responsiveness in the prehospital setting B) ROSC patients need aggressive ventilation to increase venous return to the heart C) Patients with ROSC are at a high risk for a second cardiac arrest in the prehospital setting D) Prehospital treatment of ROSC can always address the underlying cause of cardiac arrest

Answer: C - Patients with ROSC are at a high risk for a second cardiac arrest in the prehospital setting

Which of the following is a contraindication to administering hypotonic D5W IV solutions? A) Patients with congestive heart failure B) Patients with renal failure C) Patients with traumatic brain injury D) Patients with suspected hyperkalemia

Answer: C - Patients with suspected hyperkalemia

A condition in which blood accumulates in the sac surrounding the heart is called: A) Traumatic asphyxia B) Pulsus paradoxus C) Pericardial tamponade D) Jugular venous distention

Answer: C - Pericardial tamponade

The part of the nervous system that extends throughout the body is called the ______ nervous system. A) Afferent B) Somatic C) Peripheral D) Ventral

Answer: C - Peripheral

When performing the secondary assessment, which one of the following signs is MOST suggestive of a spinal-cord injury? A) Headache and nausea B) Bruising to the abdomen C) Persistent penile erection D) Pain in the right leg

Answer: C - Persistent penile erection

The branch of pharmacology concerned with the ways drugs are absorbed, distributed, metabolized, and eliminated from the body is: A) Pharmacology B) Pharmacodynamics C) Pharmacokinetics D) Pharmacostatics

Answer: C - Pharmacokinetics

You have been called to a public building for a 32-year-old male patient who fell down a flight of stairs. As you pull up, he walks to the ambulance and states that he would like to be looked at because his shoulder and lower back hurt. You immediately take inline manual spinal stabilization and examine his back, which reveals no sign of injury. How should you immobilize this patient? A) Have the patient lie on the ground and immobilize in the usual fashion B) Place the long board on the stretcher and carefully assist the patient onto the long board C) Place a long board behind the patient and immobilize in a standing position D) Place a cervical collar on the patient and transfer him to the stretcher

Answer: C - Place a long board behind the patient and immobilize in a standing position

Your patient is a 68-year-old man who has apparently suffered a stroke. He is unable to hold himself in an upright position on the stretcher and has begun to vomit. Which of the following actions would be MOST appropriate? A) Tighten the chest straps and begin suctioning. B) Insert an oral airway and lower the head of the stretcher. C) Place him in the left lateral recumbent position. D) Suction and begin ventilations with a bag-valvemask.

Answer: C - Place him in the left lateral recumbent position.

Symptoms experienced by the type 1 diabetic include: A) Lack of thirst, appetite, and urine production. B) Shallow respirations, anorexia, and mucous production. C) Polyphagia, polyuria, and polydipsia. D) Anorexia, polyuria, and dysphagia.

Answer: C - Polyphagia, polyuria, and polydipsia.

Vision impairment as a result of old age and the loss of elasticity in the lens is called ________. A) Diplopia B) Astigmatism C) Presbyopia D) Myopia

Answer: C - Presbyopia

When assessing a patient with a possible stroke, what is the priority of care? A) Identifying the risk factors for stroke. B) Determining if there is a family history of stroke. C) Recognizing the signs and symptoms of stroke. D) Determining what type of stroke has occurred.

Answer: C - Recognizing the signs and symptoms of stroke.

If blood glucose exceeds 180 mg/dL, the kidneys will: A) Retain glucose in the bloodstream. B) Increase the ability to store glucose. C) Release excess glucose in the urine. D) Retain both sodium and potassium.

Answer: C - Release excess glucose in the urine.

An elderly woman is suffering from a closed head injury after falling down five steps. She is responsive to painful stimuli and is breathing shallowly. When evaluating her airway, you note that she has both upper and lower dentures. Her upper dentures are secure, but her lower dentures are loose. Prior to providing positive pressure ventilation, you would: A) Re-secure the lower dentures B) Leave the dentures as you found them C) Remove the lower dentures D) Remove both the upper and lower dentures

Answer: C - Remove the lower dentures

Which of the following has signs and symptoms that can mimic an abdominal aortic aneurysm? A) Phimosis B) Uremia C) Renal colic D) Nephritis

Answer: C - Renal colic

You suspect that your patient has a kidney infection because he is pointing to pain in his back. In your documentation, which of the following would you use to describe the area of pain? A) Extraperitoneal B) Peritoneum C) Retroperitoneal D) Parietal peritoneum

Answer: C - Retroperitoneal

Which of the following is NOT one of the "six rights" of medication administration? A) Right person B) Right time C) Right diagnosis D) Right route

Answer: C - Right diagnosis

You are called for a 48-year-old man who is complaining of back and leg pain. Your assessment shows that he pain and tenderness in his lower back with tingling through his left buttock and down his left leg. He tells you that he has been moving furniture. You should suspect: A) Decubitus. B) Kyphosis. C) Sciatica. D) Osteoarthritis.

Answer: C - Sciatica.

You arrive on the scene to find a 20-year-old woman with a recent history of drug use. She is breathing at a rate of six breaths per minute and withdraws to pain. What is your FIRST priority in treating this patient? A) Find out what she might have taken B) Establish an intravenous line C) Secure the airway, provide ventilation and oxygenation D) Administer 2 mg Narcan

Answer: C - Secure the airway, provide ventilation and oxygenation

Bell's palsy is a temporary weakness or paralysis of which cranial nerve? A) Third B) Fifth C) Seventh D) Ninth

Answer: C - Seventh

After assisting a patient with her EpiPen, she states that it is much easier for her to breathe. However, her heart rate has increased from 92 to 118 beats per minute. In relation to the epinephrine, the AEMT recognizes the increased heart rate as a(n): A) Contraindication B) Therapeutic effect C) Side effect D) Allergy

Answer: C - Side effect

Which of the following information is NOT part of a complete drug order? A) Drug name B) Drug dosage C) Side effects D) Route of administration

Answer: C - Side effects

The presence of a potential myocardial contusion should be based on: A) Open chest wound B) Penetrating chest trauma C) Significant blunt chest trauma D) All of the above

Answer: C - Significant blunt chest trauma

All of the following are symptoms of otitis media EXCEPT ________. A) Pain B) Edema C) Sinus infection D) Pus

Answer: C - Sinus Infection

Which one of the following descriptions of patient information contains only vital signs? A) Chief complaint of dizziness, blood pressure 110/76 mmHg, breath sounds clear and equal B) Heart rate 88, respiratory rate 14, blood glucose level 98 mg/dL C) Skin warm and dry, heart rate 74, pupils equal and reactive D) Chief complaint of dizziness, skin cool and clammy, respiratory rate 16

Answer: C - Skin warm and dry, heart rate 74, pupils equal and reactive

People living in this region of the United States are at greatest risk of Bubonic plague: A) Northeast—New York, Massachusetts, Maine B) Southeast—Georgia, Alabama, Mississippi C) Southwest—New Mexico, Colorado, Arizona D) Northwest—Oregon, Washington, Idaho

Answer: C - Southwest—New Mexico, Colorado, Arizona

All of the following would be appropriate care for a patient with non-traumatic back pain EXCEPT: A) Administration of analgesia prior to moving the patient B) Application of an ice pack to the affected area C) Spinal immobilization with careful padding of voids D) Administration of anti-inflammatory medications

Answer: C - Spinal immobilization with careful padding of voids

In addition to pneumonia, which of the following is a complication of COPD? A) Allergic reaction from beta2 medications B) Difficulty controlling blood sugar C) Spontaneous pneumothorax D) Deep vein thrombosis

Answer: C - Spontaneous pneumothorax

Medical direction has asked that you obtain orthostatic vital signs for a 54-year-old woman complaining of dizziness and weakness. Her baseline vitals obtained while she was sitting in a chair were: pulse 84, respirations 26, blood pressure 118/62 mmHg, and SpO2 95 percent. Which one of the following would indicate that the patient has a positive orthostatic test? A) Standing SpO2 88percent and heart rate of 96 beats per minute B) Standing pulse of 88 with the additional complaint of nausea C) Standing blood pressure of 92/54 and heart rate of 106 D) Supine blood pressure of 100/64 mmHg and respirations of 22

Answer: C - Standing blood pressure of 92/54 and heart rate of 106

You are treating victims of a motor vehicle collision that occurred when their vehicle struck a bridge abutment head on at a high rate of speed. Due to Newton's first law, you know that: A) It would be difficult to predict any injury due to the speed B) The vehicle itself would absorb all of the kinetic energy C) The kinetic energy of the moving vehicle would be transferred to the vehicle and victims D) Survival would be predicted on safety features of the vehicle

Answer: C - The kinetic energy of the moving vehicle would be transferred to the vehicle and victims

You are assessing a patient complaining of shortness of breath. Which of the following medications would indicate the patient has a history of respiratory disease? A) Phenobarbital B) Tregretol C) Theophylline D) Lithium

Answer: C - Theophylline

An unrestrained woman driving a small car is involved in a rollover-type collision. Why her risk for serious injury and death is significantly increased? A) Rollovers are the result of high speeds B) Smaller and lighter cars tend to roll over C) There are more impacts in a roll over D) The risk for ejection is lessened, increasing injuries suffered in the car

Answer: C - There are more impacts in a roll over

Your patient has type A+ blood. Which of the following statements about the patient's blood is TRUE? A) The patient produces anti-A antibodies. B) The patient can only receive A+ blood if a transfusion is needed. C) There are type A antigens on the surface of the red blood cells. D) The blood cells lack Rh antigens.

Answer: C - There are type A antigens on the surface of the red blood cells.

When transporting an unresponsive patient with a traumatic brain injury, which one of the following assessment findings would suggest that the patient is suffering from a herniation of the brain? A) The patient starts to vomit. B) The patient remains unresponsive. C) There is flexion of the arms when a shoulder is pinched. D) Both pupils are 4 mm and react to light.

Answer: C - There is flexion of the arms when a shoulder is pinched.

Life-threatening problems encountered during the primary assessment should be: A) Ignored while completing the assessment and then addressed B) Signify a poor outcome for the patient C) Treated immediately D) All of the above

Answer: C - Treated immediatel

The term "status epilepticus" refers to a: A) Chronic seizure patient taking anticonvulsant medication regularly. B) Generalized seizure lasting more than one minute. C) Two or more seizures with no intervening periods of consciousness. D) Patient experiencing a seizure for the first time.

Answer: C - Two or more seizures with no intervening periods of consciousness

An AEMT is correctly assessing a patient's radial pulse when she: A) Uses her thumb to feel for the pulse on the patient's lower arm B) Simultaneously checks for a heart rate on both sides of the neck C) Uses her fingertips to feel for a pulse at the patient's wrist D) Uses the palm of his hand to feel the pulse on the upper arm

Answer: C - Uses her fingertips to feel for a pulse at the patient's wrist

In which step of the clotting process does the smooth muscle contract, reducing the lumen and strength of blood flow through the vessel? A) Platelet phase B) Aggregate phase C) Vascular phase D) Coagulation phase

Answer: C - Vascular phase

The MOST important aspect of care when treating a suspected thoracic injury is: A) Chest decompression B) Pain medications C) Ventilation and oxygenation D) Intravenous therapy

Answer: C - Ventilation and oxygenation

Which one of the following patients requires the AEMT to remove an impaled object in the field? A) Woman with a knife to the right side of her chest; she is short of breath and coughing up blood B) Male patient who fell on a screwdriver, which is impaled through his hand; the patient is in pain and requesting that you remove the screwdriver C) Young woman with a broken pencil through her cheek and into the oropharynx; blood from the injury is trickling into her airway D) Male patient with a six-inch knife to his left upper quadrant; there is active bleeding around the injury site

Answer: C - Young woman with a broken pencil through her cheek and into the oropharynx; blood from the injury is trickling into her airway

The external ear ends at the ________. A) Auricle B) External auditory canal C) Yympanic membrane D) Tragus

Answer: C - Yympanic membrane

The area in a burn surrounding the zone of coagulation and that is characterized by decreased blood flow is the ________. A) Zone of coagulation B) Zone of hyperemia C) Zone of stasis D) Fluid shift zone

Answer: C - Zone of stasis

The bone MOST commonly used for intraosseous access is the: A) Fibula B) Femur C) Tibia D) Sternum

Answer: C -Tibia

A problem with the eighth cranial nerve can lead to which of the following disorders? A) Photophobia B) Meningismus C) Vertigo D) Macrotia

Answer: C -Vertigo

Pain felt by the patient when pressure from palpation is released from the area is referred to as: A) Point tenderness B) Kehr's sign C) Rebound tenderness D) Murphy's sign

Answer: C- Rebound tenderness

During an in-service focusing on care of the trauma patient, the medical director asks if anyone can correctly describe the "platinum 10 minutes." Which of the following indicates the BEST response? A) "EMS should spend a total of 10 minutes assessing the patient for life-threatening injuries prior to initiating transport to a trauma center." B) "If the transport to a trauma center is going to exceed 10 minutes, EMS should strongly consider the use of an aeromedical helicopter." C) "EMS systems should be designed so that it takes an ambulance no longer than 10 minutes to respond to a trauma call." D) "EMS should initiate transport of the critically injured trauma patient to the hospital within 10 minutes of arriving on scene."

Answer: D - "EMS should initiate transport of the critically injured trauma patient to the hospital within 10 minutes of arriving on scene."

Which one of the following illustrates a question that would be asked when obtaining a medical history, using the SAMPLE mnemonic, for a female patient who is crying and complaining of dizziness? A) "You seem upset. Do you want to talk about it?" B) "Who is your doctor and when was your last office visit?" C) "Why exactly did you call the ambulance today?" D) "What were you doing when the dizziness started?"

Answer: D - "What were you doing when the dizziness started?"

For which one of the following patients involved in a motor vehicle collision is the use of a vest-type short immobilization device indicated? A) 15-year-old girl complaining of neck and back pain who was self-extricated and is standing next to the car B) 25-year-old restrained driver who is unresponsive and has a history of diabetes C) 41-year-old man who was ejected and is lying supine in the roadway complaining of back pain D) 33-year-old woman in the backseat who states that her neck hurts and she has a headache

Answer: D - 33-year-old woman in the backseat who states that her neck hurts and she has a headache

Which of the following patients demonstrates clearly that they can maintain their own airway and breathing? A) An 18-year-old man vomiting in his sleep after drinking too much at a party. B) A 62-year-old woman with shortness of breath from an acute onset of pulmonary edema. C) A 28-year-old man who is hypotensive and is complaining of palpitations. D) A 42-year-old man who appropriately answers all of your questions with full sentences.

Answer: D - A 42-year-old man who appropriately answers all of your questions with full sentences.

Which of the following would be MOST likely to present with atypical symptoms of acute coronary syndrome? A) A 56-year-old with asthma B) A 72-year-old man C) A 32-year-old woman D) A 68-year-old diabetic

Answer: D - A 68-year-old diabetic

You are called to the local school playground for a six-year-old who fell off the teeter-totter. Her only complaint is pain to her right knee. Following your initial assessment, you should perform: A) A rapid secondary sssessment B) A detailed physical exam C) A reassessment D) A modified secondary assessment

Answer: D - A modified secondary assessment

You respond to a patient with shortness of breath, confusion, and a SpO2 88percent. Which of the following would support your suspicion of asthma? A) Stridor on inspiration and expiration B) Subcutaneous emphysema C) Rhonchi in all lung fields D) A non-productive nocturnal cough

Answer: D - A non-productive nocturnal cough

Which of the following is TRUE regarding a transient ischemic attack? A) Signs and symptoms are a little different from a stroke. B) If it doesn't resolve within two hours, it is a full blown stroke. C) If not promptly treated, a TIA will progress to a stroke. . D) A patient who has had a TIA is at high risk for subsequent stroke.

Answer: D - A patient who has had a TIA is at high risk for subsequent stroke.

Which of the following mechanisms of injury would MOST likely cause a deceleration injury resulting in a pulmonary contusion? A) A patient receives a bullet wound from a .22 gauge shotgun B) A patient is struck with a baseball bat C) A patient receives a knife wound to the thorax D) A patient's thorax strikes a steering wheel

Answer: D - A patient's thorax strikes a steering wheel

Following the primary assessment of an injured soccer player, you note that the contour of the extended and injured leg drops at the knee. Assessment of the distal perfusion reveals a present pulse at the level of the malleolus. You suspect ________ and will treat by ________. A) An anterior dislocation of the hip; applying gentle traction in an attempt to reduce the deformity B) A posterior dislocation of the hip; immobilizing the affected limb in position found C) An anterior dislocation of the knee; applying gentle traction in an attempt to reduce the deformity D) A posterior dislocation of the knee; immobilizing the affected limb in position found

Answer: D - A posterior dislocation of the knee; immobilizing the affected limb in position found

Which of the following would be an example of acquired active immunity? A) Immunity passed from mother to baby during breastfeeding. B) A person is immune to varicella after having chicken pox as a child. C) Immunity formed after repeated exposure to a pathogen. D) A woman takes a yearly flu shot to prevent contracting influenza.

Answer: D - A woman takes a yearly flu shot to prevent contracting influenza.

Which of the following is the hallmark of anemia? A) Misshapen red blood cells B) Decreased WBC count C) Decrease in clotting factors D) Abnormally low hematocrit

Answer: D - Abnormally low hematocrit

Prior to and after applying a splint, the AEMT should assess: A) Neurological status of affected extremity. B) ABCs. C) Distal circulatory status. D) All of the above

Answer: D - All of the above

Your patient is a 28-year-old man who is found unconscious, breathing deeply and rapidly with a weak, rapid pulse. The patient's skin is warm with dry mucous membranes and you note a fruity odor to the breath. Treatment for this patient would include: A) Isotonic IV fluid administration. B) High-flow oxygen by mask. C) Possible dextrose administration. D) All of the above

Answer: D - All of the above

Which of the following is a phase of the medication administration process that is particularly error-prone? A) Giving the drug order B) Transcribing (interpreting) the order C) Documentation D) All of the above are error-prone

Answer: D - All of the above are error-prone

Which of the following statements are TRUE regarding pneumatic anti-shock garments (PASG)? A) Pneumatic anti-shock garments (PASG) are trousers with inflatable compartments B) Research on pneumatic anti-shock garments (PASG) indicated that they do not improve survival C) Pneumatic anti-shock garments (PASG) may complicate some conditions and their application can interfere with treatments that should have a higher priority in the management of shock D) All of the above are true statements

Answer: D - All of the above are true statements

Which of the following patients is end-tidal carbon dioxide (EtCO2) monitoring in the prehospital setting indicated for? A) Patients with respiratory complaints B) Critically ill patients C) Patients who are intubated D) All of the above patients need EtCO2 monitoring

Answer: D - All of the above patients need EtCO2 monitoring

You are treating an unresponsive diabetic patient. You have administered 25 grams of 50 percent dextrose. Which of the following is MOST important that you recheck and document on your run report? A) That your IV site was patent when dextrose was delivered. B) Results of repeated blood glucose check. C) Any change in mental status after drug administration. D) All of the above should be reassessed and documented.

Answer: D - All of the above should be reassessed and documented.

All of the following are indications for intravenous access EXCEPT: A) Drug administration B) Obtaining venous blood specimens C) Fluid and blood replacement D) Allergy testing

Answer: D - Allergy testing

The detection of a pulsating mass upon palpation of a patient's abdomen should make the EMT suspicious that the patient may be suffering from which of the following? A) A hernia B) An ulcer C) Gastroenteritis D) An abdominal aortic aneurysm

Answer: D - An abdominal aortic aneurysm

Which of the following patients are MOST at risk for nosocomial infections? A) A child with a broken arm B) A patient having a heart attack C) A woman having a TIA D) An elderly AIDS patient

Answer: D - An elderly AIDS patient

Wet gangrene, so called because the infected area oozes foul smelling liquid, occurs as a result of: A) Group A hemolytic streptococci. B) Pressure ischemia. C) Stevens-Johnson syndrome. D) An untreated infection.

Answer: D - An untreated infection.

Your elderly patient has fallen from the curb and complains of hip, leg, and back pain. You observe the foot of the injured limb turned outward. Palpation of the pelvis reveals the head of the femur in the inguinal area. You suspect the patient is suffering from ________, so your management will include ________. A) Anterior hip dislocation; reduction of the injured hip and transport to an emergency department B) Posterior hip dislocation; reduction of the injured hip and transport to an acute care facility C) Posterior hip dislocation; reduction of the injured hip and transport to a trauma center D) Anterior hip dislocation; stabilization of the injured hip and transport to an emergency department

Answer: D - Anterior hip dislocation; stabilization of the injured hip and transport to an emergency department

The minimum aseptic technique for intramuscular and subcutaneous injection and for intravenous access and intravenous injection through a medication administration port on intravenous tubing would be: A) Blowing or fanning on the skin to dry it B) Touching the site after it has been cleaned C) Puncturing the skin through wet alcohol D) Application of friction with an isopropyl alcohol wipe

Answer: D - Application of friction with an isopropyl alcohol wipe

When should high-flow oxygen be provided to a significant head trauma patient with diminished orientation? A) If there is concurrent airway compromise B) If the patient is not breathing adequately C) If the patient is to be ventilated mechanically D) At all times, regardless of method of delivery

Answer: D - At all times, regardless of method of delivery

The reassessment in the critically injured trauma patient should be done: A) Every minute B) Every 15 minutes C) Once on the way to the hospital D) At least every five minutes

Answer: D - At least every five minutes

The AEMT recognizes that the division between the thoracic and abdominal cavities lies approximately: A) At the level of the umbilicus B) Above the twelfth thoracic vertebrae C) Below the twelfth rib D) At the fifth intercostal space

Answer: D - At the fifth intercostal space

Which of the following cardiac dysrhythmias can lead to the formation of blood clots which increases the risk for stroke? A) Asystole B) Sinus arrhythmia C) Ventricular tachycardia D) Atrial fibrillation

Answer: D - Atrial fibrillation

What parasympatholytic can be used to treat organophosphate poisoning? A) Amiodarone B) Magnesium sulfate C) Epinephrine D) Atropine

Answer: D - Atropine

You have been called for a 25-year-old man who is experiencing difficulty breathing and sharp chest pain after moving some heavy furniture. The pain is located on the right side of the chest and seems to worsen on deep inspiration. With this information, you immediately suspect pneumothorax. Which of the following should you perform next to help confirm your suspicion? A) Inquire about past lung problems B) Obtain and SpO2 reading C) Palpate for subcutaneous emphysema D) Auscultate all lung fields

Answer: D - Auscultate all lung fields

________ is a major cause of death in traumatic injuries. A) Head injury B) Internal abdominal bleeding C) Hypovolemic shock D) Blunt thoracic trauma

Answer: D - Blunt thoracic trauma

Drugs for inhalation that are gases administered by the AEMT would include which of the following? A) Oxygen B) Nitrous oxide C) Carbon dioxide D) Both A and B

Answer: D - Both A and B

What is the only organ in the body that does NOT require insulin in order for glucose to enter its cells? A) Liver B) Pancreas C) Heart D) Brain

Answer: D - Brain

Which of the following is TRUE regarding glucose use by the cells? A) The brain can convert to using fats for energy. B) The brain requires insulin to absorb glucose. C) Glycogen is stored in the pancreas for cellular use. D) Brain cells require glucose to function.

Answer: D - Brain cells require glucose to function.

What is the response in the lungs to beta2 stimulation? A) Bronchoconstriction B) Decreased respiratory rate C) Increased heart rate D) Bronchodilation

Answer: D - Bronchodilation

You are assessing a patient involved in a motorcycle accident. He states that he saw an oncoming car and "laid the bike down" to avoid an outright collision. He was wearing a helmet. Which of the following injuries would make sense given the action of the motorcyclist? A) "Road rash" to the entire body B) Bilateral broken forearms C) Angulation to both femurs D) Burn to the inside of the leg

Answer: D - Burn to the inside of the leg

While the clavicle is the MOST frequently fractured bone in the human body, it: A) Is unlikely to cause serious internal injury B) Cannot be managed in a prehospital setting C) Can be managed by immobilizing the distal extremity using an air or rigid splint D) Can be managed by immobilizing the affected limb using a sling and swathe

Answer: D - Can be managed by immobilizing the affected limb using a sling and swathe

The connection points between the arterial and venous systems are called: A) Lumens. B) Venules. C) Tunica. D) Capillaries.

Answer: D - Capillaries.

You are dispatched to a residence where a three-year-old boy presents with a fever. His parents state that he has had diarrhea and been vomiting for two days. The patient has not eaten in 24 hours. To BEST assess his peripheral perfusion status, you should evaluate: A) Fingertip sensation B) Respiratory rate C) Blood pressure D) Capillary refill

Answer: D - Capillary refill

You are assessing an elderly patient with a decreased level of consciousness. Your assessment reveals the patient to have a patent airway, labored respirations, and weak, rapid pulses. The skin is pale, cool, and cyanotic in the extremities. You also observe diaphoresis and a delayed capillary refill. Vital signs for this patient are: heart rate of 136 bpm, blood pressure of 66/40 mmHg, and respirations 40 and shallow. Auscultation of the lungs reveals profound crackles located throughout each lung. His temperature is 99°F. There is obvious JVD and pedal edema. Additionally, family states that the patient has an extensive cardiac and diabetic history. Based on this information you should suspect what kind of shock? A) Hypotensive B) Distributive C) Hypovolemic D) Cardiogenic

Answer: D - Cardiogenic

You have responded for a 65-year-old man with chest pain. You note that his blood pressure is 88/62 and his heart rate is 136. Which of the following should you suspect? A) Angina pectoris B) Pain and anxiety C) Cardiac tamponade D) Cardiogenic shock

Answer: D - Cardiogenic shock

You have arrived on the scene of a 44-year-old man in cardiac arrest. According to family, the patient has a cardiac history and was complaining of chest pain most of the morning. They state that it has been about 10 minutes since he collapsed. To maximize the patient's opportunity for a successful outcome, which one of the following is the MOST important intervention that the EMT can make immediately? A) Advanced drugs by paramedics B) Defibrillation with the AED C) Oxygen through a nonrebreather mask D) Cardiopulmonary resuscitation

Answer: D - Cardiopulmonary resuscitation

Which of the following BEST describes prerenal renal failure? A) Caused by outflow obstructions that back up the urine B) Caused by diseases in the kidneys C) Caused by prostate enlargement D) Caused by disease elsewhere in the body

Answer: D - Caused by disease elsewhere in the body

Which type of burn denatures the biochemical makeup of cell membranes and destroys the cell? A) Electrical B) Thermal C) Radiation D) Chemical

Answer: D - Chemical

The AEMT should consider aspirin for the patient with which one of the following conditions? A) Fever B) Shortness of breath C) Headache D) Chest discomfort

Answer: D - Chest discomfort

When assessing a patient, which of the following should you recognize as an indication for aspirin therapy? A) A complaint of a headache after taking nitroglycerin. B) The patient takes an aspirin a day to prevent heart attacks. C) A blood pressure above 100 mmHg systolically. D) Chest discomfort that is suggestive of a heart attack.

Answer: D - Chest discomfort that is suggestive of a heart attack.

A series of one-sided headaches that are sudden and intense and that may continue for 15 minutes to four hours is referred to as: A) Syncope headaches. B) Organic headaches. C) Migraine headaches. D) Cluster headaches.

Answer: D - Cluster headaches.

An elderly patient has fallen down a flight of stairs and is complaining of neck and back pain as well as weakness to both legs. The primary assessment reveals no life threats to the airway, breathing, or circulation. Manual inline spinal stabilization is being maintained. Which of the following should the AEMT do next? A) Apply high-flow oxygen and move to the stretcher for transport B) Place a cervical collar and immobilize to the long spine board C) Place an oral airway and begin positive pressure ventilation D) Complete the secondary assessment looking for injuries

Answer: D - Complete the secondary assessment looking for injuries

A male soccer player was struck in the head with a soccer ball. Players state that he was dazed for several seconds following the impact and then asked the same questions over and over. Presently, he is conscious and oriented to person, but confused to place and time. He also has a reddened area to the side of his head and face. As you proceed with your assessment, his memory continues to improve. Based on these findings, the AEMT should suspect which one of the following injuries? A) Cerebral contusion B) Open head injury C) Epidural hematoma D) Concussion

Answer: D - Concussion

A defect of the ________ results in difficulty in visual detection of color and fine detail. A) Lens B) Cones and rods C) Rods D) Cones

Answer: D - Cones

An inflammation of the thin, transparent membrane covering the visible portion of the sclera and lining the inside of the eyelids results in ________. A) Corneal abrasion B) Lens distortion C) Choroiditis D) Conjunctivitis

Answer: D - Conjunctivitis

Cholecystitis often occurs after: A) Severe vomiting. B) Use of NSAIDs. C) Prolonged period of constipation. D) Consuming a meal high in fat.

Answer: D - Consuming a meal high in fat.

Which of the following would be MOST appropriate for an AEMT to provide for a patient with an acute onset of pulmonary edema with severe respiratory distress? A) Immediate tracheal intubation B) IV administration of nitroglycerin C) Administration of a beta2-agonist D) Continuous positive airway pressure (CPAP)

Answer: D - Continuous positive airway pressure (CPAP)

Your patient had a sudden deceleration injury and struck the windshield with his head. As a result of the sudden stoppage, you suspect that the patient has suffered a(n): A) Vascular lesion. B) Comminuted injury. C) Open brain injury. D) Coup-contrecoup injury.

Answer: D - Coup-contrecoup injury

You are on the scene of a shooting. Your assessment reveals a 23-year-old man who has been shot twice. The first wound is to the left lower quadrant of the abdomen and is actively bleeding. The second wound is to the left lateral chest and makes a sucking sound every time the patient takes a breath. The initial action of the AEMT should be which one of the following? A) Place direct pressure over the abdominal gunshot wound B) Place the patient on high-flow oxygen with a nonrebreather fire mask C) Obtain a sterile dressing and cover the chest wound D) Cover the chest wound with a gloved hand

Answer: D - Cover the chest wound with a gloved hand

A patient who was hit in the face with a cloud of dust while working in an industrial setting is complaining of pain and discomfort to his left eye. While performing your secondary assessment on the eye, you note some redness to the globe but do not see any obvious foreign object. The most appropriate care you can provide to this patient would be: A) Gently massaging the eye to promote tearing B) Having the patient hold the eye open during transport C) Carefully wiping the eye with a soft piece of sterile gauze D) Covering both eyes with a bandage

Answer: D - Covering both eyes with a bandage

While auscultating the chest, you hear sounds that sound like the initial "fizzing" sounds after a can of soda is opened. This is indicative of: A) Normal lung sounds B) Atelectasis C) A hypoinflated lung D) Crackles

Answer: D - Crackles

Which of the following is the main focus of your assessment and history taking of the patient with abdominal pain? A) Determining the possible need for immediate surgery B) Determining if the patient meets criteria to refuse treatment and transport C) Determining the cause of the pain D) Determining the presence of shock

Answer: D - Determining the presence of shock

A needle's gauge describes its: A) Sharpness B) Length C) Volume D) Diameter

Answer: D - Diameter

Atrovent, if prescribed to the patient, is indicated for which one of the following conditions? A) Chest discomfort B) Nausea or vomiting C) Headache D) Difficulty breathing

Answer: D - Difficulty breathing

As a member of the emergency medical services team, an AEMT's role includes all of the following EXCEPT to: A) Provide rapid assessment B) Transport patients to the appropriate hospital C) Triage trauma patients D) Dispatch equipment

Answer: D - Dispatch equipment

You are treating a patient with an abdominal evisceration. You should: A) Replace the protruding organs, cover with a dry sterile dressing, and an occlusive dressing B) Do not replace the protruding organs, cover with a dry sterile dressing, and an occlusive dressing C) Replace the protruding organs, cover with a sterile dressing moistened with sterile saline, and an occlusive dressing D) Do not replace the protruding organs, cover with a sterile dressing moistened with sterile saline, and an occlusive dressing

Answer: D - Do not replace the protruding organs, cover with a sterile dressing moistened with sterile saline, and an occlusive dressing

When treating a patient with a musculoskeletal injury who also has soft-tissue involvement, the AEMT should: A) Rapidly splint the injury and bandage the wound to prevent infection B) Splint the injury leaving the wound available for dressing and bandaging C) Cover an open wound with the splint to assist bleeding control D) Dress and bandage the open wound prior to splinting

Answer: D - Dress and bandage the open wound prior to splinting

You are caring for a young girl who is complaining of abdominal cramps. During you assessment, she tells you that she usually have very painful cramps during her monthly menstrual period. Which of the following would be the CORRECT terminology to use to document her symptoms? A) Endometritis B) Menorrhagia C) Paraphimosis D) Dysmenorrhea

Answer: D - Dysmenorrhea

During transport of a patient with suspected head injury, the AEMT may: A) Hyperventilate the patient. B) Administer large volumes of fluid to combat acidosis. C) Administer dextrose for LOC. D) Elevate the head of the spine board.

Answer: D - Elevate the head of the spine board

You are called for a four-year-old girl who apparently has suffered a seizure. Her mother states that she has been sick for a day or two. Which kind of seizure has she MOST likely had? A) Focal motor seizure B) Petit mal seizure C) Epileptic seizure D) Febrile seizure

Answer: D - Febrile seizure

Which one of the pulses listed below is palpated in the groin? A) Pedal B) Inguinal C) Carotid D) Femoral

Answer: D - Femoral

Your 56-year-old female patient is complaining of chronic fatigue, muscle stiffness, and tender areas on each elbow. Based on your assessment and the patient's history, you should suspect: A) Rhabdomyolysis. B) Osteoporosis. C) Osteoarthritis. D) Fibromyalgia.

Answer: D - Fibromyalgia.

The sympathetic nervous system is also known as the ______ division. A) Feed or breed B) Slow or go C) Stand or draw D) Fight or flight

Answer: D - Fight Or Flight

To test a patient's capillary refill, the AEMT will: A) Apply gentle pressure to the radial pulse and watch color changes to the hand B) Apply pressure and then release that pressure from the arm C) Examine the inside lining of one or both eyelids D) Firmly compress and then release pressure on the nail bed

Answer: D - Firmly compress and then release pressure on the nail bed

Your patient has come into contact with a strong acid that has created an open wound into the abdomen. You should classify this would as a: A) Superficial burn B) Minor injury C) Partial-thickness burn D) Full-thickness burn

Answer: D - Full-thickness burn

You respond, along with fire department medical responders, to a 48-year-old woman having a syncopal episode in the bathroom. You find the patient sitting on the commode vomiting into the trashcan. The vomitus appears to look like coffee grounds and has a foul smell. The patient is pale and has been weak for the past few days. She MOST likely has: A) Peritonitis. B) Abdominal aortic aneurysm. C) Hernia. D) GI bleeding.

Answer: D - GI bleeding.

A patient has an advanced infection in his foot. There appears to be an accumulation of gas in the tissues which causes a crackling sensation on palpation. This sign is typically seen in: A) Dry gangrene. B) Cellulitis. C) Necrotizing fasciitis. D) Gas gangrene.

Answer: D - Gas gangrene.

A patient presents with extremely pale skin. His family states that this is NOT his normal skin color. Which one of the following medical conditions would make sense given this finding? A) Respiratory infection B) Body temperature of 102.5°F C) Chest pain accompanied by a bounding pulse D) Gastrointestinal hemorrhage

Answer: D - Gastrointestinal hemorrhage

At bare minimum, what Standard Precautions should be used when starting an IV? A) Gloves and eye protection B) Gloves, eye protection, and mask C) Gloves, eye protection, mask, and gown D) Gloves

Answer: D - Gloves

A naturally occurring hormone that promotes the breakdown of glycogen in the liver to glucose to increase blood glucose levels would be: A) Oral glucose B) Dextrose 50 percent C) Epinephrine D) Glucagon

Answer: D - Glucagon

If insulin production and secretion are insufficient for metabolic needs: A) Glucose will enter the cells more efficiently. B) Glucose will rapidly decrease in the bloodstream. C) Metabolism will not be affected. D) Glucose will accumulate in the bloodstream.

Answer: D - Glucose will accumulate in the bloodstream

When choosing a site for routine intravenous access, the BEST place to start is in the: A) Antecubital fossa and work towards the hand B) Neck C) Thigh D) Hand and work towards the antecubital fossa

Answer: D - Hand and work towards the antcubital fossa

Which of the following is CORRECT regarding Tularemia? A) It is a gastrointestinal illness spread through food. B) It causes hemolytic uremic syndrome if left untreated. C) It is a bacterial infection transmitted by black-legged ticks. D) Has been identified as a potential weapon of bioterrorism.

Answer: D - Has been identified as a potential weapon of bioterrorism.

A young male golfer has been struck in the right eye with a golf club. The upper and lower lids are swollen shut and ecchymotic. There are also some blood clots noted between the two eyelids. Which of the following indicates that the AEMT is providing proper care to this patient? A) He places a numbing medication in the eye. B) He irrigates blood from the eye with sterile saline solution. C) He gently opens the lids to assess for injury to the eye. D) He wipes blood from the face but not the eye.

Answer: D - He wipes blood from the face but not the eye.

Which of the following statements regarding hemophilia is MOST accurate? A) Hemophilia is a hereditary abnormality of the platelets. B) Hemophilia is a hereditary disease in which the body is unable to produce vitamin K. C) Hemophilia is a hereditary disease that causes fibrin clots to dissolve prematurely. D) Hemophilia is a hereditary lack of certain proteins needed in the clotting cascade.

Answer: D - Hemophilia is a hereditary lack of certain proteins needed in the clotting cascade.

A patient suffering blunt trauma to the abdomen has a lacerated liver. Which one of the following represents the MOST immediate threat to life? A) Pain B) Infection C) Peritonitis D) Hemorrhage

Answer: D - Hemorrhage

One sign of _________ is asterixis, a condition in which the patient's hands involuntary "flap" when he holds his hands up with the wrists flexed, such as signaling someone to stop. A) Esophageal varices B) Cirrhosis C) Cholecystitis D) Hepatic encephalopathy

Answer: D - Hepatic encephalopathy

Acetaminophen is generally well tolerated and there are no significant side effects in therapeutic doses, however in large doses, the medication can be: A) Neurotoxic B) Gastrotoxic C) Nephrotoxic D) Hepatotoxic

Answer: D - Hepatoxic

Which of the following pathogens can lie dormant in the body but reemerge under times of stress? A) Hepatitis B) Fungi C) Giardia lamblia D) Herpes viruses

Answer: D - Herpes viruses

A 57-year-old man fell on the sidewalk, hitting his head on the concrete. According to witnesses, the patient was unresponsive for several minutes following the fall. Which one of the following would be MOST critical to relay to the physician in the emergency department? A) History of depression B) Knee replacement two years ago C) Position in which the patient was found D) History of alcoholism

Answer: D - History of alcoholism

Which of the following is a risk factor for hemorrhagic stroke? A) Atrial fibrillation B) Hypoglycemia C) Spinal injury D) Hypertension

Answer: D - Hypertension

The signs and symptoms of stroke may closely resemble those of what other condition? A) Narcotic overdose B) Syncope C) Stroke D) Hypoglycemia

Answer: D - Hypoglycemia

Which of the following is TRUE regarding blood glucose levels and stroke? A) Glucose requires insulin to enter brain cells. B) Excessive glucose in cells causes cellular dehydration. C) Hyperglycemia is protective of brain cells. D) Hypoglycemia worsens neurological outcome.

Answer: D - Hypoglycemia worsens neurological outcome.

Crystalloid solutions may be _______, _______, or ________ with respect to body fluids. A) Hypothermic, isothermic, hyperthermic B) Hypoglycemic, isoglycemic, hyperglycemic C) Hypolateral, isolateral, hyperlateral D) Hypotonic, isotonic, hypertonic

Answer: D - Hypotonic, isotonic, hypertonic

Increasing the resistance against which the heart MUST pump blood will directly: A) Decrease blood pressure B) Decrease heart rate C) Increase heart rate D) Increase blood pressure

Answer: D - Increase blood pressure

Which one of the following statements describes the benefit of using the pneumatic antishock garment (PASG) to splint a potential pelvic fracture? A) It increases blood flow into both the pelvis and distal leg B) In addition to the pelvis, it immobilizes the thoracic and lumbar spine C) It applies even pressure to the pelvis, forcing the pelvic bones to realign D) It can effectively stabilize the pelvis and help to control internal bleeding

Answer: D - It can effectively stabilize the pelvis and help to control internal bleeding

Which one of the following statements about the P wave is TRUE? A) It represents ventricular depolarization. B) It is a rounded wave appearing after the QRS complex. C) It follows the R wave on normal ECGs. D) It corresponds to atrial depolarization.

Answer: D - It corresponds to atrial depolarization.

Which of the following is TRUE concerning parietal pain? A) It is often described as "crampy" or "colicky." B) It arises from solid organs. C) It is usually intermittent in nature. D) It is generally localized to a particular area.

Answer: D - It is generally localized to a particular area.

Which of the following is CORRECT regarding peritoneal dialysis? A) A shunt graft surgically connects an artery and vein. B) Patients are treated at dialysis centers. C) Hypotension is a frequent side effect. D) It is less efficient than hemodialysis.

Answer: D - It is less efficient than hemodialysis.

You should suspect _________ when your patient has hematuria and sharp, colicky pain that radiates to the groin. A) Appendicitis B) Constipation C) Peritonitis D) Kidney stones

Answer: D - Kidney stones

Two basic principles of kinetics are: A) Velocity and mass B) Movement and energy C) Inertia and velocity D) Law of inertia and conservation of energy

Answer: D - Law of inertia and conservation of energy

An unresponsive patient has been ejected from a car as it rolled at a high rate of speed. The primary assessment is complete and you are providing positive pressure ventilation with high-flow oxygen. The patient has a significant deformity to the right side of his head, and both pupils are equal but slow to react to light. He also has deformity to the forearms. Vital signs are: pulse 144, respirations 24 and inadequate, and blood pressure 90/50 mmHg. Based on this presentation, the AEMT should: A) Apply the automated external defibrillator and prepare for possible seizure activity. B) Splint both arms and reassess the vital signs prior to departing the scene. C) Reassess the pupils and treat the patient for a severe head injury. D) Look for evidence of bleeding in an area of the body other than the brain.

Answer: D - Look for evidence of bleeding in an area of the body other than the brain.

You arrive on the scene to find a 21-year-old woman agonally breathing. Her friend states she is a habitual drug user and might have overdosed on heroin. What medication will help negate the effects of the drug and improve respirations? A) Romazicon B) Atrovent C) Albuterol D) Naloxone

Answer: D - Naloxone

The law which states that an object in motion will stay in motion unless acted upon by an outside source is: A) Conservation of energy law B) Energy formulation C) Starling's theorem D) Newton's first law

Answer: D - Newton's first law

Which IV fluid solution is essentially sterile water with sodium chloride (NaCl) added to equal the amount in the human body? A) D5W B) One-half normal saline (0.45 percent saline solution) C) Lactated Ringer's D) Normal saline (0.9 percent saline solution)

Answer: D - Normal saline (0.9 percent saline solution)

To determine specifics about chest pain, use ______ to help you. A) SAMPLE B) DCAP-BTLS C) AEIOU-TIPS D) OPQRST

Answer: D - OPQRST

Parenteral routes of drug administration in the AEMT scope of practice include all of the following EXCEPT: A) Inhalation B) Intravenous bolus C) Intramuscular injection D) Oral

Answer: D - Oral

What are the three tiny bones located in the middle ear? A) Pinnas B) Cerumen C) Cochleas D) Ossicles

Answer: D - Ossicles

A condition on the ovary where a ruptured follicle forms a fluid-filled sac is known as: A) Endometriosis. B) Cystitis. C) Dysmenorrhea. D) Ovarian cyst.

Answer: D - Ovarian cyst.

Asking a patient, "Are you being treated by a physician for anything?" represents what part of the SAMPLE question format? A) A B) M C) S D) P

Answer: D - P

Asking a patient, "Does it hurt more when you breathe deep?" represents what part of the OPQRST question format? A) Q B) O C) R D) P

Answer: D - P

A chest injury in which two or more ribs are fractured in two or more places causes the chest wall on the affected side to rise during expiration. This movement is known as: A) Flail segment B) Reduced intrathoracic pressure C) Diaphragm rupture D) Paradoxical movement

Answer: D - Paradoxical movement

The condition in which the foreskin is pulled down over the shaft of the penis, creating a small opening that can strangle the head of the penis in called: A) Orchitis. B) Phimosis. C) Epididymitis. D) Paraphimosis.

Answer: D - Paraphimosis.

While at a public pool obtaining a refusal from a patient who was stung by a bee, you hear screaming. Patrons are waving you over to a 13-year-old boy who hit the diving board with his head while diving and is now in the water motionless. The lifeguard is in the water and informs you that the patient's eyes are open and he is breathing with a pulse, but not talking. Which one of the following should you do next? A) Position the boy upright against the lifeguard with inline spinal precautions observed during removal from the pool B) Instruct the lifeguard to carefully float the boy to the side of the pool so you can assess and immobilize him C) Have the lifeguard quickly remove the patient from the pool so assessment and emergency medical care can be given D) Pass a cervical collar and long board into the water to immobilize the patient prior to extrication from the water

Answer: D - Pass a cervical collar and long board into the water to immobilize the patient prior to extrication from the water

Which of the following is a contraindication to administering isotonic crystalloid solutions such as lactated Ringer's and normal saline? A) Patients with traumatic brain injury B) Patients with stroke C) Patients which require fluid replacement D) Patients with congestive heart failure

Answer: D - Patients with congestive heart failure

A 26-year-old woman is complaining of abdominal pain. She states that the pain is generalized to her abdomen and worsens when walking. She also tells you that she is having a foul smelling yellow colored vaginal discharge. As such, you would treat her for what condition? A) Ectopic pregnancy B) Ovarian cyst C) Mittelschmerz D) Pelvic inflammatory disease

Answer: D - Pelvic inflammatory disease

In ________ the energy dissipation may create a permanent or temporary cavity transmitting to surrounding tissues. A) Deceleration forces B) Blunt trauma C) Contusions D) Penetrating trauma

Answer: D - Penetrating trauma

You have just arrived on the scene of a very serious motor-vehicle collision. EMRs have rapidly extricated an unresponsive female from the driver's seat. The EMRs report that she was unrestrained and struck the steering wheel with her chest and abdomen. As you start the primary assessment, you note that she has snoring respirations and is breathing shallowly at a rate of 24 breaths per minute. Which one of the following should you do next? A) Obtain her vital signs B) Start positive pressure ventilation C) Immobilize her to the backboard D) Perform a jaw-thrust maneuver

Answer: D - Perform a jaw-thrust maneuver

You suspect that a patient is dehydrated. When checking for skin turgor, you should: A) Palpate the skin for dryness B) Press the nail beds, then release and observe for blanching C) Use the backs of the fingers to determine temperature D) Pick up a fold of skin and then release it, looking for tenting

Answer: D - Pick up a fold of skin and then release it, looking for tenting

Petechiae is recognizable as: A) Bruising of the skin over the joints. B) Small, blotchy areas of hemorrhage. C) A collection of blood in the tissues. D) Pinpoint hemorrhages in the skin.

Answer: D - Pinpoint hemorrhages in the skin.

A seven-year-old boy choked on a grape at school. By the time the obstruction was removed, assessment revealed him to be in cardiac arrest. When you arrive, teachers are performing CPR. The school nurse informs you that CPR has been in progress for six minutes. You have an AED, but do not have a pediatric conversion device that reduces the energy of defibrillation from that of an adult to that of a child. Which one of the following should you do immediately? A) Instruct the teachers to continue CPR for another minute before applying the AED B) Continue CPR and transfer the boy to the stretcher for immediate transport C) Perform five abdominal thrusts and then one minute of CPR before using the AED D) Place the electrodes on the boy's chest and follow the AED's prompts

Answer: D - Place the electrodes on the boy's chest and follow the AED's prompts

You are dispatched to a house fire where a 67-year-old male patient has been removed from the structure. The patient is alert and oriented, with obvious burns to the hands. His respirations are 24, pulse 110, and the blood pressure 130/84. He states that the fire started in his television set and that he tried to put it out but could not. As he speaks, hoarseness is developing and blisters are starting to form on his hands. Your major concern with this patient is: A) Electrical conduction injury B) Treatment for shock C) Infection control D) Possible toxic inhalation

Answer: D - Possible toxic inhalation

You arrive on the scene of a seizure patient. In what state will you MOST often encounter the seizure patient? A) Aura phase B) Tonic phase C) Clonic phase D) Postictal phase

Answer: D - Postictal phase

Your patient is a 61-year-old man complaining of sudden onset midsternal chest pain. His color is ashen and he is diaphoretic. He takes an aspirin daily as well as medication for hypertension. His blood pressure is 100/60. His heart rate is 110. Respiration rate is 26. Pulse oximeter reads 96 percent on room air. Which of the following BEST describes side effects you could see from nitroglycerin administration? A) Anti-ischemic effects B) An increase in preload C) Bradycardia from vasodilation D) Postural hypotension

Answer: D - Postural hypotension

A teenage boy was found by friends as he attempted to hang himself in a garage using chains suspended from the ceiling. As you approach, you note that the patient is conscious, struggling to breathe, and has contusions from the chains to both the anterior and posterior portions of the neck. After taking inline spinal stabilization and opening the airway, the AEMT's next action should be to: A) Logroll him onto a long board B) Size and apply a cervical collar C) Perform a secondary assessment D) Provide positive pressure ventilation

Answer: D - Provide positive pressure ventilation

Asking a patient, "Where does it hurt?" represents what part of the OPQRST question format? A) P B) Q C) O D) R

Answer: D - R

The signs and symptoms related to hypoglycemia have a(n) ________ onset. A) Slow B) Unpredictable C) Delayed D) Rapid

Answer: D - Rapid

Which of the following is TRUE regarding capillaries? A) They consist of three layers or coats. B) The thinnest layer is the tunica adventitia. C) They contain valves to limit back flow of blood. D) Red blood cells move through in single file.

Answer: D - Red blood cells move through in single file.

While gathering a history from a 65-year-old man, you note that he takes a beta blocker. You know this is a cardiac medication which will: A) Cause smooth muscle in the heart to contract. B) Block the beta receptors of the parasympathetic nervous system C) Interrupt the signals to the vagus nerve. D) Reduce the rate and force of cardiac contractions.

Answer: D - Reduce the rate and force of cardiac contractions.

A motorcyclist wearing a full helmet was thrown from his motorcycle after hitting a patch of oil. The bike had been traveling at a high rate of speed. Manual inline spinal stabilization is being held by an off-duty AEMT. The primary assessment reveals the patient to be responsive to painful stimuli and breathing rapidly. His radial pulse is weak and fast. As you quickly scan his body, you note deformity to the left femur and lower leg. Your immediate action should be to: A) Place a nonrebreather face mask B) Expose his left lower extremity C) Apply a cervical collar D) Remove his helmet

Answer: D - Remove his helmet

You are treating the victim of a shooting who has an open chest wound that has been covered by an occlusive dressing. You notice the patient is now anxious, short of breath, and tachycardic. Your next action should be: A) Insert a supraglottic airway rapidly B) Defibrillate at 200J if heart rate exceeds 200 bpm C) Establish large-bore IVs and administer fluid D) Remove the occlusive dressing

Answer: D - Remove the occlusive dressing

Mrs. Baker fell in her home and lay immobile until the next day when her daughter found her lying on the tile floor. You recognize that Mrs. Baker is at a greatly increased risk of: A) Hypokalemia. B) Fibromyalgia. C) Renal failure. D) Rhabdomyolysis.

Answer: D - Rhabdomyolysis.

If the physician told you that the patient had a cold, you could document that in your patient care report as: A) Laryngitis. B) Pharyngitis. C) Sinusitis. D) Rhinovirus.

Answer: D - Rhinovirus.

The sensor receptive cells in the retina that function BEST in dim light and enable night vision are the ________. A) Iris B) Cones C) Pupil D) Rods

Answer: D - Rods

Which one of the following pieces of information would be obtained when getting a history using the SAMPLE mnemonic? A) Reason for calling for an ambulance B) No life threats to the airway, breathing, or circulation C) Pulse 144, respiration 16, blood pressure 132/88 D) Sensation of dizziness prior to falling

Answer: D - Sensation of dizziness prior to falling

The body's transition between normal function and death is called: A) Exsanguination B) Homeostasis C) Hemorrhage D) Shock

Answer: D - Shock

A patient with a history of COPD experiences a sudden onset of sharp chest pain. Your exam reveals an anxious patient who is breathing rapidly, and has diminished breath sounds to the left lower lobe of the lung and midline trachea. The patient has MOST likely experienced a: A) Sucking chest wound B) Flail segment C) Tension pnuemothorax D) Simple pnuemothorax

Answer: D - Simple pnuemothorax

When performing the primary assessment, which one of the following signs or symptoms BEST shows that the patient is in shock? A) Radial pulse of 72 beats per minute B) Vomitus in the airway C) Crackles heard in both lungs D) Skin that is cool and diaphoretic

Answer: D - Skin that is cool and diaphoretic

On-scene, you suspect a young female patient is suffering from hyperventilation syndrome. Which one of the following signs would BEST assist you in confirming your suspicion? A) Heart rate of 62 beats per minute B) Urinary incontinence C) Breathing through the mouth D) Spasm of the hands

Answer: D - Spasm of the hands

A male patient fell 20 feet from a cliff to a trail below. The primary assessment shows him to be confused, with an open airway and shallow breathing. His pulse is 72 beats per minute and his blood pressure is 78/50 mmHg. The skin is warm and flushed. The patient has no motor ability or sensation in his legs. Which one of the following is the MOST likely cause of this patient's presentation? A) Hypoglycemia B) Internal bleeding C) Hemorrhagic shock D) Spinal injury

Answer: D - Spinal injury

A 49-year-old man has been stabbed in the lower right chest. Police on scene report that the patient got into an argument with an unidentified man, who then stabbed him before fleeing the scene. After assessing the patient, you are suspicious that the knife punctured the lung and is causing internal bleeding. In this situation, the mechanism of injury would be: A) Blunt trauma to the lungs B) Assailant with a knife C) Internal hemorrhage D) Stab wound to the right chest

Answer: D - Stab wound to the right chest

A young boy was running with a pencil and tripped. The pencil impaled the boy's left eye and remains lodged in place. He is in his mother's arms and both are crying hysterically. Appropriate care for the eye would include: A) Carefully removing the pencil from the eye to prevent the loss of vision, then providing immediate transport. B) Applying ice to the affected eye to decrease pressure, swelling, and subsequent damage. C) Applying gentle pressure to the pencil and eye to prevent movement followed by immediate transport. D) Stabilizing the impaled object and providing immediate transport.

Answer: D - Stabilizing the impaled object and providing immediate transport

Which one of the following pieces of equipment would the EMT need to obtain a patient's vital signs? A) Oxygen B) Glucometer C) Automated defibrillator D) Stethoscope

Answer: D - Stethoscope

What type of injection places medications into the loose connective tissue between the skin and the muscle? A) Intradermal B) Sublingual C) Intramuscular D) Subcutaneous

Answer: D - Subcutaneous

A construction worker has been shot with a nail gun, resulting in a long nail going through his cheek and firmly embedding into the lower gum and jaw. There is considerable blood in his mouth and he is in excruciating pain. Given this scenario, which of the following would be your FIRST priority? A) Removing the nail B) Placing a cervical collar C) Applying oxygen D) Suctioning the airway

Answer: D - Suctioning the airway

A patient has suffered an abdominal evisceration. There is no evidence of spinal injury. How should you place the patient on the stretcher? A) Semi-Fowler's, to promote adequate breathing B) Prone, to maintain pressure on the abdomen C) Lateral recumbent with legs extended D) Supine, with knees flexed to his chest

Answer: D - Supine, with knees flexed to his chest

he three components of the circulatory system are: A) The heart, the brain, and the blood B) The heart, the arteries, and the veins C) The blood, the plasma, and the blood vessels D) The blood, the heart, and the blood vessels

Answer: D - The blood, the heart, and the blood vessels

Your partner reports that a patient's blood pressure is 156/78 mmHg. From this reading, what should the AEMT recognize? A) The pulse pressure is 224 mmHg B) The blood pressure in the veins is 156 mmHg C) The systolic blood pressure is 78 mmHg D) The diastolic blood pressure is 78 mmHg

Answer: D - The diastolic blood pressure is 78 mmHg

Which one of the following is a benefit of the automated external defibrillator? A) The need for little-to-no training B) Its ability to identify and confirm cardiac arrest C) Elimination of the need for a primary assessment D) The ease and speed by which it can be used

Answer: D - The ease and speed by which it can be used

Which one of the following statements about the circulation of blood is TRUE? A) Pulmonary arteries are the only arteries that carry oxygenated blood. B) The right myocardium is thicker than the left myocardium. C) The right atrium receives oxygenated blood from the vena cava. D) The left atrium sends oxygenated blood into the left ventricle.

Answer: D - The left atrium sends oxygenated blood into the left ventricle.

Which of the following occurs next after a person is exposed to a pathogen? A) The person gets sick from the pathogen. B) The infected patient is contagious to others. C) It dies immediately upon contact with the body. D) The pathogen begins to colonize in the body.

Answer: D - The pathogen begins to colonize in the body.

You have arrived on the scene of a call for a possible stroke. On your arrival, the patient denies signs and symptoms, is alert and oriented, and moves all extremities well. Her husband states that before you arrived the patient could not move her right arm and the left side of her face seemed to be "slack." Which of the following has MOST likely occurred? A) The patient suffered a cerebral vascular accident. B) The patient is suffering from aphasia. C) The patient has had a subarachnoid hemorrhage. D) The patient suffered a transient ischemic attack.

Answer: D - The patient suffered a transient ischemic attack.

Which of the following BEST explains why positivepressure ventilation is the correct management step for a patient with flail segment? A) The positive pressure limits any movement of the thorax, thus allowing only the unaffected side to be ventilated B) The positive pressure accentuates movement of the thorax inward, reducing the movement of the fracture site and moving the flail segment with the chest C) The positive pressure permanently inflates the flail segment in order to maximize oxygenation D) The positive pressure displaces the thorax outward, reducing the movement of the fracture site and moving the flail segment with the chest

Answer: D - The positive pressure displaces the thorax outward, reducing the movement of the fracture site and moving the flail segment with the chest

A car strikes a tree causing the passenger to strike his forehead against the dashboard. After the impact, the brain bounces back and forth within the skull. Based on the anatomy and physiology of the skull, which one of the following is TRUE? A) The frontal bone is the thinnest portion of the skull and most easily fractured. B) The brain will not be injured since the flexibility of the skull will absorb the force of the impact. C) The brain will not be injured unless an object on the dashboard penetrates the skull. D) The ridges of the basilar skull can injure the brain as it moves back and forth.

Answer: D - The ridges of the basilar skull can injure the brain as it moves back and forth.

What is the period between exposure and the production of enough antibodies to be detected in the blood? A) The disease period B) The fulminant period C) The latent phase D) The window phase

Answer: D - The window phase

A generalized motor seizure that begins with a hypertonic state of the muscles is known as a(n): A) Absence seizure. B) Hysterical seizure. C) Complex partial seizure. D) Tonic-clonic seizure.

Answer: D - Tonic-clonic seizure.

Which of the following statements about tension pnuemothorax is FALSE? A) Lung collapse will occur on the side of the injury B) Hypotension will occur when venous return is impaired C) JVD occurs when the vena cava becomes crimped D) Tracheal deviation is the most prevalent and common sign of tension pnuemothorax

Answer: D - Tracheal deviation is the most prevalent and common sign of tension pnuemothorax

A flail segment occurs when: A) Mechanism of injury involves penetrating trauma B) A tension pnuemothorax has occurred C) At least two ribs are broken during a blunt trauma incident D) Two or more ribs are broken in two or more places

Answer: D - Two or more ribs are broken in two or more places

A patient with a history of frequent urinary tract infections (UTI) is complaining of lower abdominal pain and feels as though she has one again. Which of the following assessment findings would cause you to question the presence of a UTI? A) Temperature of 101.3° F B) Flank pain C) Cloudy urine D) Vaginal bleeding

Answer: D - Vaginal bleeding

Which of the following may be used in the treatment of symptomatic paroxysmal supraventricular tachycardia (PSVT)? A) Lidocaine B) Defibrillation C) Epinephrine D) Valsalva maneuvers

Answer: D - Valsalva maneuvers

A 44-year-old electrician has been shocked. He is responsive to verbal stimuli and has garbled speech. His airway is open and he is breathing shallowly at a rate of eight times per minute. His pulse is slow and irregular. Which one of the following actions should the AEMT perform next? A) Insert an oral airway and apply 15 lpm oxygen with a nonrebreather face mask. B) Obtain the patient's pulse, blood pressure, and pulse oximetry reading. C) Attach but do not turn on the automated external defibrillator. D) Ventilate with a bag-valve mask at 12 breaths per minute.

Answer: D - Ventilate with a bag-valve mask at 12 breaths per minute.

You suspect your patient is suffering from a stroke. What information is crucial to obtain from family and bystanders that will aid in proper treatment? A) When was the last meal consumed? B) Is the patient oriented to the current time? C) Does the patient seem more agitated than usual? D) What was the time of onset of the symptoms?

Answer: D - What was the time of onset of the symptoms?

Your patient has an apparent isolated brain injury. It would be BEST to position this patient: A) With the head of the spine board lowered about 30-degrees. B) With the spine board in a level, horizontal position. C) With the foot of the spine board raised about 30-degrees. D) With the head of the spine board raised about 30-degrees.

Answer: D - With the head of the spine board raised about 30-degrees

Which of the following is TRUE regarding urinary tract infections? A) Hypertension may also occur with an overwhelming UTI. B) Fever is rarely seen with a urinary tract infection. C) Pyelonephritis is often a result of urinary tract infections. D) Women tend to get more urinary tract infections than men.

Answer: D - Women tend to get more urinary tract infections than men.

A patient has had part of his right thumb amputated in an industrial accident. Coworkers have retrieved the thumb and wrapped it in a towel. The AEMT demonstrates appropriate handling of the amputated part when he: A) Keeps the thumb warm during transport B) Places the thumb in sterile saline C) Places the thumb in ice water D) Wraps it in a dry sterile dressing

Answer: D - Wraps it in a dry sterile dressing

Which of the following blood glucose reading would you MOST likely see in patient with nonketotic hyperosmolar coma? A) ≤ 60 mg/dL. B) 80-150 mg/dL. C) 300-500 mg/dL D) ≥ 1,000 mg/dL

Answer: D - ≥ 1,000 mg/dL

One of the primary treatments for rhabdomyolysis is: A) Administration of a 50 percent dextrose solution. B) IV antibiotics to reduce inflammation of the tissues. C) IV administration of pain relievers. D) Administration of isotonic crystalloid fluids.

Answer: D -Administration of isotonic crystalloid fluids.

You are called for a 20-year-old college student respiratory distress. Of the following information, which would MOST likely indicate a possible pneumothorax? A) He has had a fever and coughing for two or three days. B) He has just returned from the Asia and is having night sweats. C) He has a rescue inhaler that he uses when he needs it. D) He began having shortness of breath after lifting a heavy object.

Answer: D -He began having shortness of breath after lifting a heavy object.

A contradiction to administering nitroglycerin to an ACS patient would be: A) Previous coronary arterial bypass graph (CABG) B) Associated hypertension within the last 24 to 36 hours C) Chest pain associated with tachycardia and dyspnea D) Use of erectile dysfunction medications within 24 to 36 hours

Answer: D- Use of erectile dysfunction medications within 24 to 36 hours

You are documenting the pain felt by a patient with pneumonia. The pain he felt in the upper quadrant of the abdomen on the affected side is known as: A) Parietal pain. B) Visceral pain. C) Referred pain. D) Tearing pain.

Answer:C - Referred pain

All of the following are examples of possible antigen ingestion EXCEPT: A) Taking a newly prescribed antibiotic B) Getting stung by a wasp C) Eating a cookie with peanuts in it D) Eating shrimp and other shellfish

B

All of the following are signs or symptoms of histamine release EXCEPT: A) Constriction of bronchioles B) Increased vascular permeability C) Edema in the airway D) Peripheral vasodilation

B

What is the preferred route and dose of epinephrine for adult suffering from allergic reaction? A) 0.3-0.5 mg SQ B) 0.3-0.5 mg IM C) 3-5 mg SQ D) 3.0-5.0 mg IV

B

Which of the following is CORRECT regarding autoimmune disease? A) Type 1 diabetes is a well known autoimmune disease. B) The immune system fails to recognize its own cells and destroys them. C) Graves' disease is a common form of collagen vascular disease. D) There is a marked decrease in production of bone marrow cells.

B

Your patient is a 23-year-old man who was complaining of shortness of breath following a bee sting. He is audibly wheezing and his respiratory rate is 22 breaths per minute. His skin is cool and clammy and he is tachycardic. What is the primary drug used to treat this patient? A) Atropine B) Epinephrine C) Diphenhydramine D) Albuterol

B

A bleacher at a college has collapsed, and you are the first EMS unit to arrive on the scene. As you enter the gym, several patients are walking around with various injuries. Which one of the following instructions is MOST appropriate for these ambulatory patients? A) "Put this yellow tag on your wrist and go outside to the red fire truck in the parking lot." B) "Go outside and wait by the red fire truck in the parking lot and stay there." C) "Sit down here so that I can do a quick assessment on you." D) "Leave this gym and wait outside. I will be out in a few minutes after I check others who cannot walk."

B) "Go outside and wait by the red fire truck in the parking lot and stay there."

You are assessing a patient with schizophrenia. Friends called 911 because he was experiencing hallucinations this morning. He is prescribed thoiridazine and risperidone for his illness. Which one of the following questions is MOST pertinent to the current situation and must be asked FIRST? A) "Has your psychiatrist considered changing the dose of your medications?" B) "Have you been taking your medications as prescribed?" C) "Do you think that the hallucinations are real?" D) "Do you drink alcohol to help get you through the day?"

B) "Have you been taking your medications as prescribed?"

You have been called to a nursing home for a confused 91-year-old male patient who became dizzy and fell, striking his head on the floor. Which one of the following statements made by the nurse should be of MOST concern to you? A) "I just checked his blood sugar and it is 154 mg/dL." B) "He has a heart valve problem and takes Coumadin for it." C) "He did not have any of his medications today." D) "He is normally confused and his family just made him a DNR."

B) "He has a heart valve problem and takes Coumadin for it."

You are on-scene of a serious motor-vehicle collision. As the officer in charge, which one of the following instructions for other rescuers is MOST appropriate? A) "Park the ambulance downhill to make movement of the patient easier." B) "Let us stop all traffic and reroute it to alternative roadways." C) "We need to route passing traffic no more than 20 feet around the scene." D) "Let us make sure to put cones around the cars."

B) "Let us stop all traffic and reroute it to alternative roadways."

You are teaching some EMRs to use the Jump START system of triage. Which one of the following comments made by an EMR requires you to intervene and provide corrective instruction? A) "I like the Jump START system since it uses the same categories of assessment as the START system of triage." B) "The Jump START system is very effective since it allows me to take two minutes to assess the patient." C) "The Jump START system allows me to classify the patient according to his ability to ambulate, his breathing, perfusion, and mental status." D) "If we have a young child injured in a multiple-casualty incident, I will use the Jump START system to triage him."

B) "The Jump START system is very effective since it allows me to take two minutes to assess the patient."

You should attempt resuscitation if the fetus is at least _____ weeks or beyond unless death is obvious. A) 18 B) 20 C) 22 D) 24

B) 20

Which one of the following patient descriptions would the AEMT classify as a behavioral emergency? A) 44-year-old woman with bipolar disorder complaining of a fever of 102.5°F B) 36-year-old man with no known history who is hearing voices C) 29-year-old man with diabetes who is suddenly confused D) 56-year-old man with schizophrenia who has stopped taking his medications

B) 36-year-old man with no known history who is hearing voices

Moderate hypothermia results when the patient is experiencing signs and symptoms of hypothermia with a core temperature between: A) 93.2°F and 97.9°F. B) 86.0° F and 93.2° F. C) 98.9°F and 105°F. D) 91.0°F and 97.9°F.

B) 86.0° F and 93.2° F.

Of the following patients, which would NOT be considered a patient with special challenges? A) A terminally ill patient B) A burn patient C) A dialysis patient D) A brain injured patient

B) A burn patient

All of the following can lead to malnourishment in the elderly population EXCEPT: A) Diminished salivation and gastric secretions B) A disruption of the sleep-wake cycle C) Decreased senses of taste and smell D) Decreased financial resources

B) A disruption of the sleep-wake cycle

Which of the following patients would benefit from using a CPAP machine? A) A patient with liver failure B) A patient with sleep apnea C) A paralyzed patient D) A dialysis patient

B) A patient with sleep apnea

A method that is commonly used in the field to assess a newborn's vital functions is known as the ________ score. A) AVPU B) APGAR C) OPQRST D) SAMPLE

B) APGAR

Nerve agents will cause overstimulation of the parasympathetic nervous system resulting in the release of: A) Dopamine B) Acetylcholine C) Epinephrine D) Norepinephrine

B) Acetylcholine

During delivery, you note bulging of the perineal area when the mother bears down to expel the fetus from the birth canal. You should: A) Tear the perineum with a gloved finger to prevent further trauma B) Apply gentle pressure to the head of the fetus to prevent explosive delivery C) Use water-based lubricant to facilitate delivery D) Attempt to spread the labial folds further to assist delivery

B) Apply gentle pressure to the head of the fetus to prevent explosive delivery

You suspect that the bruising to the face of an 88-year-old woman is the result of abuse by her caregiver. On-scene which one of the following is MOST appropriate? A) Call law enforcement and wait with the patient for them to arrive B) Apply ice to the bruise and transport the patient to the ED C) Inconspicuously investigate the house for additional clues D) Confront the caregiver about your suspicion

B) Apply ice to the bruise and transport the patient to the ED

If your patient in labor complains that she "needs to move my bowels," you should: A) Apply direct pressure to the vaginal opening B) Assess the patient for crowning C) Place her in Trendelenburg position D) Leave for the hospital immediately

B) Assess the patient for crowning

You respond to an apparent SIDS death where the baby is obviously dead. You should: A) Attempt resuscitation even though the baby has rigor mortis B) Be careful not to disturb the scene since it could be a crime scene C) Ask the parents why they placed the baby on his stomach to sleep D) Tell the parents that it looks like the child suffocated on his blankets

B) Be careful not to disturb the scene since it could be a crime scene

Which of the following would a COPD patient use at home? A) Mechanical ventilator B) CPAP or BiPAP C) Apnea monitor D) Groshong lines

B) CPAP or BiPAP

In placenta previa, bleeding occurs as a result of: A) Spontaneous abortion B) Cervical dilation C) Imminent delivery of fetus D) Uterine contractions

B) Cervical dilation

A drop in core body temperature to <95°F significantly reduces the body's blood-clotting mechanism. This decrease in the blood's ability to clot is referred to as: A) Decompensated shock B) Coagulopathy C) Multisystem trauma D) Compensated shock

B) Coagulopathy

To gain access to a patient entrapped in a car, the rescuers must use the "jaws of life" to open the door. The AEMT should recognize this situation as: A) Simple access B) Complex access C) Rapid extrication D) Hydraulic stabilization

B) Complex access

The initial blast wave associated with an incendiary explosion is likely to cause: A) Lacerations and abrasions B) Compression injury to ears, lungs, and hollow air-filled organs C) Sudden death D) Burns

B) Compression injury to ears, luncs, and hollow air-filled organs

You have been called for an 11-year-old boy with a history of mild mental retardation who has possibly ingested the leaves of a household plant. His mother states that she found him eating the leaves then immediately called 911. You note that the plant has been overturned and is missing a considerable number of leaves. Assessment reveals the boy to be lethargic and confused and complaining of abdominal pain, accompanied by nausea and vomiting. His airway is patent and his respirations are adequate. Which assessment finding would be MOST suggestive that the plant leaves have been absorbed into the body? A) Nausea and vomiting B) Confusion and lethargy C) Abdominal pain D) Nausea

B) Confusion and lethargy

A woman has called you for her middle-aged friend who has stopped eating and talking and has frequent crying spells. Your history reveals that the woman lost her husband a few months ago and has been withdrawn and despondent ever since. Throughout the assessment, the alert and oriented woman continually states she wants to be left alone. When you bring the stretcher into the room, the woman states, "I do not want to go to the hospital. Let me sign whatever I need to sign." What is your BEST response to this situation? A) Recheck mental status and orientation, and then have her sign a refusal. B) Contact medical direction and ask for advisement. C) Allow the patient to sign a refusal as long as her friend stays with her. D) Contact law enforcement to restrain the patient for transport.

B) Contact medical direction and ask for advisement.

An unrestrained female patient who was involved in a minor motor-vehicle collision refuses EMS care or transport. She is seven months pregnant and is alert and oriented. When asked, she denies any complaint. Your BEST action would be to: A) Make contact with her obstetrician B) Contact medical direction for instructions C) Have the police witness her signing the refusal D) Have her follow up with her obstetrician

B) Contact medical direction for instructions

Which of the following affects the lower airway? A) Pharyngitis B) Croup C) Pneumonia D) Laryngitis

B) Croup

When treating burns due to incendiary devices, the AEMT should expect: A) Hypoxic atmospheres B) Deep, extensive burns C) Asphyxiating mechanisms D) Pressure-wave injuries

B) Deep, extensive burns

At the scene of a multiple-casualty drill, you have been assigned to role of Transport Unit leader. As such, your responsibility is: A) Immobilizing all patients prior to transport B) Distributing patients to each medical facility C) Ensuring proper care for each patient D) Ensuring that patients in cardiac arrest leave for the hospital first

B) Distributing patients to each medical facility

Which of the following would be MOST appropriate when providing care for a blind patient? A) Interview the family members to collect information about the patient's problem B) Do not touch the patient without first letting him know what you are going to do C) Speak softly to the service dog so he will know you are not a threat to his owner D) Determine if the patient is also deaf and check to see if he has his hearing aid in

B) Do not touch the patient without first letting him know what you are going to do

What drug, described as a modified form of methamphetamine, is often seen in nightclubs? A) Barbiturates B) Ecstasy C) Alcohol D) Marijuana

B) Ecstasy

At the scene of a multiple-casualty incident caused by a weapon of mass destruction, the AEMT's FIRST priority is: A) Identifying the agent used B) Ensuring the safety of yourself and your partner C) Establishing a communications system D) Estimating the number of dead and injured

B) Ensuring the safety of yourself and your partner

You respond to an apparent SIDS death. There appears to be bruising on the baby's back and buttocks area. The parents say they put the baby down for a nap and he never woke up. What should you do? A) Ask the parents what they did to the baby B) Explain that they are not to blame for the baby's death C) Ask if the baby has been abused by someone in the household D) Find out if the baby had been placed on his back for his nap

B) Explain that they are not to blame for the baby's death

A young man convinces his grandmother to take out a home equity line of credit to loan him $50,000 even though she lives on social security. He invested the money in a business that soon went bankrupt. Now, the bank is foreclosing on grandma and she has nowhere to go. This is an example of: A) Elder abuse B) Financial abuse C) Domestic abuse D) Psychological abuse

B) Financial abuse

A tube which is surgically inserted through the abdominal wall, directly into the stomach is called a(n): A) J tube B) G tube C) OG tube D) NG tube

B) G tube

Which type of radiation can travel the farthest and penetrate the human body with the greatest ease? A) Beta B) Gamma C) Blast D) Alpha

B) Gamma

An incendiary device differs from conventional explosives because they: A) Contain many metal fragments for dispersion from the explosion B) Have less explosive power, but more heat C) Will create a larger crater when detonated D) Have greater explosive power and more heat involved

B) Have less explosive power, but more heat

When you are dealing with penetrating trauma, the major concern with solid organs is: A) Respiratory compromise B) Hemorrhage C) Spilling of contents D) Infection

B) Hemorrhage

In the elderly population, respiratory muscle function declines and the chest wall becomes less compliant. Which of the following can be a result of this deterioration? A) Increase likelihood of broken ribs B) Higher incidence of pneumonia in the elderly C) A build-up of mucus in the airways D) Pleuritic chest pain that is worse with inspiration

B) Higher incidence of pneumonia in the elderly

Secondary injuries associated with explosions include all of the following EXCEPT: A) Blunt injury B) Hollow organ rupture C) Burns D) All of the above

B) Hollow organ rupture

Which of the following is CORRECT regarding a patient with cystic fibrosis? A) Avoid providing IV fluids as it may cause further lung congestion B) Humidified oxygen is preferred over non-humidified oxygen C) Cysts form in the lungs inhibiting exchange of oxygen and carbon dioxide D) IM epinephrine is used to dilate the bronchioles and alveoli

B) Humidified oxygen is preferred over non-humidified oxygen

You have been called for an eight-year-old girl who is having seizures. The mother tells you that the child has not been sick, no fever or vomiting. However, she does have a shunt. You should suspect her seizures are caused by: A) A seizure disorder B) Increased intracranial pressure C) A recent head injury D) An acute increase in temperature

B) Increased intracranial pressure

A patient attempting to compensate for blood loss and shock may present with any or all of the following EXCEPT: A) Decreasing pulse pressure and cool, clammy skin B) Increasing pulse pressure and cool, clammy skin C) Increasing respiratory rate and decreasing tidal volume D) Increasing pulse rate and decreasing pulse strength

B) Increasing pulse pressure and cool, clammy skin

When an AEMT surveys an accident scene and anticipates the injuries related to the event, they are developing their: A) Protocols B) Index of suspicion C) Plan of action D) Mechanism of injury

B) Index of Suspicion

Spinal injury care steps performed during the primary assessment include: A) Apply a cervical collar independent of spinal stabilization B) Initiate proper manual spinal stabilization followed by mechanical immobilization C) Perform a rapid rollout independent of manual spinal stabilization D) Administer medications to prevent further paralysis

B) Initiate proper manual spinal stabilization followed by mechanical immobilization

A hollow tube that is surgically placed in a ventricle of the brain extending to the abdomen is called a(n): A) Ostomy tube B) Intraventricular shunt C) Central venous catheter D) Tracheostomy tube

B) Intraventricular shunt

An intoxicated 24-year-old woman has been struck by lightning and is conscious but confused. Which one of the following assessment findings should the AEMT investigate FIRST? A) Deformity to the left wrist B) Irregular heart beat C) Burns to both arms D) Unequal pupils

B) Irregular heart beat

A problem with the Simple Triage and Rapid Transport (START) system of triage is it: A) Is only used for traumatic injuries B) Is difficult to apply to young children C) Is complex and difficult to use D) Requires at least 10 rescuers

B) Is difficult to apply to young children

Which of the following statements about the umbilical cord is TRUE? A) It contains one artery and two veins B) It contains two arteries and one vein C) The function is to surround and protect the fetus D) It can be over 10 feet long

B) It contains two arteries and one vein

You are caring for a child that has been repeatedly dunked in extremely hot water. He has blisters on his lower body all the way to his waist. How should you handle this situation? A) Let the abuser know that you have called the police B) Keep your anger in check and focus on treating the child C) Give the abuser a taste of his own medicine D) Ask the neighbor if they have observed any abuse

B) Keep your anger in check and focus on treating the child

Which of the following is TRUE regarding home oxygen equipment? A) An oxygen concentrator can deliver up to 95 percent oxygen B) Liquid oxygen lasts much longer than compressed gas C) They can be complicated and require frequent servicing D) They are connected to a dehumidifier and oxygen cylinder

B) Liquid oxygen lasts much longer than compressed gas

You are interviewing an elderly patient about his eyesight. He tells you that he has good peripheral vision, but has diminished central vision. You should suspect: A) Glaucoma B) Macular degeneration C) Proprioception D) Confabulation

B) Macular degeneration

You are attending to a patient who reports that she is three months pregnant and is receiving pre-natal care. Your partner obtains a blood pressure of 98/60, pulse of 84, and respirations of 14. The patient is complaining of nausea and vomiting. You should suspect: A) Hypovolemia B) Morning sickness C) Spontaneous abortion D) Internal bleeding

B) Morning sickness

Signs and symptoms of ______________include postural rigidity, tremors, and loss of facial expression. A) Alzheimer's disease B) Parkinson's disease C) Meniere disease D) Starling's disease

B) Parkinson's disease

Beta radiation represents the greatest threat to a person's well being when the: A) Source is heated to greater than 100°F B) Person ingests it by eating contaminated food C) Radiation occurs in liquid form D) Person is within 30 feet of the source

B) Person ingests it by eating contaminated food

You are in charge of triage at a scene involving 30 patients exposed to carbon monoxide. You find a six-year-old boy who is not breathing. You open his airway and find that breathing does not return. Furthermore, you cannot locate a radial pulse. You should: A) Apply oxygen and check the patient's mental status B) Place a black tag on the patient and move to the next C) Provide 15 seconds of positive pressure ventilation D) Provide 15 seconds of CPR and recheck for a radial pulse

B) Place a black tag on the patient and move to the next

You have encountered a patient who is approximately one mile from the road where you parked your ambulance. The patient was cross-country skiing and became disoriented in the dark and has been in a crude shelter all night. Examination reveals a moderately hypothermic patient who is alert and oriented. His feet are white and hard to palpation with decreased sensation. All of the following are true EXCEPT: A) Hypothermia is a true concern and warming should begin immediately. B) Prior to walking the patient to the ambulance, you should attempt to rewarm the feet. C) When warming the patient's feet, you should immerse in warm water and avoid rubbing. D) Transport should be to a facility that can provide active internal and external rewarming.

B) Prior to walking the patient to the ambulance, you should attempt to rewarm the feet.

You have just arrived on the scene of a motor-vehicle collision where a car rolled several times at a high rate of speed. The car is on its wheels with the restrained driver still in his seat. After ensuring that the scene is safe and gaining access to the patient, which one of the following should you do immediately? A) Establish a plan to extricate the patient B) Provide manual inline spinal stabilization C) Remove pieces of broken glass from around the patient D) Cover the patient and force the door open

B) Provide manual inline spinal stabilization

The AEMT shows he is correctly using the Jump START triage system on a three-year-old male trauma patient when he: A) Makes a conscious effort to look for bruises to the patient's body B) Quickly determines the perfusion status and mental status C) Carefully examines the chest for signs of retractions D) Obtains a systolic and diastolic blood pressure

B) Quickly determines the perfusion status and mental status

You have been called for a 91-year-old woman who fell. Since your assessment reveals bruises to her body in various stages of healing and inconsistencies in information related to the fall from her family, you are suspicious of geriatric abuse. Your BSET action would be to: A) Notify the Office of Elder Affairs prior to leaving the scene B) Quietly transport and inform the emergency department physician C) Confront the family members regarding your suspicions D) Have the EMS supervisor contact the patient's primary care physician

B) Quietly transport and inform the emergency department physician

The MOST obvious sign of the potential chemical weapon deployment is: A) High incidence of illness over a period of days in the same geographical area B) Rapidly spreading symptoms among a group of people C) Large explosions involving urban sites D) Large amount of burned victims

B) Rapidly spreading symptoms among a group of people

Which of the following would be appropriate to care for an elderly patient? A) Cut away excess clothing when trying to examine the chest B) Recheck breath sounds after the patient has taken a few breaths C) Apply more pressure to open the airway if the neck is stiff D) Pad the cervical collar with extra padding to keep it snug

B) Recheck breath sounds after the patient has taken a few breaths

You have been trained to the level of OSHA "First Responder Awareness." In relation to this level of training, which one of the following activities are you qualified to perform? A) Direct the operations at a hazardous materials emergency until the chemical is identified B) Recognize a potential hazardous materials emergency and call for the appropriate resources C) Stop and patch a leak in a broken valve emitting an unknown gas D) Provide detailed instructions on how to contain a spill of an unknown liquid

B) Recognize a potential hazardous materials emergency and call for appropriate resources

If a scene becomes unsafe after you have entered, you should FIRST: A) Notify the EMS supervisor and medical director B) Remove yourself and your crew from the scene C) Attempt to neutralize the threat D) Call for assistance on the radio

B) Remove yourself and your crew from the scene

After being assaulted by her boyfriend, an 18-year-old woman who is 33 weeks pregnant is found unresponsive in her bedroom. She appears to have been beaten in the head and abdomen as evidenced by multiple hematomas and lacerations to these areas. Which assessment finding should concern the AEMT most and be addressed FIRST? A) Abdominal contusions B) Room air SpO2 at 93 percent C) Bright red vaginal bleeding D) Rapid heart rate

B) Room air SpO2 at 93 percent

At the scene of a motor-vehicle collision, a driver is entrapped in his car after striking the back of a truck at a high rate of speed. Which action would be performed FIRST when providing stabilization of the car? A) Placing the transmission in "park" B) Shutting off the engine C) Setting the parking brake D) Disconnecting the battery

B) Shutting off the engine

Decreased clearance of drugs that are metabolized by the liver may contribute to: A) Constipation and bowel obstruction B) Signs and symptoms of drug overdose C) Increased risk of infection D) Impaired proprioception

B) Signs and symptoms of drug overdose

Your patient is an eight-year-old girl. As you approach her, you note that she is holding her right arm against her body, cradling it with the left arm. To find out what happened, which of the following is the BEST approach? A) Ask the parents to tell you what happened while you examine the child B) Sit next to the patient, and ask her what happened C) Take the parents aside, and ask them what happened D) Ask the parents to leave the room so you can talk to the patient

B) Sit next to the patient, and ask her what happened

While responding to an MCI drill involving an explosion, you pass a dozen ambulances sitting in a parking lot three blocks away from the incident. You should recognize this area as the: A) Treatment unit B) Staging unit C) Transport unit D) Triage area

B) Staging unit

As you pull up to a motor vehicle collision, you quickly scan the scene for clues as to the type and severity of injuries. Which of the following scene clues would BEST indicate that a patient may have a head injury? A) Blood on a patient's clothing B) Starring of a windshield C) Airbag deployment D) Frontal-type collision

B) Starring of a windshield

A young man has been struck by lightning on a golf course and is in cardiac arrest. Other golfers say that he was struck about 10 minutes ago. Which one of the following should you do immediately? A) Ventilate 10 to 12 breaths per minute. B) Start cardiopulmonary resuscitation. C) Do a secondary assessment to locate the burn sites. D) Attach the automated external defibrillator.

B) Start cardiopulmonary resuscitation.

You are called to a nursing home for a male patient with chest pain. On arrival you find the patient in a wheelchair with a noticeable hematoma to his forehead and a skin tear to his left wrist. Staff states that the patient had an episode of chest pain just before falling to the ground. When asked, the patient states that he still has chest discomfort. Which one of the following should you do immediately? A) Perform the primary assessment and apply oxygen B) Take manual inline spinal stabilization C) Apply oxygen and treat patient using the chest pain protocol D) Apply oxygen and look for other injuries the patient may have suffered

B) Take manual inline spinal stabilization

A car has struck a utility pole at a high rate of speed. The male driver appears to be unresponsive with snoring respirations. The vehicle has just been stabilized, and you have donned the appropriate personal protective equipment. Which one of the following should you do next? A) Place a tarp over some spilled antifreeze B) Take manual spinal stabilization C) Apply a cervical collar to his neck D) Perform a jaw-thrust airway maneuver

B) Take manual spinal stabilization

A reference tool that should be placed in every emergency vehicle for use when responding to a possible "hazmat" incident is/are A) MSDS sheets from local chemical users and suppliers B) The Emergency Response Guidebook C) The Merck manual D) The Physician's Desk Reference

B) The Emergency Response Guidebook

A six-month-old has been stung on the tongue after putting a bottle in his mouth that had a bee on the nipple. In comparison to the same injury in an adult, why would you be more concerned with this child? A) A child's airway has more blood vessels than an adult's, making bleeding more of a concern B) The child's tongue is proportionally larger, increasing the chance of airway occlusion from swelling C) Children tend to be highly allergic to bee stings while the same allergy in an adult is rare D) When injured, a child's mouth produces more saliva, making airway occlusion a major concern

B) The child's tongue is proportionally larger, increasing the chance of airway occlusion from swelling

When treating a pediatric trauma patient at the scene of a MCI, which one of the following is it important for the AEMT to remember? A) The elevated metabolism of a child causes them to retain heat, thus a blanket should not be applied when in shock B) The flexibility of the chest wall in a child can allow for internal injury without obvious external signs C) Since the body of a child is smaller than an adult's body, there is less chance of multisystem trauma D) Because children tend to be healthier than adults, they are able to compensate longer when in shock

B) The flexibility of the chest wall in a child can allow for internal injury without obvious external signs

The terrorist who chooses chlorine or phosgene as a weapon of choice does so because: A) The highly volatile state renders it as an explosive B) The low specific gravity keeps it close to the ground where more exposure can occur C) It will travel large distances and contaminate many victims D) The explosive potential is a benefit

B) The low specific gravity keeps it close to the ground where more exposure can occur

Which of the following would be typical for a patient in hospice care? A) As death nears, the patient is weaned from pain medications B) The patient and family have regular visits from a social worker C) The patient is transported to the hospital for weekly evaluation D) Pain medications are used to keep the patient sedated

B) The patient and family have regular visits from a social worker

Your patient states that he "passed out" and fell down. He regained consciousness a few moments later, but had shortness of breath and sharp chest pain. Which one of the following in the patient's history is of greatest concern? A) He had hip surgery two years ago B) The patient states his left calf is tender and painful C) The patient took nitroglycerin and now has a headache D) The patient injured his wrist when he fell

B) The patient states his left calf is tender and painful

At a drill involving the rollover of a tourist bus, you have been assigned to the transport area. Which one of the following activities will you perform within this section of the disaster response? A) Triage and assign priorities of EMS care and transport to the hospital B) Transfer patients to ambulances for transport to the hospital C) Triage and provide patient care prior to transport to the hospital D) Transport patients to the hospital or other appropriate facilities

B) Transfer patients to ambulances for transport to the hospital

At the scene of a hazardous materials emergency, you have been assigned to the cold zone. In this zone, which one of the following activities will you be performing or assisting with? A) Rescue and initial decontamination B) Triage, vital signs, and medical histories C) Removal of contaminated rescue gear D) Initial decontamination and triage

B) Triage, vital signs, and medical histories

The risk of ____________ is substantially increased by pulling on the umbilical cord in an attempt to speed delivery of the placenta. A) Placenta previa B) Uterine inversion C) Abruptio placenta D) Uterine rupture

B) Uterine inversion

You have been asked by your medical director to put together an emergency kit specifically for patients who have been stung or bitten by small marine life animals. Which one of the following should you place in that kit? A) Cayenne pepper extract B) Vinegar C) Ethanol D) Cold packs

B) Vinegar

Your patient is a 32-year-old woman who has taken an unknown quantity of unknown pills. All of the following are questions that should be asked during your assessment EXCEPT: A) When did you ingest the substance? B) Where did you get the substance? C) What did you ingest? D) How much did you ingest?

B) Where did you get the substance?

You are called for a seven-year-old boy with difficulty breathing. Using the pediatric assessment triangle, you notice that the boy is sitting upright in the tripod position. This observation is obtained during which part of the assessment triangle? A) Appearance B) Work of breathing C) Circulation to the skin D) Respiratory status

B) Work of breathing

Treatment of heat cramps should include: A) deep muscle massage. B) a tepid solution containing a small amount of sodium. C) rapid infusion of cooled IV fluids. D) very cold water provided orally until the patient feels better.

B) a tepid solution containing a small amount of sodium.

Compared to dementia, delirium often has a ________ onset. A) chronic B) acute C) fatal D) delayed

B) acute

Convection causes the body to lose heat by: A) direct contact with another surface. B) air passing over a surface. C) radiation of heat. D) evaporation of moisture.

B) air passing over a surface.

During the primary assessment of a patient with a behavioral emergency, you should: A) use a reverse order to the ABCs. B) be aware of potential lifethreats. C) assume drug and alcohol abuse exists. D) all of the above

B) be aware of potential lifethreats.

Other than intentional injections of illicit drugs, MOST injected poisonings result from: A) accidental needle sticks. B) bites and stings of insects and animals. C) rusty nails. D) infected wounds.

B) bites and stings of insects and animals.

You are dispatched to an auto collision. Because of very heavy traffic, your response time is 25 minutes. The closest hospital is 40 miles from the accident site. You should: A) have state law enforcement stop traffic while you transport B) call for air transport C) request that law enforcement escort you to a hospital D) request additional ground units

B) call for air transport

The MOST important assessment technique in evaluating behavioral emergencies is: A) auscultating lung sounds. B) conversational skills. C) defensive tactics. D) obtaining vital signs.

B) conversational skills.

A 42-year-old construction worker faints while mixing cement. The patient presents warm and diaphoretic, with rapid and shallow respirations. He responds to verbal stimuli and complains of being extremely weak. Management should consist of: A) encouraging the patient to drink iced tea. B) cooling the patient. C) administering and IV of D5W. D) giving the patient salt tablets.

B) cooling the patient.

You are treating a patient who feels that the government has control agents who monitor all of his moves in order to control his income; this patient is experiencing: A) schizophrenia. B) delusions. C) bipolar episodes. D) mania.

B) delusions.

Your patient is standing outside where the ambient temperature is 100 degrees F. Due to the thermal gradient, you would expect: A) heat transfer from the patient to the environment. B) heat transfer from the environment to the patient. C) heat exchange from the patient by conduction. D) heat loss to occur quickly.

B) heat transfer from the environment to the patient.

If transport time is extended for a patient who has severe deep frostbite to an extremity, it is BEST to attempt warming by: A) vigorously massage the affected areas to promote circulation. B) immersing in water that is approximately 104 degrees F. C) administer vasopressors to promote circulation. D) rubbing the affected area gently.

B) immersing in water that is approximately 104 degrees F.

Soon after being placed in a cool environment, the body will respond by: A) inhibiting conduction through piloerection. B) inducing thermogenesis to increase heat production. C) inducing thermolysis to release heat. D) all of the above

B) inducing thermogenesis to increase heat production.

You are treating a construction worker who has been assembling large, iron girders with a torch. It is very warm where you are treating the patient. Suddenly, he experiences a severe cramp to his shoulder region. This has probably occurred because the patient: A) may have dislocated his shoulder during the activity. B) lost both water and sodium in his sweat. C) is probably not strong enough to maneuver the iron girders. D) was not sweating enough to keep him cool.

B) lost both water and sodium in his sweat.

As part of the focused medical examination of the behavioral emergency, the AEMT should perform: A) field sobriety tests. B) mental status evaluation. C) pulse oximetry monitoring. D) head-to-toe trauma assessment.

B) mental status evaluation.

You are by the side of a young female college student who is unresponsive. Friends say that she is not a heavy drinker, but tonight she drank an excessive amount of alcohol. She responds to painful stimuli and has snoring respirations. Her respiratory rate is 8 breaths per minute and her radial pulse is weak. Your immediate action would be to: A) determine if drugs are also involved. B) perform the head-tilt/chin-lift maneuver. C) administer positive pressure ventilation. D) apply oxygen via a nonrebreather mask.

B) perform the head-tilt/chin-lift maneuver.

There are sensors throughout the surface of the skin known as: A) chemoreceptors. B) peripheral thermoreceptors C) baroreceptors. D) central thermoreceptors

B) peripheral thermoreceptors

Shivering is a mechanism in which: A) metabolic activity decreases in order to create heat. B) physical activity increases generating heat. C) the muscle cells lose their ability to compensate for heat loss. D) blood shunts from the muscles generating heat.

B) physical activity increases generating heat.

In order to appropriately restrain a patient: A) handcuffs need to be present. B) protocols should be in place. C) more than five people must be present. D) all of the above

B) protocols should be in place

Although difficult to define, normal behavior is BEST defined as behavior that is: A) dangerous to the patient or others. B) readily acceptable to society. C) considered normal for the patient. D) all of the above

B) readily acceptable to society.

A 24-year-old woman who was hiking in shorts informs you that she has been bitten by an insect on her leg. She states no other complaints. Assessment reveals a tick embedded in her leg. As an AEMT, you should provide care to this patient by: A) administering high-concentration oxygen. B) removing the tick with tweezers. C) wrapping the tick and bite area with roller gauze. D) applying a hot pack over the tick.

B) removing the tick with tweezers.

Chronic depression greatly increases the possibility of: A) PTSD. B) suicide. C) chronic anxiety. D) all of the above

B) suicide.

The main compensatory mechanisms for heat dissipation are: A) vasoconstriction and shivering. B) vasodilation and sweating. C) vasodilation and shivering. D) shivering and sweating.

B) vasodilation and sweating.

A disaster survivor who is experiencing behavioral issues most likely​ needs: A. to be placed in law enforcement custody B. psychological support C. physical support D. to be transported to the ED under suicide watch

B.

Emission of radiation that results in radiation exposure or contamination is known​ as: A. nuclear hazards B. radiologic hazards C. radioactive emission hazards D. hazardous material hazards

B.

What is the safest assumption for the responder to make following the detonation of a conventional​ explosive? A. This was an isolated incident and should be the primary focus on the scene. B. The bomber has the​ material, expertise, and ability to deliver a secondary device. C. The bomber only has the material to detonate one explosive. D. The bomber was is a novice and probably does not have the means to create a secondary explosive device.

B.

Which of the following acronyms indicates signs and symptoms to assess for following a terrorist attack using nerve​ agents? A. ​DCAP-BTLS B. SLUDGE C. RAIN D. ​AEIOU-TIPS

B.

Which of the following is an example of a riot control​ agent? A. ricin B. oleoresin capsaicin spray C. anthrax D. mustard gas

B.

Which of the following is the highest priority on the scene of a natural disaster or terrorist​ attack? A. patient care B. safety C. responder rehabilitation D. setting up incident command

B.

You and your partner are the first to arrive on the scene of a bombing at a local nightclub. What should you initially​ do? A. Assume incident command while triaging all patients. B. Perform a windshield assessment from a safe distance. C. Immediately begin care of the most critically wounded. D. Stage far away until law enforcement arrives.

B.

Your patient has been exposed to a nerve agent and is presenting with massive​ diarrhea, vomiting, and copious salivation. You treatment should include quick administration​ of: A. oxygen in a hyperbaric chamber B. atropine and pralidoxime chloride C. racemic epinephrine D. acetylcholine and epinephrine

B. Atropine and pralidoxime chloride​ (2-PAM) is used to reverse the effects of nerve agents.

Which of the following special teams are essential to help the responder and the patient deal with the acute stress of a disaster and combat PTSD following the​ event? A. the American Red Cross B. disaster​ mental-health services C. the incident command system D. rehabilitative services

B. Disaster​ mental-health services are a critical part of the response to natural disasters and terrorism events. Do not hesitate to make use of those services. Medical and psychological monitoring does not end when you leave the scene. Depending upon the type of​ event, you might be required to perform a​ follow-up physical at a​ doctor's office hours or even days after an​ event, particularly if you begin to experience unusual physiologic signs and symptoms.

Upon arrival to the scene of a terrorist attack involving a conventional​ explosion, what must all responders be alert​ for? A. exposure to hazardous materials B. possibility for a secondary explosion C. signs and symptoms of exposure to nerve agents D. wind speed and direction

B. When responding to a suspected terrorist​ incident, always be alert to the possibility of secondary​ devices, attacks, or weapons targeted at injuring or killing emergency response personnel.

Which of the following is an example of a biologic agent that may be employed as a weapon of mass​ destruction?

Botulism toxin

A sensitization is defined as: A) an exaggerated response to a particular antigen. B) the cascade of events that occurs following activation by a foreign substance. C) the primary exposure of an individual to an antigen. D) the manifestation of severe, systemic effects of a histamine release.

C

An allergic reaction, as contrasted with an anaphylactic reaction will: A) have a more rapid onset with more serious symptoms. B) require fluids to support blood pressure. C) have a more gradual onset with milder symptoms. D) present a patient with an altered mental status.

C

What is an unfortunate side effect of the treatment for autoimmune disease? A) Patients often develop peripheral neuropathy. B) The patient has a dramatic increase in red blood cells. C) The patient is less able to fight infectious diseases. D) Patients experience increased gastrointestinal motility.

C

You are by the side of a patient who fell. He has a hematoma to his forehead as well as deformity of his left forearm. His daughter informs you that he has severe dementia and she cares for him at home. As you assess him, he continually screams out, "Mary, help me. I need help!" Which one of the following questions is MOST appropriate to ask the daughter? A) "Was Mary his wife?" B) "Do you know what he means by this?" C) "Is this how he normally is?" D) "Isn't dementia is a terrible disease?"

C) "Is this how he normally is?"

Which of the following statements regarding treatment of the pediatric patient in the prehospital setting is CORRECT? A) "If the primary assessment of a sick pediatric patient shows no deficits to the ABCs, the patient is stable and will not deteriorate." B) "Since pediatric patients can deteriorate very quickly, they must all be transported with lights and sirens during the ride to the hospital." C) "It is critical that the AEMT be able to determine the respiratory status of the patient as this is most likely the reason for acute deterioration." D) "When treating a sick pediatric patient in the prehospital setting, the AEMT must determine the exact illness so as to provide the most appropriate treatment."

C) "It is critical that the AEMT be able to determine the respiratory status of the patient as this is most likely the reason for acute deterioration."

The AEMT correctly identifies the primary benefit of triage when she states: A) "Triage is a system used by the AEMT to assign the order of care when an incident contains more than five patients." B) "Triage is a system that can be used by the AEMT to determine the number of patients who will most likely die." C) "Triage is used to determine the order in which patients will receive medical care and transport." D) "Triage is a process by which the number of patients is quickly estimated so the appropriate resources can be summoned."

C) "Triage is used to determine the order in which patients will receive medical care and transport."

Which of the following is an open-ended question? A) "Is this your prescription?" B) "Did you do this?" C) "What are your thoughts right now?" D) "Do you take any medications?"

C) "What are your thoughts right now?"

A 44-year-old woman was hiking in a state park when she was bitten by a snake in her left calf. To get help, she had to hike four miles to the first-aid station where you are located. The hike took her one and a half hours. Your assessment reveals a red and swollen area to her leg, and she complains of discomfort to that area. Closer inspection reveals two puncture wounds in the center of the affected area. She asks you if she is going to die from the snake bite. Which one of the following is the BEST reply? A) "Based on the way the bite looks, I think that you were bitten by a nonpoisonous snake." B) "Walking all of that distance probably saved you since your body metabolized the poison faster." C) "You need to get checked out, but if venom was injected, the signs and symptoms of a generalized reaction typically appear very quickly." D) "We need to get you to the hospital immediately. Anytime that a snake bites it injects venom from its fangs."

C) "You need to get checked out, but if venom was injected, the signs and symptoms of a generalized reaction typically appear very quickly."

Fetal heart tones can be heard with a Doppler at about ________ weeks. A) 20 B) 15 C) 10 D) 5

C) 10

Your patient is a toddler who was burned when he accidentally pulled the cord of a deep fryer sitting on a countertop and spilled hot oil on his head and back. He has burns to the back half of his head, as well as his entire back. What percentage of total body surface area does this account for? A) 40 B) 36 C) 27 D) 18

C) 27

You are caring for a young boy who is very lethargic and his mother tells you he has had diarrhea and vomiting for several days. What would be the proper amount of fluids to administer in the initial bolus if the boy weighs 44 pounds? A) 440 mL B) 600 mL C) 400 mL D) 220 mL

C) 400 mL

You are providing IV fluids to a two-year-old boy who is dehydrated. You should use a ______ gtt/mL drip set. A) 10 B) 30 C) 60 D) 80

C) 60

A 42-year-old patient weighs 154 pounds (70 kg). How much activated charcoal would you administer to him? A) 500 grams B) 100 grams C) 70 grams D) 154 grams

C) 70 grams

What would be the minimally acceptable blood pressure for a nine-year-old girl who is suffering from a significant bleed from broken glass? A) 96 mmHg B) 79 mmHg C) 88 mmHg D) 100 mmHg

C) 88 mmHg

Because it is NOT practical for an AEMT to always calculate a MAP in the field, use a systolic blood pressure (SBP) of ______ as a sign of adequate perfusion. A) 100 mmHg B) 120 mmHg C) 90 mmHg D) 60 mmHg

C) 90 mmHg

Which of the following would transfer the MOST force to a patient? A) A car moving 70 miles an hour and entering the water B) A car moving 70 miles an hour and hitting brush C) A car moving 70 miles an hour and hitting a concrete wall D) A car moving 70 miles an hour and hitting a collapsible sign post

C) A car moving 70 miles an hour and hitting a concrete wall

You are on the scene of a multiple-casualty incident where a church van carrying 15 parishioners has overturned. You and your partner are in charge of the treatment unit and are presented with the following four patients. Who will you treat first? A) A 21-year-old man in cardiac arrest B) An elderly female patient with a femur fracture and a yellow tag on her wrist C) A confused 69-year-old man with a respiratory rate of 40 and a rapid and weak radial pulse D) A 13-year-old boy who walked to the treatment area complaining of a severe headache

C) A confused 69-year-old man with a respiratory rate of 40 and a rapid and weak radial pulse

Which of the following patients has the GREATEST potential for violent behavior? A) A patient who has witnessed the death of a spouse B) A patient with delusions of being someone else C) A patient with a knife who is stabbing a stuffed animal D) A patient talking to himself

C) A patient with a knife who is stabbing a stuffed animal

The AEMT would recognize which of the following as TRUE regarding poisoning? A) The majority of poisonings is intentional and result in homicide or suicide. B) All toxins are poisonous when in contact with the human body. C) A poison is any substance that impairs a person's health by its chemical action. D) The number one cause of poisoning in the United States is medications.

C) A poison is any substance that impairs a person's health by its chemical action.

In the case of MCI trauma patients, the ideal goal for completing on-scene triage procedures are: A) Set by medical direction B) To treat as you find patients C) A quick sorting process D) To treat women and children first

C) A quick sorting process

You are treating a patient with a suspected obstetrical complaint. General treatment of the patient should include: A) Large-bore IVs and head-down positioning during transport B) ABCs, oxygen, large-bore IV D5W, and rapid transport C) ABCs, oxygen, IV with isotonic solution, left-lateral recumbent position D) Immediately load the patient and transport with lights and sirens

C) ABCs, oxygen, IV with isotonic solution, left-lateral recumbent position

Medical direction has ordered you to administer activated charcoal to a patient who ingested a large amount of poison. When looking in your medical kit, which of the following medications would you prepare and administer? A) Nitrostat B) Glyburide C) Actidose D) Ecotrin

C) Actidose

A patient has inhaled cyanide gas. He is alert and complaining of shortness of breath. His airway is patent, his breathing is adequate, and his radial pulse is strong. His skin is warm and dry, and his breath sounds are clear. Given these assessment findings, what should the AEMT do FIRST? A) Administer the antidote for cyanide B) Monitor the patient and transport C) Administer oxygen via nonrebreather mask D) Obtain vital signs with the patient in a supine position

C) Administer oxygen via nonrebreather mask

You are assessing an elderly patient who was just released from the hospital following surgery to replace a knee joint. The patient is complaining of shortness of breath, sharp pain with breathing, and is anxious. Vital signs are: blood pressure 90/66, pulse 110, rate 34. The BEST treatment for this patient is to: A) Administer oxygen, establish an IV, and rapidly infuse fluids B) Have the patient re-breathe CO2 from a paper bag until ventilations decrease C) Administer oxygen, establish an IV, and keep TKO unless needed D) Begin positive pressure ventilation via BVM using 100 percent oxygen

C) Administer oxygen, establish an IV, and keep TKO unless needed

You are performing CPR on a three-monthold baby who stopped breathing during his nap. There are no obvious signs of death so you begin resuscitation. The parents are watching you as you perform CPR on the baby. You should: A) Have the parents removed from the room while you treat the baby B) Call law enforcement to management the parents in case they get upset C) Allow the parents to watch and explain to them that you are doing everything possible D) Have the parents take over CPR while you get ready to transport to the hospital

C) Allow the parents to watch and explain to them that you are doing everything possible

Which of the following patients would be the MOST likely to be abused? A) A 68-year-old man with asthma B) A 62-year-old woman with dementia C) An 82-year-old woman with Alzheimer's disease D) A 74-year-old man with incontinence

C) An 82-year-old woman with Alzheimer's disease

Which of the following conditions does NOT cause an upper airway obstruction? A) Croup B) Epiglottitis C) Asthma D) Bronchiolitis

C) Asthma

Which of the following statements about bipolar disorder is TRUE? A) It may take years to manifest the mania. B) Patients with bipolar disorder often are delusional. C) Bipolar disorder is characterized by one or more episodes of mania. D) Bipolar disorder is characterized by repeated depressive episodes.

C) Bipolar disorder is characterized by one or more episodes of mania.

Which one of the following patient triage tags indicates the lowest priority for care and transport on the scene of a multiple-casualty incident? A) Yellow B) Green C) Black D) Red

C) Black

Which of the following is NOT one of the problems commonly encountered by a patient who is paralyzed? A) Respiratory infections B) Urinary tract infections C) Blocked intraventricular shunts D) Decubitus ulcers

C) Blocked intraventricular shunts

An unresponsive patient is in a car with the doors locked and the windows up. Which one of the following actions is MOST appropriate when attempting to gain access to the patient? A) Use a crowbar to break the lock on the door B) Call for a rescue truck to forcibly remove a door C) Break the window opposite the patient with a spring-loaded punch D) Break the window by striking it with a crowbar

C) Break the window opposite the patient with a spring-loaded punch

You are called for a baby with respiratory distress. The 18-month-old patient has a fever of 104.1°F, has expiratory wheezing and his lungs sound as if they have mucus in the lower airways. His mother reports the baby has been sick for a couple of days with an upper respiratory infection. You should suspect: A) Croup B) Pertussis C) Bronchiolitis D) Pneumonia

C) Bronchiolitis

When victims are exposed to nuclear radiation, which of the following will occur? A) Blindness will result B) Irreversible damage occurs immediately to the lung tissue C) Cells are damaged as radiation passes through D) Victims will rapidly die when exposed

C) Cells are damaged as radiation passes through

You have been called to an assisted living facility for a 43-year-old man who has a history of anxiety and schizophrenia and who, according to staff, is more confused today. The staff also states that he has refused to take his medications for the past three days. Your assessment reveals him to have no life threats to the ABCs, but is slurring his speech and not making sense when he talks. As you review paperwork provided to you by staff, you note that he has a history of thyroid problems, diabetes, and hypertension. His pulse is 124, respirations 16, and blood pressure 154/70 mmHg. At this point, which one of the following actions is MOST important? A) Determine reason for refusing medications. B) Recheck the blood pressure. C) Check his blood glucose level. D) Restrain and transport the patient.

C) Check his blood glucose level.

As the AEMT responsible for triage at a motor-vehicle collision, you find a patient who is breathing 18 times per minute and has a radial pulse. Which one of the following should you do next? A) Check the blood pressure B) Move the patient to the treatment area C) Check the mental status D) Place a yellow tag on the patient

C) Check the mental status

A layer of the extraembryonic membrane that develops blood vessels within finger-like projections is called the: A) Endometrium B) Villi. C) Chorion D) Placenta

C) Chorion

Which one of the following situations represents simple access to a patient? A) Unlocking a car door by breaking the window using a screwdriver B) Extrication of a patient from a car after removing one door C) Climbing stairs to gain access to a patient on a porch D) Forcing entry into a house

C) Climbing stairs to gain access to a patient on a porch

You have stopped at a distance from a train car that has derailed and appears to be leaking some kind of gas. What will help you identify the substance and precautions you should take? A) Experience with previous incidents can be most in identifying the substance by color, smell, and volatility B) CHEMTREC papers that are carried in all emergency vehicles that contain specific information related to any emergency situation involving chemicals C) Emergency Response Guidebook will identify placards and give basic instructions in initial management steps D) Shipping papers that are located on the end of the cars will contain all MSDS, including any specific hazards that exist and management strategies

C) Emergency Response Guidebook will identify placards and give basic instructions in initial management steps

A large bus with 52 passengers overturned on a busy highway. On-scene 12 people were dead and 23 others were taken to various hospitals, many with life-threatening injuries. As commander of the incident, which one of the following is MOST appropriate after the last patient has left the scene? A) Promote lighthearted talk about the incident to decrease rescuer tension and stress B) Mandate that all rescuers attend a stress debriefing C) Encourage rescuers to talk amongst themselves about their feelings D) Bring all rescuers to a central point and hold a discussion to critique the rescue effort

C) Encourage rescuers to talk amongst themselves about their feelings

You are the first emergency personnel on the scene of an overturned tanker truck. The driver is in the cab and appears motionless. It also appears that a liquid is leaking from the rear-valve assembly of the tanker. After properly positioning your ambulance, your immediate priority should be to: A) Attempt to determine what chemical is contained in the tanker B) Take manual inline spinal stabilization C) Evacuate people from cars near the tanker D) Perform the primary assessment on the truck driver

C) Evacuate people from cars near the tanker

A patient has sustained an injury to his arm from an unknown vesicant. After assessing the airway, breathing, and circulation, you should: A) Transport the patient to the hospital immediately B) Brush the powder from the skin C) Flush the injured area with water D) Apply a wet sterile dressing to the arm

C) Flush injured area with water

You are obtaining a history from an obese, 15-year-old who is pregnant. She tells you this is her first pregnancy and she called 911 because her stomach has been cramping. Which part of her history would lead you to be concerned about gestational diabetes? A) First pregnancy B) Stomach cramps C) Her weight D) Her age

C) Her weight

In which of the following would you MOST likely see a child with eyes that appear to bulge with increased visibility of the superior part of the globe, resulting in "sunset eyes"? A) Cystic fibrosis B) Meningitis C) Hydrocephalus D) Seizures

C) Hydrocephalus

You arrive on the scene of a popular nightclub to find a 22-year-old woman in custody of the police. Witnesses state she had been taking her clothes off on the dance floor, and when club staff asked her to stop she struck one of them. She is now sitting in the back of the police cruiser crying. She admits to "X-ing." Which of the following physical findings would you not expect to find in this patient? A) Tachycardia B) Anxiety C) Hypotension D) Fever

C) Hypotension

All of the following are appropriate in treating the patient who is suspected of having preeclampsia EXCEPT: A) Left-lateral recumbent position B) High-flow oxygen C) IV fluid bolus D) Reduce sensory stimulation

C) IV fluid bolus

Chemical weapons are MOST often designed for use in: A) Direct contact with the victim B) Explosive situations C) Indoor or confined spaces D) Incendiary devices

C) Indoor or confined spaces

Which of the following correctly identifies the routes by which a poison can enter the human body? A) Inhalation, Injection, Nasal-Oral, Visual B) Exposure, Ingestion, Inhalation, Injection C) Ingestion, Inhalation, Injection, Absorption D) Topical, Inhalation, Nasal-Oral, Absorption

C) Ingestion, Inhalation, Injection, Absorption

Orogastric tubes are: A) Slightly smaller than a nasogastric tube B) Inserted through the nose C) Inserted through the mouth D) Slightly shorter than a nasogasatric tube

C) Inserted through the mouth

The driver of a truck carrying a radioactive substance has accidentally come into contact with the material. You are first on the scene and a quick observation indicates that he still has the powder on his arm. Which one of the following should you do immediately? A) Rinse the powder off the arm and wrap with a sterile dressing and roller gauze B) Provide basic care, but avoid touching the patient's arm C) Instruct the patient to remain still until proper removal of product D) Place the patient in the ambulance, but avoid touching him en route to the hospital

C) Instruct the patient to remain still until proper removal of product

Which of the following is TRUE regarding preeclampsia? A) Occurs when the blood pressure surpasses 160/90 mmHg B) Usually occurs to women who are over the age of 35 C) It can develop within 48 hours after delivery D) Presents with seizures and hypertension

C) It can develop within 48 hours after delivery

________ is another name for bipolar disorder. A) Delusional B) Expansive depression C) Manic-depressive D) Schizophrenia

C) Manic-depressive

An example of a incendiary devices include: A) Tear gas canister B) Smoke bomb C) Molotov cocktail D) Pepper spray

C) Molotov cocktail

A chemical agent that interferes with normal functioning of nervous transmission is known as: A) Chemical asphyxiant B) CNS depressant C) Nerve agent D) All of the above

C) Nerve agent

What chemical, often used as a pesticide, is easily absorbed through dermal contact? A) Tricyclics B) Carbamates C) Organophosphates D) Hemotoxins

C) Organophosphates

Which of the following is a respiratory infection that results in coughing fits that can interfere with breathing? A) Bronchiolitis B) Croup C) Pertussis D) Epiglottitis

C) Pertussis

When treating a poisoning patient, what should be one of your main objectives? A) Administering Narcan B) Determining the reason for the poisoning C) Preventing aspiration D) Administering syrup of ipecac

C) Preventing aspiration

All of the following are good indicators of affect EXCEPT: A) Body position B) Mental status C) Religious beliefs D) Verbal tone

C) Religious beliefs

Emergencies involving a patient with a tracheostomy tube generally results from which of the following? A) The tube fails or ruptures, or the tube becomes dislodged B) The tube splits from excess airway pressures or becomes occluded C) The tube becomes plugged with mucus, or the tube becomes dislodged D) The tube softens, or becomes occluded with foreign material

C) The tube becomes plugged with mucus, of the tube becomes dislodged

An unrestrained female driving a small car is involved in a rollover-type collision. Why her risk for serious injury and death is significantly increased? A) The risk for ejection is lessened, increasing injuries suffered in the car B) Rollovers are the result of high speeds C) There are more impacts in a roll over D) Smaller and lighter cars tend to roll over

C) There are more impacts in a roll over

Which of the following is TRUE regarding ostomy bags? A) They divert urine from the patient's urinary bladder B) A common complication is a urinary tract infection C) They collect feces directly from the patient's colon D) They collect excess CSF from an intraventricular shunt

C) They collect feces directly from the patient's colon

________ is a collection of signs and symptoms that are indicative of poisoning. A) Toxicant B) Toxicology C) Toxidrome D) Toxicodynmics

C) Toxidrome

You are treating a young child whom you suspect may be a victim of abuse. As you are examining him, you notice bruises of various colors on his back and legs. He is conscious and vitally stable with no immediate life threats. What should you do? A) Call law enforcement and have them take the parents into custody B) Advise the parents that you will notify law enforcement of your findings C) Transport the child for further evaluation by professionals at the ED D) Be careful to not record your suspicions in your patient care report

C) Transport the child for further evaluation by professionals at the ED

All of the following are roles of the poison center EXCEPT: A) Determining the potential toxicity of the agent B) Notifying the receiving hospital before arrival of the patient C) Treating the patient based on the agent introduced D) Suggesting the most current, definitive treatment

C) Treating the patient based on the agent introduced

Which of the following statements is MOST appropriate regarding dealing with caregivers and children during a medical emergency? A) It is best to separate the caregiver from the child so that a proper assessment and care can be given B) Tell the caregiver that everything will be okay so they are more calm and better able to help the child C) Try to include the caregiver in all you do with the child so that the child and caregiver are more comfortable D) Tell the caregiver to step out of the room so you can assess the child without their interference

C) Try to include the caregiver in all you do with the child so that the child and caregiver are more comfortable

A young woman was weeding her garden when she was bitten on her left hand by an unidentified snake. The primary assessment does not reveal any life threats and vital signs are stable. In caring for this patient, it is critical that the AEMT: A) apply ice packs to the area of the snake bite. B) attempt to locate the snake for identification. C) have the patient remove her wedding ring from her left hand. D) administer the epinephrine auto-injector into the site.

C) have the patient remove her wedding ring from her left hand.

If heat exhaustion is NOT treated, the patient is at risk for: A) pulmonary edema. B) respiratory distress. C) heat stroke. D) heat cramps.

C) heat stroke.

Using the Centers for Disease Control and​ Prevention's biologic agent categorization which category is the least​ virulent? A. A B. B C. C D. D

C.

What is the biologic agent that causes the bubonic form of the​ plague? A. Francisella tularensis B. Bacillus anthracis C. Yersinia pestis D. Clostridium botulinum toxin

C.

Which of the following could indicate a potential MCI following a suspected terrorist attack using hazardous​ materials? A. A single responder presents with signs and symptoms of exposure to radiation. B. Multiple patients are anxious but otherwise in good medical condition. C. Multiple patients present with similar signs and symptoms. D. A single patient presents with altered mental status.

C.

Which of the following would be considered an example of a category B​ bioagent? A. hantavirus B. anthrax C. brucellosis D. botulism

C.

You respond to the home of a​ 55-year-old college professor who has been burned. Upon arrival to the​ scene, law enforcement tells you that a small pipe bomb detonated when he opened his mail box. Which type of terrorism has he most likely been the victim​ of? A. domestic terrorism B. international terrorism C. ​lone-wolf terrorism D. patriot terrorism

C.

The purpose of preplanning for an MCI or disaster is​ to: A. assure that all services understand each of the other​ service's SOPs B. assure all responders get paid when they respond C. anticipate any problems that may occur and work toward removing them D. make sure that an MCI or disaster does not occur in your community

C. Anything that can be planned in advance involving an MCI should be planned in advance. The first step to preplanning includes a complete assessment of potential​ hazards, both natural and​ man-made, that could occur in your area.

On the scene of a terrorist attack that involved a single conventional​ explosive, you are performing a primary assessment on several patients. Which type of injury would you most likely​ expect? A. complex lower extremity fractures B. burns and blunt trauma C. burns and penetrating trauma D. barotrauma to the lungs and ears

C. Conventional explosives create high heat upon detonation and are designed to detonate​ violently, often leading to the dispersion of shrapnel that will create penetrating injuries in patients in close proximity of the explosion.

The best way to manage a stressful situation during a multiple casualty incident is​ to: A. ask for relief from duties B. only care for less critical patients C. remain task oriented D. inform the incident commander

C. EMS providers can be put in overwhelmingly stressful situations in which they still must perform their jobs. To maintain control in such​ situations, focus on accomplishing the tasks at hand. You must temporarily put aside intrusive thoughts and​ feelings, with the understanding that you will need to examine and manage them later. Next Question

All of these statements concerning response to terrorism events are true except​: A. Even in a terrorism​ event, all patients exposed to chemicals must be decontaminated before treatment and transport. B. An apparent victim at a terrorism event may actually be a perpetrator. C. Unlike other crime​ scenes, evidence identification and preservation are not a consideration in the response to terrorism. D. Unlike typical hazardous materials​ scenes, it is nearly impossible to quickly identify the chemical involved in a terrorist event.

C. Terrorism is a​ crime, and terrorism events are crime scenes. Follow evidence preservation protocols established by incident command. Make note of anything that could be of interest in the investigation by law enforcement. For​ example, gasoline​ cans, electrical​ components, or pieces of a cell phone may be evidence. Next Question

The Centers for Disease Control​ A, B, and C classifications for biological agents refer to the​ __________________ of the agents. A. lethality B. ability to multiply C. virulence D. communicability

C. The Centers for Disease Control and Prevention categorize biologic agents by their​ virulence, with Category A agents being the most​ virulent, and Category C the least virulent.

Which of the following indicates a potential threat to responding personnel on the scene of a terrorist attack involving incendiary​ devices? A. hazardous materials exposure B. high amounts of radiation C. secondary explosion devices D. shrapnel spread throughout the scene

C. When responding to a suspected terrorist​ incident, always be alert to the possibility of secondary​ devices, attacks, or weapons targeted at injuring or killing emergency response personnel.

The Centers for Disease Control and Prevention​ (CDC) categorize biologic agents by their virulence. Which category is the most​ virulent?

Category A

Fluid released from leaky capillaries as a result of histamine results in: A) mottling. B) eczema. C) hypersensitivity. D) urticaria.

D

What is the role of diphenhydramine in anaphylaxis? A) Relaxes the smooth muscle in the lungs B) Causes constriction of peripheral blood vessels C) Reduces wheezes in the upper airways D) Prevents histamine from binding to histamine receptors

D

Which of the following is a potentially life-threatening condition involving the head, neck, face, and upper airway? A) Hives B) Wheals C) Urticaria D) Angioedema

D

Your patient is experiencing profuse hives, itching, dyspnea, coughing, tachycardia, and dizziness after eating seafood five minutes ago. You might also expect to see all of the following additional signs EXCEPT: A) Cyanosis B) Laryngeal edema C) Bronchospasm D) Hypertension

D

You are transporting a 44-year-old man who was stung multiple times by fire ants. Which one of the following patient statements should concern you MOST? A) "I took some Motrin this morning for a headache." B) "The bites are really beginning to hurt more." C) "My hand seems to be more swollen now." D) "I am beginning to feel itchy all over."

D) "I am beginning to feel itchy all over."

You are on the scene of a car that hit a tree head on. The patient has a laceration to his forehead and a deformity to his left forearm. The car is a hybrid that runs on traditional as well as hydrogen gas. There is moderate front-end damage to the vehicle, but the airbags did not deploy. The patient was wearing a seatbelt. Which one of the following instructions is MOST appropriate in this situation? A) "Let us deactivate the airbags by turning the engine off." B) "We will need to puncture the hydrogen tank to prevent an explosion." C) "Place a flare near the oil spill so that no one slips." D) "I need to have the battery cables disconnected."

D) "I need to have the battery cables disconnected."

A patient is unconscious with bradypnea after a possible heroin overdose. Which of the following medications and dosages would be MOST appropriate? A) 0.2 mg of flumazenil B) 1 mg of flumazenil C) 5 mg of naloxone D) 1 mg of naloxone

D) 1 mg of naloxone

The normal heart rate for a newborn is between ________ and ________. A) 80-100 B) 100-120 C) 120-140 D) 140-160

D) 140-160

Which of the following characteristics of a neonate would require some form of resuscitation? A) Baby born at 39 weeks of gestation B) A baby who is crying loudly C) A baby who is moving vigorously D) A baby whose heart rate is 90

D) A baby whose heart rate is 90

A patient has been bitten by a snake. Bystanders do not know what kind it was but state that it had red, yellow, and black rings and that the red rings were banded on either side by yellow rings. The patient has generalized numbness and weakness in addition to slurred speech. Which of the following signs or symptoms would you also expect? A) Dysuria B) Dry mouth and mucous membranes C) Hyperactivity D) Abdominal pain

D) Abdominal pain

You are treating a multigravida female who is not receiving prenatal care. The patient is 42-years-old and is in her third trimester of pregnancy. She admits to using cocaine earlier in the day and throughout her pregnancy. She is currently experiencing a painful, rigid abdomen and signs and symptoms of shock. You should suspect: A) Spontaneous abortion B) Placenta previa C) Eclampsia D) Abruptio placenta

D) Abruptio placenta

What route of entry is the typical method of entry for poisonous plants, such as poison ivy? A) Injection B) Inhalation C) Ingestion D) Absorption

D) Absorption

To BEST communicate with a geriatric patient who has diminished hearing, the AEMT should do which one of the following? A) Increase both the pitch and volume of your voice B) Ask the family members about the patient's history C) Try to communicate using sign language D) Be sure the patient has his hearing aid in place

D) Be sure the patient has his hearing aid in place

A premature delivery occurs: A) Anytime before 40 weeks B) Between 30 and 38 weeks C) Before 37 weeks of gestation D) Between 28 and 36 weeks

D) Between 28 and 36 weeks

Chemicals that cause a disruption in the metabolism of oxygen by the cell like cyanide are called: A) Pulmonary agents B) Hypoxemic agents C) Ataxic agents D) Blood agents

D) Blood agents

When conducting the size up of a MCI, you will identify: A) The estimated number of patients B) The need for additional resources C) The person who caused the accident D) Both A and B

D) Both A and B

A frantic mother called 911 because she found her 18-month-old daughter playing with a bottle of drain cleaner. When assessing this patient, which of the following provides the strongest evidence that the child did indeed ingest some of the liquid? A) Increased heart rate B) Garbled speech C) Continual crying D) Burns to her mouth

D) Burns to her mouth

You have responded to a bar for a fight involving approximately 20 patrons. Weapons were used and all the patrons have varying degrees and types of injuries. You are in charge of triage and must use the Simple Triage and Rapid Transport (START) system. You find a young person lying on the floor with blood covering his shirt. Quick assessment reveals him to be breathing at 24 times per minute. Which one of the following should you do next? A) Determine the mental status B) Place a yellow tag on his wrist C) Start positive pressure ventilation D) Check for a radial pulse

D) Check for a radial pulse

__________ are longer-term exposures, typically a month or more, of a low dose with decreased intensity. A) Toxicants B) Acute exposures C) Toxidromes D) Chronic exposures

D) Chronic exposures

You are on a scene in which a radioactive material is emitting alpha rays. As a knowledgeable AEMT, you recognize that the MOST appropriate shielding for this material is: A) Concrete B) Aluminum C) Lead D) Clothing

D) Clothing

You have been called for a 13-year-old boy who was caught sniffing glue by his parents. The boy is lethargic and confused and complaining of burning to the mouth and nose as well as nausea and vomiting. Based on the clinical presentation and mechanism of illness, how has this poison affected the boy's brain? A) Absorbing through the small intestine into the blood stream and then on to the brain B) Directly contacting the brain by crossing the mucous membranes of the mouth and nose C) Fumes from the glue were absorbed through the mucous membranes of the eyes and then on to the brain D) Crossing from the lungs into the blood stream and then circulating to the brain

D) Crossing from the lungs into the blood stream and then circulating to the brain

Which of the following can increase the potential for organ injury in blunt trauma in the elderly? A) Decreased bone density B) Decreased sensitivity to pain C) Decreased circulation to the organs D) Decreased amount of adipose tissue

D) Decreased amount of adipose tissue

An 83-year-old woman has fallen down the basement stairs. She complains of left hip pain and rates it 10/10. Since the patient is downstairs, you immediately recognize the need for assistance in extricating her to the ambulance. As you start the primary assessment, you note a dog at the top of the stairs growling. At this time, what is your primary concern? A) Pain B) Airway C) Need for additional help D) Dog

D) Dog

The five components of the top step in the neonatal resuscitation inverted pyramid are: A) Dry, warm, suction, ventilate, stimulate B) Warm, dry, suction, ventilate, compressions C) Warm, position, suction, ventilate, compressions D) Dry, warm, position, suction, stimulate

D) Dry, warm, position, suction, stimulate

Which of the following would demonstrate empathy for a patient? A) Advise an alcoholic that he must quit drinking if he hopes to get better B) Talking to a COPD patient about the dangers of smoking cigarettes C) Tell a patient that it would be less expensive to take a cab to the hospital D) During a cold snap, try to locate a jacket for a homeless man

D) During a cold snap, try to locate a jacket for a homeless man

The mechanism of injury that suggests transport to a trauma center is: A) Extrication time of less than 20 minutes B) A fall from a distance twice the patient's height C) Vehicle impact at a speed greater than 20 mph D) Ejection from a vehicle

D) Ejection from a vehicle

The mechanism of injury that suggests transport to a trauma center is: A) Extrication time of less than 20 minutes B) Vehicle impact at a speed greater than 20 mph C) A fall from a distance twice the patient's height D) Ejection from a vehicle

D) Ejection from a vehicle

The MOST likely weapon that a modern terrorist may use to inflict damage is/are: A) Biological B) Nuclear C) Chemical D) Explosive

D) Explosive

What is the level of training for those who respond to the release of hazardous materials in order to protect people, property, and the environment? A) Hazardous material specialist B) First responder awareness C) Hazardous materials technician D) First responder operations

D) First responder operations

There is a high prevalence of urinary tract infection in patients with ___________. A) Diabetes B) Urinary incontinence C) Diarrhea D) Foley catheters

D) Foley catheters

An intoxicated driver drove his car into the side of a slow-moving train, hitting the tanker car. To get information on what is contained in the tanker, you are going to call CHEMTREC, Inc. What piece of information is it critical to provide this agency so they can help identify the material inside the tanker car? A) Name of the railroad carrier B) Description of the leaking material C) Type of railroad car involved D) Four-digit identification number

D) Four-digit identification number

Which of the following BEST describes the goal of a trauma system? A) Providing all trauma patients with a complete battery of diagnostic tests and access to specialists B) Providing the highest level of care possible within a 10 minute transport time for all patients C) Transporting as many patients as possible by air medical transportation D) Getting the right patient to the right facility in the right amount of time

D) Getting the right patient to the right facility in the right amount of time

Which of the following BEST describes the goal of a trauma system? A) Providing all trauma patients with a complete battery of diagnostic tests and access to specialists B) Providing the highest level of care possible within a 10minute transport time for all patients C) Transporting as many patients as possible by air medical transportation D) Getting the right patient to the right facility in the right amount of time

D) Getting the right patient to the right facility in the right amount of time

On the scene of a hazardous materials emergency, which one of the following individuals is BEST suited to provide command of the incident? A) Fire service rescue technician B) Hazardous materials technician C) Paramedic D) Hazmat specialist

D) Hazmat specialist

On the National Fire Protection Association hazardous materials classification placard, the blue diamond represents: A) Reactivity B) Fire hazard C) Specific hazard D) Health hazard

D) Health hazard

Which of the following is considered a narcotic? A) Crack B) Ritalin C) Mescaline D) Heroin

D) Heroin

The public health approach to injury prevention is an effective way to address problems within the community. Which of the following questions is NOT asked about a specific injury to aid in the development and implementation of an injury prevention program? A) Who B) What C) Where D) How

D) How

All of the following may require the use of an ostomy bag as a result of medical conditions EXCEPT: A) Diverticulitis B) Colon cancer C) Crohn's disease D) Hydrocephalus

D) Hydrocephalus

A second-story deck has collapsed. Several patients have fallen and are lying on the ground below. As the first emergency personnel to arrive on the scene, your initial priority is to determine: A) The number of patients B) The number of ambulances required C) Type and severity of injuries D) If the scene is safe to enter

D) If the scene is safe to enter

At a multiple-casualty incident, a crew brings you a patient with a red tag tied to his wrist. You should recognize which one of the following? A) The patient is deceased B) Delayed care and transport are permissible C) The patient has a minor injury to his wrist D) Immediate care and transport are necessary

D) Immediate care and transport are necessary

For the patient exposed to a nerve agent, which one of the following signs should the AEMT expect? A) Dilated pupils B) Decreased body temperature C) Elevated blood pressure D) Increased salivation

D) Increased salivation

You have responded for a five-year-old girl in severe respiratory distress apparently from an allergic reaction. You remember that the treatment for severe anaphylaxis for a child this age is: A) Intravenous epinephrine 1:1000 at a dosage of 0.01 mg/kg B) Subcutaneous epinephrine 1: 10,000 at a dosage of 0.01 mg/kg C) Intramuscular epinephrine 1: 1000 at a dosage of 0.1 mg/kg D) Intramuscular epinephrine 1:1000 at a dosage of 0.01 mg/kg

D) Intramuscular epinephrine 1:1000 at a dosage of 0.01 mg/kg

Which of the following is NOT a nerve agent? A) "G" agents B) Sarin C) VX D) Lewisite

D) Lewisite

You are caring for a child who has been burned. Which of the following are TRUE regarding burn prevention? A) Toddlers are unable to reach the stove so kitchen burns are not an issue B) Children lack the manual dexterity to light matches or use a lighter C) The older a child gets, the less likely he is to suffer a burn injury D) Lowering the temperature of the hot water heater can prevent serious burns

D) Lowering the temperature of the hot water heater can prevent serious burns

The MAP for a patient with a BP of 120/80 is calculated as follows: A) MAP = 120 + 1/3(120-80) = 133 mmHg B) MAP = 80 + ½(120-80) = 100 mmHg C) MAP = 120 + ½(120-80) = 140 mmHg D) MAP = 80 + 1/3(120-80) = 93 mmHg

D) MAP = 80 + 1/3(120-80) = 93 mmHg

You have arrived on the scene where a tractor trailer has overturned. As you pull onto the scene, the driver of the truck approaches and informs you that he is hauling radioactive medical wastes and warns you that there is a car underneath the overturned trailer. Which one of the following should you do immediately? A) Attempt to identify the material and type of rays being emitted B) Determine if any radioactive material has been released C) Position the ambulance in front of the vehicle to block oncoming traffic D) Move the ambulance upwind of the scene

D) Move the ambulance upwind of the scene

Which of the following statements concerning trauma is TRUE? A) Mechanism of injury is the best predictor of patient outcome B) A "spider web" or "star" pattern of cracks on the windshield means the patient impacted the windshield with his head C) Falls are the leading cause of trauma deaths D) Multisystem trauma has a higher mortality rate than single-system injuries

D) Multisystem trauma has a higher mortality rate than single-system injuries

Bleeding that occurs with placenta previa is usually: A) Painful, and bright red B) Painful, dark, and clotted C) Painless, dark, and clotted D) Painless, and bright red

D) Painless, and bright red

The mother of a four-year-old boy states that he fell off the top bunk bed and hit his head. After falling, he appeared to seize for approximately 30 seconds. Presently he has non-purposeful movement to painful stimuli, with a patent airway and slow respirations. His radial pulse is intact, and his skin is warm and dry. You note urinary incontinence. After taking manual inline spinal stabilization, which one of the following should you do immediately? A) Perform a head-tilt/chin-lift and administer high-concentration oxygen B) Apply oxygen, fully immobilize the patient, and them initiate rapid transport C) Insert an oropharyngeal airway and provide high-concentration oxygen D) Perform a jaw-thrust maneuver and start positive pressure ventilation

D) Perform a jaw-thrust maneuver and start positive pressure ventilation

________________occurs in about one in 8500 births and is characterized by a shorter-than-normal chin and large tongue easily leads to airway obstruction. A) Choanal atresia B) Cleft palate C) Omphalocele D) Pierre Robin syndrome

D) Pierre Robin syndrome

What is the temporary organ of pregnancy, which functions to supply the developing fetus with oxygen and nutrients? A) Cervix B) Uterus C) Amnion D) Placenta

D) Placenta

You are teaching a group of AEMTs about multiple-casualty incidents and the Incident Command System (ICS). Which one of the following statements about ICS is TRUE? A) There is a separate commander for EMS, fire, and law enforcement. B) Each EMS agency can use its own codes and terms. C) Each individual responder is accountable to two supervisors. D) Plain English is used for all communications.

D) Plain English is used for all communications.

Pain that worsens with movement of the chest wall during inspiration is called ________ pain. A) Acute B) Somatic C) Visceral D) Pleuritic

D) Pleuritic

In pediatric patients, except newborns, cough and fever are common early signs of: A) Bronchiolitis B) Pertussis C) Epiglottitis D) Pneumonia

D) Pneumonia

What agency is available to help you determine toxicity of a poison based on type of agent, amount and time of exposure, and physical condition of patient? A) Emergency departments B) Pharmacies C) Intensive care units D) Poison control centers

D) Poison control centers

A midsize car has been struck by a small delivery truck and sustained moderate damage to the front passenger area. The car is on its tires, and wood chocks have been placed under the undercarriage of the car. The airbags did not deploy. Which one of the following is the next step in this rescue? A) Extricate the patient B) Forcibly open the trunk C) Carefully deploy the airbags D) Release air from the tires

D) Release air from the tires

Your EMS system uses the Simple Triage and Rapid Transport (START) system for triaging patients at a multiple-casualty incident. Which one of the following should you include in your triage assessment to determine the order in which patients will receive emergency care? A) Chief complaint, vital signs, and past medical history B) Chief complaint and vital signs C) Level of consciousness and past medical history D) Respiratory and perfusion status

D) Respiratory and perfusion status

The differentiation of preeclampsia and eclampsia is: A) Visual disturbances B) Full body edema C) Hypertension D) Seizures

D) Seizures

It is important to remember that emergencies may NOT present with the normal signs and symptoms experienced by younger populations because: A) Poor skin turgor diminishes pain sensations B) Hyperreflexia is present in the elderly C) The brain is unable to interpret signals D) Sensations may be seriously diminished

D) Sensations may be seriously diminished

Degeneration of the vertebrae and intervertebral disc caused by ____________, can affect the neck and back, limiting mobility and increasing the risk for spine injury. A) Arthritis B) Pathological fractures C) Osteoporosis D) Spondylosis

D) Spondylosis

As an infant delivers, you are presented with a feet-first presentation. You should: A) Place the mother in a head-down position and transport immediately B) Grasp the infant's feet and attempt to pull the infant from the vagina C) Rotate the legs to a supine position and gently apply traction D) Support the body and legs and allow the mother to attempt to deliver on her own

D) Support the body and legs and allow the mother to attempt to deliver on her own

On the scene of a motor-vehicle collision, which one of the following AEMTs is engaged in complex extrication of the patient? A) The AEMT entering the car to open the patient's airway B) The AEMT constantly monitoring the scene for safety hazards C) The AEMT stabilizing the car to prevent it from moving D) The AEMT removing the patient from the car

D) The AEMT removing the patient from the car

Which one of the following BEST describes the term "silent heart attack"? A) The pain did not radiate into the arms, neck, or back B) The patient had a single episode of chest pain followed by cardiac arrest C) The chest pain was so intense the patient could not speak D) The patient had little to no chest pain or discomfort

D) The patient had little to no chest pain or discomfort

Which of the following physical assessment findings would make your patient a candidate for rapid transport to a trauma center? A) Systolic blood pressure of 100 B) Pulse rate of 110 C) Respiratory rate of 24 D) Unstable pelvic girdle

D) Unstable pelvic girdle

A patient was mixing pesticides to spray in his garden and suddenly began experiencing excessive salivation, abdominal pain, and dizziness. The MOST likely route of toxic exposure is: A) ingestion. B) inhalation. C) injection. D) absorption.

D) absorption.

A patient is found in an alleyway, conscious, alert, and breathing. The patient complains of extreme nausea, chills, and is having hallucinations. He keeps asking for a "drink" of beer. You suspect: A) schizophrenia. B) cognitive disorder. C) bipolar disorder. D) acute withdrawal.

D) acute withdrawal.

An environmental emergency is an emergency involving: A) weather and terrain. B) temperature. C) atmospheric pressure. D) all are examples of an environmental emergency

D) all are examples of an environmental emergency

A factor that will contribute to the severity of heat illness is: A) general health. B) age. C) certain medications. D) all of the above

D) all of the above

You lose heat through respiration by: A) evaporation. B) radiation. C) convection. D) all of the above

D) all of the above

A condition that occurs as a result of panic disorder, phobia, or posttraumatic stress is termed: A) dementia. B) manic depression. C) delirium. D) anxiety.

D) anxiety

A state of uneasiness, discomfort, apprehension, and restlessness is called: A) dementia. B) anger. C) delirium. D) anxiety.

D) anxiety.

Hyperthermia may manifest itself as: A) heat stroke. B) heat cramps. C) heat exhaustion. D) any of the above

D) any of the above

Your patient is experiencing hallucinations and is sure that there are bugs crawling on the inside of the ambulance. You should: A) avoid recognizing the hallucination and act as if it is not happening. B) use the hallucination to your advantage in keeping the patient safe. C) agree with the patient, but assure the patient that they are safe. D) assure the patient that there are no bugs, but acknowledge the patient's feelings.

D) assure the patient that there are no bugs, but acknowledge the patient's feelings.

According to family, a 24-year-old woman suddenly began to act in a hostile manner. After assessing her, you determine that she cannot remember her name or present location and is drooling. Based on these findings, you should suspect a: A) psychological condition related to a traumatic experience. B) psychological emergency caused by alcohol consumption. C) behavioral emergency without physical cause. D) behavioral emergency resulting from a physical condition.

D) behavioral emergency resulting from a physical condition.

You are treating a patient who has been exposed to a cold environment without appropriate clothing. You would expect the: A) bloodflow to remain constant throughout the body. B) bloodflow to increase to the periphery. C) body to shunt blood to the skin to increase perfusion. D) body to shunt blood to the core to preserve organ function.

D) body to shunt blood to the core to preserve organ function.

An unrestrained female patient who was involved in a minor motorvehicle collision refuses EMS care or transport. She is seven months pregnant and is alert and oriented. When asked, she denies any complaint. Your BEST action would be to: A) make contact with her obstetrician B) have the police witness her signing the refusal C) have her follow up with her obstetrician D) contact medical direction for instructions

D) contact medical direction for instructions

An integral part of care for the patient suffering a psychological emergency should include: A) police presence. B) oxygen by nonrebreather mask. C) an authoritative presence. D) developing a rapport with the patient.

D) developing a rapport with the patient.

A 16-year-old boy has been inhaling paint fumes to "feel good." His grandmother called 911 when she found him doing this in the basement and he told her he was having a hard time breathing. He is alert and oriented, with a patent airway and adequate breathing. His radial pulse is strong and skin warm and dry. Your partner reports mild wheezing in both lungs. Vital signs are: pulse 92, respirations 20, blood pressure 148/62, and SpO2 98 percent on room air. He has no medical history. Appropriate initial care would include: A) positive pressure ventilation with supplemental oxygen. B) breathing treatment with a metered dose inhaler. C) oxygen at four liters per minute through a nasal cannula. D) high flow oxygen through a nonrebreather mask.

D) high flow oxygen through a nonrebreather mask.

A patient has been bitten by what appears to be a black widow spider. The bite to her left hand is dull and achy and she states that the muscles in her shoulders and back are starting to hurt. After ensuring that NO life-threatening conditions are present and administering oxygen, the AEMT should: A) pour a vinegar solution over the bite. B) massage the muscles of the back and shoulders. C) apply hot packs to the bite. D) keep the hand below the level of the heart.

D) keep the hand below the level of the heart.

A biological attack will likely be recognized​ by: A. the odor on the breath of patients B. the presence of strange or unusual odors C. blisters on the skin D. patient clusters after the incubation period

D.

When working a disaster where the incident command system has been​ established, what is most important for ALL responders to remember to ensure the most efficient scene management and​ teamwork? A. Each responder only reports to the direct supervisor. B. The AEMT must initiate incident command for patient care only. C. Each responder is must concentrate only on his or her assigned role and is only responsible for his or her own safety. D. Each responder must work within the incident command system structure and carry out the duties he or she is assigned.

D.

Which chemical​ agents, that are likely to be used in terrorist​ attacks, cause​ salivation, lacrimation,​ urination, defecation, gastric​ distress, and​ emesis? A. vomiting agents B. blood agents C. vesicants D. nerve agents

D.

Which of the following should you anticipate in the response to a natural​ disaster? A. Normal routes of ingress and egress may not be available. B. Hospitals in the area may be inoperable because of the event. C. Continued dangerous weather or earthquake aftershocks. D. all of the above.

D.

You are the first to arrive on the scene involving chemical weapons. Which of the following special teams should you contact to manage the​ scene? A. biological agent expert team B. search and rescue team C. law enforcement team D. hazardous materials team

D.

Which stage of radiation sickness includes experiences of nausea and​ vomiting, loss of​ appetite, and​ diarrhea? A. recovery B. illness C. latent D. prodromal

D. A person who receives a large dose of​ radiation, or even a smaller dose over a very short period of​ time, may develop acute radiation sickness​ (ARS). Signs and symptoms of ARS occur in four​ phases: prodromal,​ latent, illness, and recovery or death. In the prodromal​ stage, the patient experiences nausea and​ vomiting, loss of​ appetite, and diarrhea. Higher doses can cause​ fever, respiratory​ symptoms, and increased excitability. The symptoms last from hours to days. In the latent​ stage, the illness seems as if it is in remission and the patient appears to be healthy. The latent period is shorter than the prodromal phase. At higher​ dosages, the latent phase may not occur at all. In the illness​ stage, signs and symptoms recur. Depending on the​ dose, recovery or death occurs over the following weeks.

Which of the following could indicate chemical terrorism while you perform your windshield​ survey? A. dry powder covering the scene B. a large structure fire C. an unusually colored haze around the scene D. the presence of dead or incapacitated animals

D. A possible indication of chemical terrorism in your windshield survey is the presence of dead or incapacitated​ animals, such as​ livestock, flocks of​ birds, or fish kills. Although an unusual odor can tip you off to a possible chemical​ weapon, not all chemicals that could be used for a terrorism event have an odor or other physical properties that make recognizing the nature of the event easy.

What is the first step to preplanning for an MCI or​ disaster? A. assigning roles within the services responding B. testing the preplan C. developing a plan that outlines standard operating procedures D. performing a complete assessment of hazards in your area

D. Anything that can be planned in advance involving an MCI should be planned in advance. The first step to preplanning includes a complete assessment of potential​ hazards, both natural and​ man-made, that could occur in your area.

What considerations should you make when caring for patients who have survived a natural disaster that destroyed their​ neighborhood? A. You must be authoritative and calm down emotional patients. B. You must be prepared to allow law enforcement to control all behavioral emergencies. C. You must focus only on physical injuries. D. You must be prepared to give psychological support.

D. Disaster survivors may be disoriented and panicked. Anticipate the possibility of behavioral emergencies and the need for psychological​ support, including activation of disaster​ mental-health services.

Which of the following resources is best equipped to manage emotionally distraught or panicked patients following a​ disaster? A. law enforcement B. the responding ambulance unit C. the American Red Cross D. disaster​ mental-health services

D. Disaster​ mental-health services are a critical part of the response to natural disasters and terrorism events. Do not hesitate to make use of those services. This group will be best suited to handle distraught patients and have the resources available to properly care for them.

Where is the windshield assessment​ performed? A. from inside a law enforcement vehicle B. in the hot zone of an MCI C. ​nowhere: this is not part of scene​ size-up D. inside of the cab of the ambulance

D. If you are first on the scene and see signs that the event is an MCI as a result of natural disaster or​ terrorism, stay in your vehicle and perform a windshield assessment. That​ is, ask​ yourself: What can I see from the windows of my​ vehicle?

You are the first to arrive on the scene of a potential MCI that is the result of a bombing at a local university. What should you do​ first? A. Immediately begin patient triage and prioritize treatment. B. Set up incident command and report back to dispatch. C. Stage uphill and upwind from the incident and wait for the incident commander to arrive. D. Stay in your vehicle and perform a windshield assessment.

D. If you are first on the scene and see signs that the event is an MCI as a result of natural disaster or​ terrorism, stay in your vehicle and perform a windshield assessment. That​ is, ask​ yourself: What can I see from the windows of my​ vehicle? Relay the information to dispatch and establish command until the MCI or disaster plan can be implemented and you are relieved of command by designated personnel. Report the nature of the incident​ and, to the best of your​ ability, the number of patients involved and the need for additional resources. In cases where you are the first to​ respond, do not immediately rush to care for patients. Carefully assess the potential for danger at the scene. Depending on the nature of the​ situation, you must anticipate different hazards.

Which of the following has high potential to cause​ post-traumatic stress of the​ responder? A. working the scene of a daycare center that has been destroyed by a tornado B. working the scene of motor vehicle collision that involved multiple patient fatalities C. working the scene of a bombing during a large sporting event D. all of the above

D. Some aspects of health care and public safety create unique stressors. These include multiple casualty​ incidents, calls involving dead​ patients, and calls involving children. The responder must recognize the signs of stress and find healthy ways of coping with stressful situations.

Which of the following is an example of a biological weapon of mass​ destruction? A. organophosphates B. cyanide C. beta particle radiation D. Yersinia pestis

D. Some pathogens that could be used occur naturally. Others are known only to be produced in laboratories. For​ example, Yersinia​ pestis, the biologic agent that causes the bubonic form of the​ plague, occurs naturally in parts of the United States and affects several people each​ year, particularly in the Southwestern United States. Organophosphates and cyanide are examples of chemical​ weapons, and radiation is not a biological weapon.

While performing a scene​ size-up after a large tornado destroyed a rural​ community, you note downed power lines in the middle of the road. What should you immediately​ do? A. Check the lines to see if they are live. B. Call law enforcement to provide traffic control. C. Set up a road block with your ambulance. D. Contact the power company and inform them.

D. The power company is the most appropriate resource to manage downed power lines. You should never check power lines to see if they are charged. Law enforcement should be contacted to set up a road block and manage traffic flow only after the power company has been informed.

Which of the following is a section of the U.S. Department of Health and Human services that is responsible for managing the federal medical response to an emergency or​ disaster? A. Federal Emergency Management Agency B. National Highway Safety and Traffic Administration C. National Incident Command System D. National Disaster Medical System

D. You may become involved in an MCI outside your jurisdiction as part of mutual aid or because you opt to be active in the National Disaster Medical System​ (NDMS). NDMS is a section of the U.S. Department of Health and Human Services that is responsible for managing the federal medical response to an emergency or disaster. It consists of locally based teams that respond jointly where disaster medical assistance is required.

The AEMT would recognize which one of the following statements as the BEST example of the use of a weapon of mass​ destruction?

Death of 49 people in a crowded mall following intentional infection with unknown bacteria.

You arrive on the scene and smell gasoline. Which of the following devices might be​ involved?

Incendiary

A localized terrorist attack involves pepper spray being released at a high school dance. As you​ respond, the incident commander reports approximately 50 people have been exposed and are calling for the county disaster plan to be implemented. Given the nature of the​ weapon, what type of symptoms would you expect to​ encounter?

Respiratory​ distress, burning of the eyes and other mucous membranes

You are on the scene of an accident where patients are exhibiting the following​ symptoms: respiratory​ distress, burning of the eyes and other mucous​ membranes, coughing,​ nausea, and vomiting. What type of chemical weapon might be​ suspected?

Riot control agents

The natural occurrence of this biologic agent was eradicated years​ ago, yet a supply of the virus could potentially be stockpiled for use as a biologic weapon.

Smallpox

First responder operations

What is the level of training for those who respond to the release of hazardous materials in order to protect people, property, and the environment?

protective gear must be worn.

When working in the "warm" zone, the EMT must remember that:

"Life or health should never be risked if the threat is only to the environment."

Which one of the following statements shows that the EMT has a proper understanding of basic operations at the scene of a hazardous materials emergency?

contamination

While he is moving a container of a radioactive liquid, some of the liquid splashed onto the technician's arm. This situation is best be described as:

contamination.

While he is moving a container of a radioactive liquid, some of the liquid splashed onto the technician's arm. This situation is best be described as:

clothing

You are on a scene in which a radioactive material is emitting alpha rays. As a knowledgeable EMT, you recognize that the most appropriate shielding for this material is:

evacuate people from cars near the tanker

You are the first emergency personnel on the scene of an overturned tanker truck. The driver is in the cab and appears motionless. It also appears that a liquid is leaking from the rear-valve assembly of the tanker. After properly positioning your ambulance, your immediate priority should be to:

Cab of the truck

You are working the scene of a collision involving a tractor trailer that is hauling an unknown chemical. After it has been determined that no material has been spilled, the rescue chief asks you to get the truck's shipping papers. Where would these most likely be located?

flammable but represents a low health hazard

You have arrived on the scene of a chemical leak at an industrial park. The placard has a "1" in the blue diamond and a "4" in the red diamond, which means the material is:

flammable but represents a low health hazard.

You have arrived on the scene of a chemical leak at an industrial park. The placard has a "1" in the blue diamond and a "4" in the red diamond, which means the material is:

Move the ambulance upwind of the scene.

You have arrived on the scene where a tractor trailer has overturned. As you pull onto the scene, the driver of the truck approaches and informs you that he is hauling radioactive medical wastes and warns you that there is a car underneath the overturned trailer. Which one of the following should you do immediately?

Recognize a potential hazardous materials emergency and call for the appropriate resources.

You have been trained to the level of OSHA "First Responder Awareness." In relation to this level of training, which one of the following activities are you qualified to perform?

Nerve agents inhibit acetylcholinesterase resulting in the accumulation of​ ________________ in the synapse.

acetylcholine.

To treat nerve agent​ poisoning, you​ should:

administer atropine and Pralidoxime​ (2-PAM) chloride.

You are teaching an EMT class about the role of an EMS response to a terror attack involving weapons of mass destruction​ (WMD). A student asks you what constitutes a WMD. You should respond by stating that a WMD is​ any:

agent that is used to cause indiscriminate death and destruction.

You are working at a​ multiple-casualty incident​ (MCI) involving the use of an incendiary device that was thrown into a large group of people. The victims could be expected to​ have:

burn injuries

The safest assumption to make on the scene of a detonated device is that​ the:

detonation of the first device is evidence that the bomber has the ability to deliver a secondary device.

At the scene of a​ multiple-casualty incident caused by a weapon of mass​ destruction, the​ AEMT's FIRST priority​ is:

ensuring the safety of yourself and your partner.

Weapons of mass destruction include all of the following​ EXCEPT:

experimental. This is the correct answer.

Vomiting agents mostly​ cause:

irritation of the upper respiratory tract and the eyes.

The purpose of the National Disaster Medical System is​ to:

manage the federal medical response to an emergency or disaster.

Yersinia pestis can be used in​ terrorism, although it occurs naturally in the​ U.S., and​ causes:

plague.

Exposure to chemical vesicants​ (mustard) agents results​ in:

slowly developing symptoms.

A type of biological agent​ is:

smallpox.

Of the​ following, the MOST virulent biologic agent​ is:

smallpox.

Incubation periods for biologic​ agents:

vary

A​ ________ is a chemical agent that causes blisters to form where exposure occurs.

vesicant

A class of diseases that includes the ebola virus is known​ as:

viral hemmorhagic fever.


Ensembles d'études connexes

🍡💜🍡Endocrine System🍡💜🍡

View Set

pharmacology online practice 2017 B

View Set

AIT 524 - Week 8 (Table Creation and Management)

View Set

Art History: Romanesque Europe - Chapter 12

View Set